You are on page 1of 179

PREFACE

This book is for you since you aspire to achieve a perfect score on the ITB AQ. With consistent
use of this book as part of your plan of study, you could achieve Week after week, read through the
strategies and work hard on the practice exercise. Make sure that you’re able to achieve the best
possible state of preparation, allowing you to maximize your score potential - no matter if your
actual aptitude has been measured. The higher your ITB AQ score, the better your chances of
admission will be for a respected.
This test is designed to judge your verbal and mathematical ability in the ways that colleges
feel is vital to the success of first year college students. To some extent, you have already gradually
obtained these abilities over the length of your academic career. However, what you probably have
not yet become familiar with is the capability to use these abilities for the purpose of maximizing
performance within the complex and profound environment of a standardized, skills-based
examination.
There are different strategies, mindsets and perspectives that you will be required to apply
throughout the SAT. You’ll need to be prepared to use your whole brain as far as thinking and
assessment is concerned, and you’ll need to do this in a timely manner. This is not something you
can learn from taking a course or reading a book, but it is something you can develop through
practice and concentration.
This guide provides you with the professional instruction you require for understanding the ITB
AQ test. Covered are all aspects of the test and preparation procedures that you will require
throughout the process. Upon completion of this guide, you’ll have the confidence and knowledge
you need for maximizing your performance on your ITB AQ test.

Best Regards,

Sang Juara School

BERANI BERJUANG|1
GENERAL INFORMATION ITB
INTERNATIONAL UNDERGRADUATE PROGRAM

ITB International Undergraduate Program is an educational program that requires students to be


active in various global academic activities. Students taking part in this program will experience the
atmosphere of international education. In addition, Undergraduate International Program is part
of ITB's efforts to serve prospective Indonesian students from non-national curriculum high
schools. It is hoped that this broader service will provide good opportunities for Indonesian citizens
abroad to study at ITB.

ITB International Undergraduate Program is open for both Indonesian and foreign citizenships
students. In 2022, Institut Teknologi Bandung (ITB) will accept new students for this program
through two mechanisms: ITB International Class and ITB International Track.

STUDY PROGRAMS OFFERED


INTERNATIONAL CLASS INTERNATIONAL TRACK
Pharmaceutical Science and Technology Geological Engineering
(School of Pharmacy) (Faculty of Earth Sciences and Technology)
Clinical and Community Pharmacy Geodesy and Geomatics Engineering
(School of Pharmacy) (Faculty of Earth Sciences and Technology)
Chemical Engineering Engineering Physics
(Faculty of Industrial Technology) (Faculty of Industrial Technology)
Architecture
(School of Architecture, Planning, and Policy
Environmental Engineering Development)
(Faculty of Civil and Environmental
Engineering) Urban and Regional Planning
(School of Architecture, Planning, and Policy
Development)
Electrical Power Engineering
Mechanical Engineering (School of Electrical Engineering and
(Faculty of Mechanical and Aerospace Informatics)
Engineering) Telecommunication Engineering
(School of Electrical and Informatics)
Materials Engineering
Aerospace Engineering (Faculty of Mechanical and Aerospace
(Faculty of Mechanical and Aerospace Engineering)
Engineering) Fine Arts
(Faculty of Art and Design)
Craft
Management (Faculty of Art and Design)
(School of Business & Management) Interior Design
(Faculty of Art and Design)
Visual Communication Design – Digital Visual
Narrative
Entrepreneurship (Faculty of Art and Design)
(School of Business & Management)
Product Design
(Faculty of Arts and Design)

2|IUP ITB
SCHEDULE
ITB IUP
PERIOD 1 PERIOD 2 PERIOD 3 (EXTENDED)
PROGRAM
 Foreigners (WNA)  Foreigners (WNA)
 Indonesian citizens  Indonesian citizens
 Foreigners (WNA) (WNI) from schools (WNI) from schools
 Indonesian citizens outside the territory outside the territory of
(WNI) from schools of the Republic of the Republic of
outside the territory of Indonesia (non SRI). Indonesia (non SRI).
the Republic of  Indonesian citizens  Indonesian citizens
Participant
Indonesia (non SRI). (WNI) from schools (WNI) from schools
 Indonesian citizens with a non-national with a non-national
(WNI) from schools curriculum curriculum
with a non-national  Indonesian citizens  Indonesian citizens
curriculum (WNI) from schools (WNI) from schools
with a national with a national
curriculum curriculum
Account
February - March April - May June - July
Registration
Documents
Completion,
Account
Finalization, February - March April - May June - July
and
Registration
Payment
ITB AQ Test
Registration
- April - May June - July
and
Payment
ITB AQ Test - June July
Drawing
April June July
Test
Selection
Result April June July
Notification
New
Student
April June - July July
Registration
(online)
Tuition Fee
Payment April July August
Schedule
*The schedule may be subject to change. Please visit https://admission.itb.ac.id page regularly to
get the latest information.
*Definition of non-national curriculum high school is if its students are not eligible for SNMPTN or
SBMPTN

BERANI BERJUANG|3
ITB ACADEMIC QUALIFICATION TEST (ITB AQ TEST)

To overcome difficulties to obtaining SAT result due to the Covid-19 Pandemic condition, ITB will
conduct ITB Academic Qualification Test (ITB AQ Test) as replacement of SAT result.
 ITB AQ Test is conducted in English in the form of Computer Based Test (CBT), in in a 110-
minutes session.
 The test intended to assess individual’s learning potential or learning ability, consisting 3
subtest
1. Verbal Test
Measuring an individual’s ability to solve problems that associated with words or
language.
2. Quantitative Test
Measuring individual’s ability in regular mathematical, understanding the mathematical
concepts and applying logic numbers to solve problems.
3. Reasoning Test
Measuring individual’s ability to select and organize relevant information in problem
solving.

ITB AQ TEST ONLINE APPLICATION


 Registration Fee: IDR 1,250,000.- per submission
 Document Requirement:
 High School Graduation Certificate/Diploma, or Official Statement from the High School
Headmaster confirming that the applicant has/will be graduated from high school.
 Identity Card (KTP/Passport/other ID Card with photograph)
 Recent color photograph
 ITB AQ Test registration considered complete when payment has been made.

ITB ACADEMIC QUALIFICATION TEST SCHEDULE


FIRST TERM SECOND TERM
ACTIVITY
(SECOND ADMISSION PERIOD) (THIRD ADMISSION PERIOD)
Online Submission April - May June - July
ITB AQ Test (online) June 2023 July
ITB AQ Test Result June 2023 July

4|IUP ITB
TABLE OF CONTENTS

PREFACE ________________________________________________________________________ 1
GENERAL INFORMATION ITB INTERNATIONAL UNDERGRADUATE PROGRAM ________________ 2
ITB ACADEMIC QUALIFICATION TEST (ITB AQ TEST) _____________________________________ 4
TABLE OF CONTENTS ______________________________________________________________ 5
VERBAL - SUBJECTS _______________________________________________________________ 6
A. SENTENCE LEVEL READING __________________________________________________ 6
B. PARAGRAPH-LEVEL READING ________________________________________________ 7
C. LONG-PASSAGE READING ___________________________________________________ 7
VERBAL REASONING - SUBJECT ____________________________________________________ 10
A. CRITICAL REASONING _____________________________________________________ 10
B. SENTENCE CORRECTION ___________________________________________________ 10
QUANTITATIVE - SUBJECTS _______________________________________________________ 11
A. NUMBER AND OPERATION _________________________________________________ 11
B. LINEAR EQUATIONS _______________________________________________________ 13
C. COMPOSITION FUNCTIONS _________________________________________________ 15
D. QUADRATIC EQUATIONS ___________________________________________________ 16
E. LINEAR INEQUALITIES AND GRAPH ___________________________________________ 19
F. STATISTIC AND GRAPHICAL DISPLAY __________________________________________ 20
G. GEOMETRY ______________________________________________________________ 22
H. TRIGONOMETRY _________________________________________________________ 28
I. EXPONENT ______________________________________________________________ 31
J. RATES AND RATIO ________________________________________________________ 33
K. PERCENTAGE ____________________________________________________________ 34
L. PROBABILITY ____________________________________________________________ 35
M. SURFACE AREAS AND VOLUMES _____________________________________________ 36
QUANTITATIVE REASONING - SUBJECT ______________________________________________ 38
A. QUANTITATIVE COMPARASION _____________________________________________ 38
B. DATA-SUFFICIENCY _______________________________________________________ 38
C. ESSAY __________________________________________________________________ 38
SET-1 _________________________________________________________________________ 39
SET-2 _________________________________________________________________________ 53
SET-3 _________________________________________________________________________ 67
SET-4 _________________________________________________________________________ 82
SET-5 _________________________________________________________________________ 96
SET-6 ________________________________________________________________________ 110
SET-7 ________________________________________________________________________ 123
SET-8 ________________________________________________________________________ 137
SET-9 ________________________________________________________________________ 151
SET-10 _______________________________________________________________________ 165

BERANI BERJUANG|5
VERBAL - SUBJECTS

A. SENTENCE LEVEL READING


STRATEGIES
B Bring your word to the blank.
L Locate a context clue.
A Assign a positive or negative value to the blank.
N Never do’s!
K Keep level-headed.
S Say the sentence to yourself to ensure that it makes sense

BRING YOUR WORD TO THE BLANK


Read through
Let your own word roll off your tongue as you carefully read through the the sentence,
entire sentence. When you practice at home, read each sentence aloud to and predict a
absorb its sense. Go with your hunch and supply your own word for every word that you
blank. think would fit
in the blank.

LOCATE A CONTEXT CLUE


When you work on hard sentence completions, filling in the blanks has nothing to do with
guessing or with what sounds right! You must have a method to find the best answer choice. If you
have no clue as to what type of word the blank requires, then get a clue! A context clue!

ASSIGN A POSITIVE (+) OR NEGATIVE (–) VALUE TO THE BLANK


If you feel tongue-tied when it comes to expressing your own word for
Word values
the blank or if you can’t seem to find a context clue, then simply jot down a move you closer
positive (+) or negative (–) sign in the blank to indicate your feel for the needed to the right
word. Even if it’s hard for you to come up with a denotation (dictionary answer.
definition or literal meaning), you can very often come up with the blank’s
connotation—its feel or undertone.

NEVER DO’S!
When you’re filling in the blank, You’re looking for the best choice, that is, a choice firmly
supported by a context clue and/or your word prediction Never settle for the first answer choice
that seems to fit. If choice B sounds good, don’t stop there. Search thoroughly, leaving no stone
unturned.
Study the top-tier
KEEP LEVEL-HEADED vocabulary lists at
the end of this book.
This is helpful to know as you work through the sentence-completion
Start with the four
section since these questions start off easy (levels 1 and 2) and then become letter words, and
medium in difficulty (level 3); at the end of the section are the hardest work your way up to
questions (levels 4 and 5). the heavy hitters

SAY THE SENTENCE TO YOURSELF TO ENSURE THAT IT MAKES SENSE


The acid test: Read the sentence as you have completed it. Does it make sense? A final reading
of the sentence is recommended before you move on to the next question. Make sure, of course,
that you have prudently considered all five answer choices
6|IUP ITB
B. PARAGRAPH-LEVEL READING
STRATEGIES
P Paraphrase as you read
P Predict the answer.
P Pare down the answer choices.
P Pick the best answer.

PARAPHRASE
As you read, translate the author’s language into your own words. Start by paraphrasing
phrases, then go on to full sentences. Soon you’ll work up the stamina to paraphrase several
sentences at a time. Paraphrasing is a valuable skill that helps you build meaning, layer by layer, as
you read.

PREDICT
As you read each question, predict what the answer will be. Predict before you read the answer
choices and you will put yourself in the driver’s seat. Predict before considering the answer choices
and you will prevent the befuddlement that sometimes comes with reading several answer choices
that sound convincing.

PARE DOWN
Cross off choices that you can rule out for one reason or another. If even a small part of an
alluring answer is not supported by the passage, then that choice should be scrapped

PICK
Choose the very best answer, never settling for second best. On easy and medium-level-of
difficulty questions, most test takers can reduce the answer choices to two. What remains? The
best answer and the “runner-up.” Selecting the second-best answer (the runner-up) may deserve
a pat on the back, but you will lose one-fourth point for the wrong answer.

C. LONG-PASSAGE READING

STRATEGIES
R Read longer passages piece by piece.
E Extract the main idea from the passage.
A Answer line-reference and sidebar questions first.
D Delve into higher-order thinking questions.
I Identify the author’s mood and tone as you read.
N Narrow in on the best answer choice.
G Global questions should be answered at the very end

BERANI BERJUANG|7
READ LONGER PASSAGES PIECE BY PIECE
Breaking a long passage into “pieces” is one way to downsize a
Do not read the
potentially daunting task. You are probably a strong and avid reader;
entire long passage
nevertheless, breaking a long passage into pieces gives you an edge by at once. Likewise,
helping you to stay focused as you read and to answer the questions that don’t answer all the
correspond to each piece. questions in one
spurt. Instead,
Do not read the entire long passage at once. Likewise, don’t answer
follow a strategic
all the questions in one spurt. Instead, follow a strategic and pragmatic and pragmatic
process or “modus operandi.” process or “modus
operandi.”
EXTRACT THE MAIN IDEA FROM THE PASSAGE
Read with your eye peeled for the main idea. Cultivate this habit as you practice reading long
passages. Knowing the main idea is very helpful in answering questions involving the author’s main
purpose, the best title for the passage, and the author’s point of view.

ANSWER LINE-REFERENCE AND SIDEBAR QUESTIONS FIRST


Before you begin reading the passage piece by piece, you should skim
the questions for line numbers and mark the text accordingly. Numbers Answer the more
straightforward,
within questions signal that you are going to be asked about a specific line reference
area of the text. According to the line numbers given in the questions, questions first. Then
you should mark the passage with “sidebars” and “line reference” answer the higher-
order inference
questions that
DELVE INTO HIGHER-ORDER THINKING QUESTIONS typically take more
Pace yourself by answering higher-order thinking questions (time- time.
sapping doozies) after you have answered the “directed” questions for
which you have marked sidebars and line references. Higher-order questions can be recognized by
their use of words and phrases such as infer, suggests, implies, it can be concluded that, it seems
that, most closely means.

IDENTIFY THE AUTHOR’S MOOD AND TONE AS YOU READ


More often than not, the problem is not that you don’t get the author’s tone or mood—it’s
that you may not know the word that describes what you are sensing. Challenge yourself to learn
this additional tier of vocabulary. As you read the passages, be aware of expressions of tone and
mood so that you’ll be prepared to answer related questions.

THE VOCABULARY THE VOCABULARY


MEANING MEANING
OF TONE OF TONE
Admonishing warning Explanatory explaining
Aloof detached Inquisitive asking questions
being sorry; admitting
Apologetic Nostalgic remembering the past
fault
Assertive strongly stating Penitent remorseful
Cautious hesitant Reflective remembering the past
Confrontational argumentative Sardonic sarcastic
Contrite remorseful Scathing harsh

8|IUP ITB
Cynical seeing worst in people Scholarly intellectual, cerebral
protecting one’s point of
Defensive Sensationalistic exaggerating emotions
view
Derisive ridiculing Skeptical doubtful
Empathetic having feeling for others Tentative hesitant
mildly suggested;
Emphatic stressing one’s points Understated
unstressed
Entreating begging, pleading Why bitter or ironic

THE VOCABULARY THE VOCABULARY


MEANING MEANING
OF MOOD OF MOOD
Amusement humor Irony incongruity
derision; insincere
Bemusement puzzlement Mockery
imitation
realism; factualism;
Detachment lack of concern Objectivity
having no opinion
Disdain scorn Optimism positive thinking
Disparagement belittlement Pessimism negative thinking
self-satisfaction;
Indignation anger Smugness
complacency

NARROW IN ON THE BEST ANSWER CHOICE


Narrow in on the best answer choice by using the good old, tried-and-true process of
elimination. First, cross off answer choices that are not supported in any way, shape, or form by
the passage. These are “bogus” or “fluff” answer choices that are not confirmed by something,
somewhere, that the author says. Also, avoid picking answers just because they appear fair,
reasonable, true, or intelligent.
Use your active
GLOBAL QUESTIONS SHOULD BE ANSWERED AT THE VERY END pencil to underline
Global questions are those that focus on the overarching ideas of key words in the
the passage. In other words, they are based on the whole passage. You question itself. This
discipline will keep
should save these questions until the end, after you have digested the you focused on what
passage as a whole. Be aware that global questions may appear you are supposed to
anywhere in the lineup of critical reading questions. answer

BERANI BERJUANG|9
VERBAL REASONING - SUBJECT

A. CRITICAL REASONING
At a quick glance, the Critical Reasoning questions may look similar to the Reading
Comprehension questions; however, they are testing two different skills. While Reading
Comprehension is testing your ability to break down the text, Critical Reasoning is assessing your
ability to evaluate an argument being made. As in a Reading Comprehension question, a Critical
Reasoning question will start with a text for you to read. This text will present an argument and is
generally shorter than a Reading Comprehension text. After reading the prompt, you are to answer
a multiple-choice question regarding information that can weaken or strengthen the argument
being made.
SOLUTION STRATEGIES
#1 STRATEGIES
Skim the text for key words which will point to the claim or conclusion being made in
the passage.
#2 STRATEGIES
Find the conclusion the author is drawing in the passage provided.
#3 STRATEGIES
Once you have found the conclusion, you should put it into your own words.
#4 STRATEGIES
Reread the text with the question and conclusion in mindSENTENCE CORRECTION

B. SENTENCE CORRECTION
Sentence Correction questions assess the test-taker's grasp on grammar rules in written
standard English. You will be presented with a sentence, likely quite wordy and complicated. Some
or all of the sentence will be underlined, and you will be presented with five multiple-choice
answers. Your objective is to choose the option that is the grammatically correct version of the
sentence. The first option is always the same as the original sentence, followed by four options
changing one ore more parts of the underlined section.

SOLUTION STRATEGIES
#1 STRATEGIES
Simplify the sentence given and look for 'extra' details. First, you can look for information
offset by commas.
#2 STRATEGIES
Remove adjective and adverb phrases to continue to distill the sentence down to its main
idea.
#3 STRATEGIES
You can spot differences in the answers and isolate groups to quickly eliminate the wrong
answers.

10 | I U P I T B
QUANTITATIVE - SUBJECTS

A. NUMBER AND OPERATION

Variables and Expressions


In algebra, variables are symbols used to represent unspecified numbers or values. An algebraic
expression is a collection of numbers, variables, operations, and grouping symbols.
Verbal Phrase Algebraic Expression
The sum of twenty and a number 𝑛 20 + 𝑛
Addition Nine more than twice a number 𝑎 2𝑎 + 9
A number 𝑚 increased by 12 𝑚 + 12
𝑥−7
The difference between a number 𝑥 and 7
𝑏−3
Three less than a number 𝑏
Subtraction 3𝑘 − 5
Three times a number 𝑘 decreased by five
9−𝑛
Nine minus a number 𝑛

11𝑧 3
Eleven times 𝑧 to the third power −2𝑛
Multiplication The product of −2 and a number 𝑛 2
𝑥
Two thirds of a number 𝑥 3

𝑛/8
𝑛 divided by 8
Division 15
The quotient of fifteen and a number 𝑑
𝑑

Exponents and Order of Operations

An expression like 35 is called a power. The number 3 is the base, and the number 5 is the exponent.
35 = 3 ∙ 3 ∙ 3 ∙ 3 ∙ 3
To evaluate an expression involving more than one operation, we agree to perform operations
in the following order.

Order of Operations
1. Simplify the expressions inside grouping symbols, such as parentheses, brackets, and fraction
bars.
2. Evaluate all powers.
3. Do all multiplications and divisions in order from left to right.
4. Do all additions and subtractions in order from left to right.
BIDMAS
B Brackets 10 × (4 + 2) = 10 × 6 = 60
I Indices 5 + 22 = 5 + 4 = 9
D Division 10 + 6 ÷ 2 = 10 + 3 = 13
M Multiplication 10 − 4 × 2 = 10 − 8 = 2
A Addition 10 × 4 + 7 = 40 + 7 = 47
S Subtraction 10 ÷ 2 − 3 = 5 − 3 = 2

B E R A N I B E R J U A N G | 11
Simplifying Algebraic Expressions

Important Terms Examples


Term
A number, a variable, or a product or quotient of 5, 𝑥, 7𝑎, 𝑏 2 , 𝑎𝑛𝑑 2𝑚3 𝑛 are all terms
numbers and variables.
Coefficient of a term
in 8𝑥 2 𝑦, the coefficient is 8
A number that multiplies a variable.

Distributive Property
𝑎(𝑏 + 𝑐) = 𝑎𝑏 + 𝑎𝑐 𝑎(𝑏 − 𝑐) = 𝑎𝑏 − 𝑎𝑐

Commutative Property
𝑎 =𝑏 =𝑏+𝑎 𝑎∙𝑏 =𝑏∙𝑎

Associative Property
(𝑎 + 𝑏) + 𝑐 = 𝑎 + (𝑏 + 𝑐) (𝑎𝑏)𝑐 = 𝑎(𝑏𝑐)

Rational, Irrational, and Decimal

Important Terms Examples


natural numbers {𝟏, 𝟐, 𝟑, . . . }
whole numbers {0, 1, 2, 3, . . . }
integers {. . . , −3, −2, −1, 0, 1, 2, 3, . . . }
𝑎
A rational number is one that can be expressed as a ratio , where 𝑎 and
𝑏
rational numbers 2
𝑏 are integers and 𝑏 is not zero, such as −3.72, − , 0, 2, 𝑎𝑛𝑑 4.23.
3

Any real number that is not rational is irrational. −√3, √2, 𝑎𝑛𝑑 𝜋 are
irrational numbers
irrational.

Rounding Decimals
To round a decimal to the desired place, underline the digit in the place to be rounded.
1. If the digit to the right of the underlined digit is 5 or more, increase the underlined digit by one
(round up).
2. If the digit to the right of the underlined digit is less than 5, leave the underlined digit as it is
(round down).
3. Drop all digits to the right of the underlined digit.

12 | I U P I T B
B. LINEAR EQUATIONS

Solving Equations
To solve an equation means to find all values of the variable that make the equation a true
statement. One way to do this is to isolate the variable that has a coefficient of 1 onto one side of
the equation. You can do this using the rules of algebra called properties of equality.

Properties of Equality Symbols Examples


If 𝑥 − 3 = 5, then (𝑥 − 3) + 3 =
1. Addition Property If 𝑎 = 𝑏, then 𝑎 + 𝑐 = 𝑏 + 𝑐
(5) + 3
If 𝑥 + 2 = 6, then (𝑥 + 2) − 2 =
2. Subtraction Property If 𝑎 = 𝑏, then 𝑎 − 𝑐 = 𝑏 − 𝑐
(6) − 2
1 1
3. Multiplication Property If 𝑎 = 𝑏, then 𝑐𝑎 = 𝑐𝑏 If 𝑥 = 3, then 2 ∙ 𝑥 = 2 ∙ 3
2 2
𝑎 𝑏 3𝑥 15
4. Division Property If 𝑎 = 𝑏 and 𝑐 ≠ 0, then = If 3𝑥 = 15, then =
𝑐 𝑐 3 3

Solving Equations with Variables on Both Sides


1. Use the Addition or Subtraction Property of Equality to write an equivalent equation that has
all of the variables on one side.
2. Use the Multiplication or Division Property of Equality to simplify the equation if necessary.
When solving equations that contain grouping symbols, use the Distributive Property to
remove the grouping symbols

Rate of Change and Slope

The slope 𝒎 of a line though (𝒙𝟏 , 𝒚𝟏 ) and The standard form of a linear equation of a
(𝒙𝟐 , 𝒚𝟐 ) line

𝑦2 −𝑦1 𝑐ℎ𝑎𝑛𝑔𝑒 𝑖𝑛 𝑦 𝑟𝑖𝑠𝑒 𝐴𝑥 + 𝐵𝑦 = 𝐶


𝑚= = = .
𝑥2 −𝑥1 𝑐ℎ𝑎𝑛𝑔𝑒 𝑖𝑛 𝑥 𝑟𝑢𝑛
which 𝐴 , 𝐵 , 𝑎𝑛𝑑 𝐶 are integers.

Slope-intercept form Point-slope form


𝒚 = 𝒎𝒙 + 𝒃 𝑦 − 𝑦1 = 𝑚(𝑥 − 𝑥1 )
Description Description
𝒎 : 𝒕𝒉𝒆 𝒔𝒍𝒐𝒑𝒆 (𝑥1 , 𝑦1 ) : 𝑡ℎ𝑒 𝑐𝑜𝑜𝑟𝑑𝑖𝑛𝑎𝑡𝑒 𝑜𝑓 𝑎 𝑝𝑜𝑖𝑛𝑡 𝑜𝑛 𝑡ℎ𝑒 𝑙𝑖𝑛𝑒
𝒃 : 𝒚 − 𝒊𝒏𝒕𝒆𝒓𝒄𝒆𝒑𝒕 𝑚 : 𝑠𝑙𝑜𝑝𝑒 𝑜𝑓 𝑡ℎ𝑒 𝑙𝑖𝑛𝑒

B E R A N I B E R J U A N G | 13
Parallel and Perpendicular Lines

Solving Systems of Linear Equations

Solving
Linear
Systems by
Graphing

Solving 1. Solve one of the equations for one of its variables.


Linear 2. Substitute the resulting expression into the other equation and solve.
Systems by 3. Substitute the resulting value into either of the original equations and solve for
Substitution the other variable
1. Arrange the equations with the like terms in columns.
Solving 2. Multiply one or both equations to obtain new coefficients for 𝑥 (𝑜𝑟 𝑦 ) that are
Linear opposites.
Systems by 3. Add the equations and solve for the remaining variable.
Elimination 4. Substitute this value into either of the original equations and solve for the other
variable.

14 | I U P I T B
C. COMPOSITION FUNCTIONS

Relations and Functions


A coordinate plane is formed by the horizontal line called the
𝑥 − 𝑎𝑥𝑖𝑠 and the vertical line called the 𝑦 − 𝑎𝑥𝑖𝑠, which
meet at the origin (0, 0). The axes divide the plane into four
parts called quadrants.

An ordered pair gives the coordinates and location of a point.


The ordered pairs (2, 1), (−2, 2), (−4, −4), and (3, −2) are
located in 𝑄𝑢𝑎𝑑𝑟𝑎𝑛𝑡 𝐼, 𝑄𝑢𝑎𝑑𝑟𝑎𝑛𝑡 𝐼𝐼, 𝑄𝑢𝑎𝑑𝑟𝑎𝑛𝑡 𝐼𝐼𝐼, and
𝑄𝑢𝑎𝑑𝑟𝑎𝑛𝑡 𝐼𝑉 respectively.

Relation Function
A relation is a set of A function is a special type of
ordered pairs. A relation relation in which each element
can be represented by a of the domain is paired with
graph, a table, or a exactly one element of the
mapping. range

Function Values
Equations that are functions can be written in a form called function notation.

The equation 𝑦 = 2𝑥 + 3 is written as (𝑥) = 2𝑥 + 3 .

The function value of 𝑓 at 𝑥 = 𝑐 is denoted as 𝑓(𝑐). For instance, if 𝑓(𝑥) = 5𝑥 − 3, 𝑓(2) is the
value of 𝑓 at 𝑥 = 2 and 𝑓(2) = 5(2) − 3 = 7

Composition of Functions
Given the two functions 𝑓 and 𝑔, the composite function, denoted by 𝑓 ∘ 𝑔, is defined As

(𝑓 ∘ 𝑔)(𝑥) = 𝑓 ∘ 𝑔(𝑥) = 𝑓(𝑔(𝑥))

In order for a value of 𝑥 to be in the domain of 𝑓 ∘ 𝑔, two conditions must be satisfied:


1. 𝑥 must be in the domain of 𝑔.
2. 𝑔(𝑥) must be in the domain of 𝑓

Recursive Formula
A recursive formula for a sequence describes how to find the 𝑛 term from the term(s) before it.
A recursive formula consists of two parts:
1. An initial condition that shows where the sequence starts.
2. A recursion equation that shows how to find each term from the term(s) before it.

B E R A N I B E R J U A N G | 15
D. QUADRATIC EQUATIONS

Graphs of Quadratic Equations

The standard form of a quadratic function


𝑓(𝑥) = 𝑎𝑥 2 + 𝑏𝑥 + 𝑐

The graph of a quadratic function is called a parabola.

If 𝑎 > 0, graph opens upward and the vertex is


the minimum point.
If 𝑎 < 0, graph opens downward and the vertex is
the maximum point.

The equation of the axis of symmetry The vertex form of a quadratic function
𝑏 𝑓(𝑥) = 𝑎(𝑥 − ℎ)2 + 𝑘
𝑥=−
2𝑎

The coordinates of the vertex The factored form of a quadratic function


𝑏 𝑏 𝒇(𝒙) = 𝒂(𝒙 − 𝒃)(𝒙 − 𝒄)
(− , 𝑓(− ))
2𝑎 2𝑎

Factoring Trinomials: 𝒙𝟐 + 𝒃𝒙 + 𝒄
To factor trinomials of the form 𝑥 2 + 𝑏𝑥 + 𝑐, you need to find two integers with 𝑠𝑢𝑚 𝑏 and
𝑝𝑟𝑜𝑑𝑢𝑐𝑡 𝑐. When two binomials (𝑥 + 𝑚) and (𝑥 + 𝑛) are multiplied using FOIL method,
(𝑥 + 𝑚)(𝑥 + 𝑛) = 𝑥 2 + 𝑛𝑥 + 𝑚𝑥 + 𝑚𝑛, or 𝑥 2 + (𝑚 + 𝑛)𝑥 + 𝑚𝑛. So, you can 𝑥 2 + 𝑏𝑥 + 𝑐 as
(𝑥 + 𝑚)(𝑥 + 𝑛), then as 𝑚 + 𝑛 = 𝑏 and 𝑚𝑛 = 𝑐

Factoring Trinomials: 𝒂𝒙𝟐 + 𝒃𝒙 + 𝒄


To factor trinomials of the form 𝑥 2 + 𝑏𝑥 + 𝑐, in which 𝑎 ≠ ±1, you need to find two integers 𝑚
and 𝑛 with sum 𝑏 and product 𝑎𝑐 . Then write 𝑏𝑥 as 𝑚𝑥 + 𝑛𝑥 and use the method of factoring by
grouping.

Factoring Differences of Squares and Perfect Square Trinomials

Factoring Differences of Squares and Perfect Square Trinomials


Perfect Square Trinomials Difference of Squares
𝒂𝟐 + 𝟐𝒂𝒃 + 𝒃𝟐 = (𝒂 + 𝒃)𝟐
𝑎2 + 𝑏 2 = (𝑎 + 𝑏)(𝑎 − 𝑏)
𝒂𝟐 − 𝟐𝒂𝒃 + 𝒃𝟐 = (𝒂 − 𝒃)𝟐

16 | I U P I T B
Solving Quadratic Equations by Completing the Square
Definition of Square Root
For any number 𝑎 > 0, if 𝑥 2 = 𝑎, then 𝑥 is a square root of 𝑎 and 𝑥 = ±√𝑎 .

Completing the Square


1 Subtract 𝑐 from each side: 𝑎𝑥 2 + 𝑏𝑥 = −𝑐
𝑏 𝑐
2 If 𝑎 ≠ 1, divide both sides by a : 𝑎 ∶ 𝑥 2 + 𝑥 = −
𝑎 𝑎

1 𝑎 𝑏 𝑏 𝑏 𝑐 𝑏
3 Add of square which is ( )2 , to both sides: 𝑥 2 + 𝑥 + ( )2 = − + ( )2
2 𝑏 𝑎 𝑎 2𝑎 𝑎 2𝑎
𝑏 2 𝑐 𝑏
4 You have completed the square: (𝑥 + ) = − + ( )2
2𝑎 𝑎 2𝑎

𝑏 𝑐 𝑏 𝑏 𝑐 𝑏
5 Square root each side: 𝑥 + = ±√− + ( )2 𝑜𝑟 𝑥 = − ± √− + ( )2
2𝑎 𝑎𝑐 2𝑎 2𝑎 𝑎𝑐 2𝑎

Definition of Equal Polynomials


If 𝑎𝑥 2 + 𝑏𝑥 + 𝑐 = 𝑝𝑥 2 + 𝑞𝑥 + 𝑟 for all values of , then 𝑎 = 𝑝 , 𝑏 = 𝑞, and 𝑐 = 𝑟

Solving Quadratic Equations by Using the Quadratic Formula

−𝑏 ± √𝑏 2 − 4𝑎𝑐
𝑥=
2𝑎
The Discriminant

B E R A N I B E R J U A N G | 17
Solving Systems Consisting Linear and Quadratic Equations

18 | I U P I T B
E. LINEAR INEQUALITIES AND GRAPH

Verbal Expressions Inequality Graph

All real number less than 4 𝑥<4

All real number greater than −𝟑 𝑥 > −3

All real number less than or equal to 2


All real number at most 2 𝑥≤2
All real number no greater than 2
All real number greater than or equal to -
1
𝑥≥1
All real number at least -1
All real number no less than -1

Compound and Absolute Value Inequalities


Two or more inequalities that are connected by the words and or are called a compound inequality.
A compound inequality containing or is true if one or more of the inequalities is true. Its graph is
the union of the graphs of the two inequalities.

Since the absolute value of any number 𝑥 is its distance from zero on a number line, |𝑥| < 1 means
that the distance from zero to 𝑥 is less than 1 unit and |𝑥| > 1 means that the distance from zero
to 𝑥 is greater than 1 unit. Therefore the inequality |𝑥| < 1 is equivalent to −1 < 𝑥 < 1 and |𝑥| >
1 is equivalent to 𝑥 < −1 or > 1 .

To translate more general absolute value inequalities into compound inequalities, use the following
properties.
1. The inequality |𝑎𝑥 + 𝑏| < 𝑐 is equivalent to −𝑐 < 𝑎𝑥 + 𝑏 < 𝑐, in which 0 ≤ 𝑐 .
2. The inequality |𝑎𝑥 + 𝑏| > 𝑐 is equivalent to 𝑎𝑥 + 𝑏 < −𝑐 𝑜𝑟 𝑎𝑥 + 𝑏 > 𝑐, in which 0 ≤ 𝑐 .

Graphing Inequalities in Two Variables


A linear inequality in 𝑥 𝑎𝑛𝑑 𝑦 is an inequality that can be written in one of the following forms.

𝑎𝑥 + 𝑏𝑦 < 𝑐, 𝑎𝑥 + 𝑏𝑦 ≤ 𝑐, 𝑎𝑥 + 𝑏𝑦 > 𝑐, 𝑎𝑥 + 𝑏𝑦 ≥ 𝑐

1. Sketch the graph of the corresponding linear equation. Use a dashed line for inequalities with
< 𝑜𝑟 > and a solid line for inequalities with ≤ 𝑜𝑟 ≥. This line divides the coordinate plane
into two half planes.
2. Test a point in one of the half planes to find whether it is a solution of the inequality.
3. If the test point is a solution, shade its half plane. If not, shade the other half plane.

B E R A N I B E R J U A N G | 19
F. STATISTIC AND GRAPHICAL DISPLAY

Mean
The mean of a data set is the sum of the values in the data set divided by the number of values in
the data set.
𝑠𝑢𝑚 𝑜𝑓 𝑡ℎ𝑒 𝑣𝑎𝑙𝑢𝑒𝑠 𝑥1 + 𝑥2 +. . . . +𝑥𝑛 ∑ 𝑥𝑖
𝑴𝒆𝒂𝒏 = = =
𝑛𝑢𝑚𝑏𝑒𝑟 𝑜𝑓 𝑣𝑎𝑙𝑢𝑒𝑠 𝑛 𝑛

Median
The median of a set of data is the middle number when the data are arranged in order. If there are
two middle numbers, the median is the mean of the two numbers.

7+10
set of data {3,7,10,14}, the median is , 𝑜𝑟 8.5
2

Mode
The mode of a set of data is the number that appears most frequently.

Set of data {3,7,7,10,14,19,19, 25}, the modes are 7 and 19, both numbers appear twice.

Range
The range in a set of data is the difference between the greatest value and the least value of the
data

Graphical Displays

Frequency of a Particular Category The number of times the category appears in the data.
A table or graph that presents the categories along with their
Frequency Distribution
associated frequencies
Relative Frequency of a Category The associated frequency divided by the total number of data.

Line Graph Bar Graph Histogram


A histogram is useful in
A line graph shows the In a bar graph, rectangular
displaying frequency
change in a certain data over bars represent the categories
distributions that are similar
a period of time. of data.
to bar graphs

20 | I U P I T B
Scatter Plots and the Regression Lines
A scatter plot is a mathematical diagram represented by a set of dots that display the relationship
between two numerical variables.

The 𝑥 − 𝑐𝑜𝑜𝑟𝑑𝑖𝑛𝑎𝑡𝑒, measured along the horizontal axis, gives the value of one variable
The 𝑦 − 𝑐𝑜𝑜𝑟𝑑𝑖𝑛𝑎𝑡𝑒, measured along the vertical axis, gives the value of the other
variable.

When the set of dots are plotted, usually there is no single line that passes through all the data
points,
but we can approximate a linear relationship by finding a line that best fits the data. This line is
called
the regression line or the best fit line.

If the slope of a regression line is positive, then the two variables have a positive association.
If the slope of a regression line is negative, then the two variables have a negative association.
If the slope of a regression line cannot be determined, then the two variables have no association.

Quadratic And Exponential Models

B E R A N I B E R J U A N G | 21
G. GEOMETRY
Angles
Angles are classified according to their measures.

Acute angle
𝑬𝒙. ∠𝑷𝑶𝑸 𝒂𝒏𝒅 ∠𝑸𝑶𝑹
measures between 𝟎 𝒂𝒏𝒅 𝟗𝟎.
Right angle
𝐸𝑥. ∠𝑃𝑂𝑅 𝑎𝑛𝑑 ∠𝑆𝑂𝑅
measures 𝟗𝟎
Obtuse angle
measures between 𝐸𝑥. ∠𝑄𝑂𝑆
𝟗𝟎 𝒂𝒏𝒅 𝟏𝟖𝟎
Straight angle
𝐸𝑥. ∠𝑃𝑂𝑆
measures 𝟏𝟖𝟎

Parallel and Perpendicular Lines


For two parallel lines 𝑙 𝑎𝑛𝑑 𝑚 which are cut by the transversal :
1. Corresponding Angles are equal in measure.
𝑚∠1 = 𝑚∠5 𝑚∠2 = 𝑚∠6
𝑚∠3 = 𝑚∠7 𝑚∠4 = 𝑚∠8

2. Alternate Interior Angles are equal in measure.


𝑚∠3 = 𝑚∠5 𝑚∠4 = 𝑚∠6

3. Alternate Exterior Angles are equal in measure.


𝑚∠1 = 𝑚∠7 𝑚∠2 = 𝑚∠8

4. Consecutive(Same Side) Interior Angles are supplementary.


𝑚∠1 + 𝑚∠6 = 180° 𝑚∠4 + 𝑚∠5 = 180°

Theorem
In a plane, if a line is perpendicular to one of two parallel lines, it
is also perpendicular to the other.

If 𝑡 ⊥ 𝑙 and || 𝑚 , then 𝑡 ⊥ 𝑚 .

Angles of a Triangle

Angle Sum Theorem


The angle sum of a triangle is 𝟏𝟖𝟎°.

𝒎∠𝑨 + 𝒎∠𝑩 + 𝒎∠𝑪 = 𝟏𝟖𝟎°

Exterior Angle Theorem


The measure of an exterior angle of a triangle
is equal to the sum of the measures of the
two remote interior angles.

𝒎∠𝑩𝑪𝑫 = 𝒎∠𝑨 + 𝒎∠𝑩

22 | I U P I T B
Isosceles Triangle Theorem
If two sides of a triangle are congruent, the
angles opposite of those sides are congruent.

If = 𝑩𝑪 , then 𝒎∠𝑪 + 𝒎∠𝑨.

Isosceles Triangle Theorem - Corollary


If a line bisects the vertex angle of an isosceles
triangle, the line is the perpendicular bisector
of the base.

If 𝑨𝑩 = 𝑩𝑪 and 𝒎∠𝑨𝑩𝑫 = 𝒎∠𝑪𝑩𝑫, then


𝑩𝑫 ⊥ 𝑨𝑪 and 𝑨𝑫 = 𝑪𝑫.

Similar Triangles and Proportional Parts


If ∠𝐴 = ∠𝐷 and ∠𝐵 = ∠𝐸, then ∠𝐶 = ∠𝐹,

Therefore ◺ 𝐴𝐵𝐶 −◺ 𝐷𝐸𝐹, and


𝐴𝐵 𝐵𝐶 𝐴𝐶 𝑝𝑎𝑟𝑒𝑖𝑚𝑒𝑡𝑒𝑟 𝑜𝑓 ◺ 𝐴𝐵𝐶
= = =
𝐷𝐸 𝐸𝐹 𝐷𝐹 𝑝𝑒𝑟𝑖𝑚𝑒𝑡𝑒𝑟 𝑜𝑓 ◺ 𝐷𝐸𝐹

Triangle Proportionality Theorem

If a line parallel to one side of a triangle intersects the other two


sides, it divides those sides proportionally.
In ∆𝐴𝐵𝐶 , if 𝐴𝐶||𝐷𝐸 then ∆𝐴𝐵𝐶~∆𝐷𝐵𝐸 by 𝐴𝐴 Similarity.

1
If 𝐷 and 𝐸 are the midpoints of 𝐴𝐵 and , 𝐴𝐶||𝐷𝐸 and 𝐷𝐸 = 𝐴𝐶
2

𝑨𝑩 𝑪𝑩 𝑨𝑪 𝑩𝑫 𝑩𝑬 𝑩𝑨 𝑩𝑪 𝑩𝑫 𝑩𝑨 𝑩𝑬 𝑩𝑪
= = = , = , = , =
𝑫𝑩 𝑬𝑪 𝑫𝑬 𝑫𝑨 𝑬𝑪 𝑫𝑨 𝑬𝑪 𝑫𝑬 𝑨𝑪 𝑫𝑬 𝑨𝑪

Area of a Triangle
The area 𝐴 of a triangle equals half the product of a base and the height to that base.

1
𝐴= 𝑏∙ℎ
2

Area of equilateral triangle with side length of 𝑎

1 √3 √3 2
𝐴 = (𝑎) ( 𝑎) = 𝑎
2 2 4

B E R A N I B E R J U A N G | 23
The area of ∆𝐴𝐵𝐶 at the right can be written in 3 different ways:

1 1 1
area of ∆𝐴𝐵𝐶 = 𝐵𝐶 ∙ 𝐴𝐷 = 𝐴𝐶 ∙ 𝐵𝐸 = 𝐴𝐵 ∙ 𝐶𝐹
2 2 2

The perimeter 𝑃 of a triangle is the sum of the lengths of all three sides.

𝑃 = 𝐴𝐵 + 𝐵𝐶 + 𝐶𝐴

Ratios of Areas of Two Triangles


1. If two triangles are similar with corresponding sides in a ratio of 𝑎 ∶ 𝑏: , then the ratio of their
areas equals 𝑎2 ∶ 𝑏 2
2. If two triangles have equal heights, then the ratio of their areas equals the ratio of their bases.
3. If two triangles have equal bases, then the ratio of their areas equals the ratio of their heights

Parallelograms
A parallelogram is a quadrilateral with two pairs of parallel
opposite sides.
𝐴𝐵|| 𝐶𝐷
𝐵𝐶||𝐴𝐷

Properties of Parallelogram
Opposite sides are congruent 𝐴𝐵 ≅ 𝐶𝐷 and 𝐵𝐶 ≅ 𝐴𝐷
∠𝐵𝐴𝐷 ≅ ∠𝐵𝐶𝐷
Opposite angles are congruent
∠𝐴𝐵𝐶 ≅ ∠𝐴𝐷𝐶
Consecutive angles are 𝑚∠𝐴𝐵𝐶 + 𝑚∠𝐵𝐴𝐷 = 180
supplementary 𝑚∠𝐴𝐷𝐶 + 𝑚∠𝐵𝐶𝐷 = 180
The diagonals bisect each other 𝐴𝐸 = 𝐶𝐸 𝑎𝑛𝑑 𝐵𝐸 = 𝐷𝐸

A rhombus is a parallelogram with four sides of equal measure.

𝐴𝐵 = 𝐵𝐶 = 𝐶𝐷 = 𝐷𝐴
𝐴𝐶 ⊥ 𝐵𝐷
𝑚∠1 = 𝑚∠2 = 𝑚∠5 = 𝑚∠6
𝑚∠3 = 𝑚∠4 = 𝑚∠7 = 𝑚∠8

Theorem

The area of a parallelogram equals the product of a base and


𝐴=𝑏∙ℎ
the height to that base
The area of a rhombus is half the product of the lengths of its 1
𝐴= 𝑑 ∙𝑑
diagonals (𝒅𝟏 𝒂𝒏𝒅 𝒅𝟐 ) 2 1 2

.
24 | I U P I T B
Regular Polygons
Important Terms Definitions
A convex polygon with all sides congruent and all angles
A regular polygon
congruent.
Inscribed in a circle and the circle is circumscribed
A polygon about the polygon where each vertex of the polygon
lies on the circle.
The radius of a regular polygon The distance from the center to a vertex of the polygon.
An angle formed by two radii drawn to consecutive
A central angle of a regular polygon
vertices.
The apothem of a regular polygon The distance from the center to a side.

Center: 𝑂
Radius: 𝑂𝑃, 𝑂𝑄, 𝑂𝑅, ….
Center: ∠𝑃𝑂𝑄, ∠𝑄𝑂𝑅, ….
Center: ∠𝑃𝑄𝑅, ∠𝑄𝑅𝑆, ….
Center: 𝑂𝐴, 𝑂𝐵, … . (𝐷𝑒𝑛𝑜𝑡𝑒𝑑 𝑤𝑖𝑡ℎ 𝑙𝑒𝑡𝑡𝑒𝑟 𝑎)

Theorems - Angles and Areas of Regular Polygons


The sum of the measures of the interior angles of an 𝒏-sided polygon (𝑛 − 2)180
(𝑛 − 2)180
The measure of each interior angle of a regular 𝒏-sided polygon
𝑛
The sum of the measures of the exterior angles of any polygon 360
The area of a regular polygon is half the product of the apothem 𝒂 , and the 1
𝐴 = 𝑎𝑝
perimeter 𝒑 2

Regular Polygons Inscribed in Circles

B E R A N I B E R J U A N G | 25
Arcs, Angles, and Tangents
Important Terms Definition

Chord a segment whose endpoints lie on a circle.


Secant a line that contains a chord.
a line in the plane of a circle, and intersects the circle at exactly
Tangent
one point: the point of tangency.
Central angle an angle whose vertex is the center of the circle

The measure of a minor arc 𝑙𝑒𝑠𝑠 𝑡ℎ𝑎𝑛 180°


The measure of a semicircle 180°
The measure of a major arc 360° − 𝑡ℎ𝑒 𝑚𝑒𝑎𝑠𝑢𝑟𝑒 𝑜𝑓 𝑖𝑡𝑠 𝑚𝑖𝑛𝑜𝑟 𝑎𝑟𝑐

Arc Addition Postulate


The measure of an arc formed by two adjacent arcs is the sum of ̂ = 𝑚𝑃𝑄
𝑚𝑃𝑄𝑅 ̂ + 𝑚𝑄𝑅
̂
the measure of the two arcs

Theorems - Tangent Lines


If a line is tangent to a circle, then the line is perpendicular to the
radius at the point of tangency.

𝑃𝐴 ⊥ 𝑂𝐴 𝑎𝑛𝑑 𝑃𝐵 ⊥ 𝑂𝐵

Tangents to a circle from the same exterior point are congruent.


𝑃𝐴 = 𝑃𝐵

Arc Lengths and Areas of Sectors


̂
Length of 𝑨𝑩
Circumference of a circle:
𝑚∠𝐴𝑂𝐵
𝐶 = 2𝜋𝑟 𝑜𝑟 𝐶 = 𝜋𝑑 ̂ = 2𝜋𝑟 ∙
𝐴𝐵
360

Area of sector
Area of circle:
𝑚∠𝐴𝑂𝐵
𝐴 = 𝜋𝑟 2 𝐴𝑂𝐵 = 𝜋𝑟 2 ∙
360

26 | I U P I T B
Inscribed Angles
The measure of an inscribed angle is half the measure of its intercepted
arc and half the measure of its central angle.

1 1
̂ = 𝑚∠𝐴𝑂𝐶
𝑚∠𝐵 = 𝑚∠𝐴𝐶
2 2

Corollaries to the Inscribed Angle Theorem


Corollary 1 Corollary 2 Corollary 3
Two inscribed angles that If a quadrilateral is inscribed
An angle inscribed in a
intercept the same arc are in a circle, its opposite angles
semicircle is a right angle
congruent are supplementary

Arcs and Chords

Theorem 1 Theorem 2 Theorem 3


In the same circle or in
In the same circle or in
If a diameter is to a chord, it congruent circles, chords
congruent circles, congruent
bisects the chord and its arc. equidistant to the center(s)
arcs have congruent chords
are congruent

Circles in the Coordinate Plane

Equation of a Circle

(𝑥 − ℎ)2 + (𝑦 − 𝑘)2 = 𝑟 2

B E R A N I B E R J U A N G | 27
H. TRIGONOMETRY

Trigonometric Ratios of Acute Angles


A ratio of the lengths of sides of a right triangle is called a trigonometric ratio.

Trigonometric
𝒔𝒊𝒏𝒆 𝒄𝒐𝒔𝒊𝒏𝒆 𝒕𝒂𝒏𝒈𝒆𝒏𝒕 𝒄𝒐𝒔𝒆𝒄𝒂𝒏𝒕 𝒔𝒆𝒄𝒂𝒏𝒕 𝒄𝒐𝒕𝒂𝒏𝒈𝒆𝒏𝒕
Ratios
Abbreviations 𝑠𝑖𝑛 𝑐𝑜𝑠 𝑡𝑎𝑛 𝑐𝑠𝑐 𝑠𝑒𝑐 𝑐𝑜𝑡

Complementary Angle Theorem Trigonometric Identities

sin 𝜃 = cos(90° − 𝜃) sin 𝜃


tan 𝜃 =
cos 𝜃

cos 𝜃 = sin(90° − 𝜃)
𝑠𝑖𝑛2 𝜃 + 𝑐𝑜𝑠 2 𝜃 = 1
If sin ∠𝐴 = cos ∠𝐵, then 𝑚∠𝐴 + 𝑚∠𝐵 = 90° .

The Radian Measure of an Angle


One radian is the measure of a central angle θ whose intercepted arc has a length equal to the
circle’s radius.
If 𝐴𝐵 = 𝑂𝐴, then 𝑚∠𝐴𝑂𝐵 = 1 𝑟𝑎𝑑𝑖𝑎𝑛.

2𝜋 𝑟𝑎𝑑𝑖𝑎𝑛𝑠 = 360°, or 𝜋 𝑟𝑎𝑑𝑖𝑎𝑛𝑠 = 180°

180° 𝜋
1 𝑟𝑎𝑑𝑖𝑎𝑛 = ≈ 57.3° and 1° = 𝑟𝑎𝑑𝑖𝑎𝑛𝑠
𝜋 180

28 | I U P I T B
Periodic Properties of the Trigonometric Functions
𝒔𝒊𝒏 (𝜽 ± 𝟑𝟔𝟎°) = 𝒔𝒊𝒏 𝜽 𝑐𝑜𝑠 (𝜃 ± 360°) = 𝑐𝑜𝑠 𝜃 𝑡𝑎𝑛 (𝜃 ± 180°) = 𝑡𝑎𝑛 𝜃

Trigonometric Functions and the Unit Circle


Suppose 𝑃(𝑥, 𝑦) is a point on the circle 𝑥 2 + 𝑦 2 = 𝑟 2 and 𝜃 is an angle in
standard position with terminal side 𝑂𝑃, as shown at the right.

𝑦 𝑥
sin 𝜃 = cos 𝜃 =
𝑟 𝑟

The circle 𝑥 2 + 𝑦 2 = 1 is called the unit circle. This circle is


the easiest one to work with because 𝑠𝑖𝑛𝜃 𝑎𝑛𝑑 𝑐𝑜𝑠𝜃 are
simply the 𝑦 − 𝑐𝑜𝑜𝑟𝑑𝑖𝑛𝑎𝑡𝑒𝑠 and the 𝑥 − 𝑐𝑜𝑜𝑟𝑑𝑖𝑛𝑎𝑡𝑒𝑠 of
the points where the terminal side of θ intersects the circle.

𝑦 𝑦 𝑥 𝑥
sin 𝜃 = = cos 𝜃 = = =𝑥
𝑟 1 𝑟 1
=𝑦

Familiar Angle

Reference Angle

B E R A N I B E R J U A N G | 29
Angles with a reference angle of 𝟑𝟎°, 𝟏𝟓𝟎°, 𝟐𝟏𝟎°. 𝟑𝟑𝟎°

Angles with a reference angle of 𝟔𝟎°, 𝟏𝟐𝟎°, 𝟐𝟒𝟎°. 𝟑𝟎𝟎°

Angles with a reference angle of 𝟒𝟓°, 𝟏𝟑𝟓°, 𝟐𝟐𝟓°. 𝟑𝟏𝟓°

30 | I U P I T B
I. EXPONENT

Laws of Exponents and Scientific Notation


Laws of Exponents
For all integers 𝑚 and 𝑛 and any nonzero numbers 𝑎 and , the following properties hold
Symbols Example

𝒂𝒎 ∙ 𝒂𝒏 = 𝒂𝒎+𝒏 23 ∙ 25 = 𝑎3+5 = 28
(𝒂𝒎 )𝒏 = 𝒂𝒎∙𝒏 (𝑎3 )5 = 𝑎3∙5 = 𝑎15
(𝒂𝒃)𝒎 = 𝒂𝒎 𝒃𝒎 (−2𝑥)5 = (−2)5 𝑥 5 = −32𝑥 5
𝒂𝒎 25
= 𝒂𝒎−𝒏 = 25−3 = 22
𝒂𝒏 23
𝒂 𝒎 𝒂𝒎 2𝑥 3 (2𝑥)3 8𝑥 3
( ) = 𝒎 ( ) = =
𝒃 𝒃 5 53 125
0
𝒂𝟎 = 𝟏 (−2𝑥𝑦) = 1
𝟏 𝟏 1 1
𝒂−𝒏 = 𝒏 𝒂𝒏𝒅 −𝒏 = 𝒂𝒏 5−2 = 2 𝑎𝑛𝑑 −3 = 43
𝒂 𝒂 5 4
𝒂 −𝒎 𝒃 𝒎 𝒃𝒎 −2𝑥 −2 3𝑦 3
2
(3𝑦 3 )2 9𝑦 6
( ) =( ) = 𝒎 ( 3) = ( ) = =
𝒃 𝒂 𝒂 3𝑦 −2𝑥 (−2𝑥)2 4𝑥 2

FOIL Method
The product of two binomials is the sum of the products of the first terms, the outer terms, the
inner terms,

Product of Product of Product of Product of Last


First Terms Outer Terms Inner Terms Terms
↓ ↓ ↓ ↓
(2𝑥 + 3)(𝑥 − 5) = (2𝑥)(𝑥) + (2𝑥)(−5) + (3)(𝑥) + (3)(−5)
= 2𝑥 2 − 10𝑥 + 3𝑥 − 15
= 2𝑥 2 − 7𝑥 − 15

Special Products
Square of a Sum (𝑎 + 𝑏)2 = (𝑎 + 𝑏)(𝑎 + 𝑏) = 𝑎2 + 2𝑎𝑏 + 𝑏 2
Square of a Difference (𝑎 − 𝑏)2 = (𝑎 − 𝑏)(𝑎 − 𝑏) = 𝑎2 − 2𝑎𝑏 + 𝑏 2
Square of a Sum and Difference (𝑎 + 𝑏)(𝑎 − 𝑏) = 𝑎2 − 𝑏 2

Exponential Functions and Graphs


An exponential function is a function of the form𝑓(𝑥) = 𝑎𝑏 𝑥 , in which 𝑎 ≠ 0, 𝑏 > 0, and 𝑏 ≠ 1

B E R A N I B E R J U A N G | 31
Exponential Growth and Decay
Compound Interest Formulas
If initial amount 𝑃 is invested at annual interest rate, the investment will grow to final amount 𝐴 in
𝑡 years
𝐴 = 𝑃(1 + 𝑟)𝑡

We can use the same formula for the population or value of goods that is increasing or decreasing.

Exponential Growth and Doubling-Time Growth Formula


If a population is increasing at a constant rate 𝑟 each year, the population at the end of 𝑡 years
would be
𝐴 = 𝑃(1 + 𝑟)𝑡

If an initial population of size 𝑃 doubles every 𝑑 years (or any other unit of time), the final
number 𝐴 in 𝑡 years is given by
𝐴 = 𝑃(2)𝑡/𝑑

Exponential Decay and Half-Life Decay Formula


If a population is decreasing at a constant rate 𝑟 each year, the population at the end of 𝑡 years
would be
𝐴 = 𝑃(1 − 𝑟)𝑡

The half-life of a substance is the amount of time it takes for half of the substance to decay. If an
initial population of size 𝑃 has a half-life of 𝑑 years (or any other unit of time), the final number 𝐴
in 𝑡 years is given by
1
𝐴 = 𝑃( )𝑡/𝑑
2

32 | I U P I T B
J. RATES AND RATIO

Ratios
A ratio is a comparison of two quantities by division.
The ratio of a to b can be written in three different ways

𝑎
𝑎 𝑡𝑜 𝑏 𝑎∶𝑏
𝑏

If two quantities are in the ratio of 𝑎 to , then the two numbers can be expressed as 𝑎𝑥 𝑎𝑛𝑑 𝑏𝑥,
in which 𝑥 is a positive integer.

Rates
A rate is a ratio of two measurements having different units of measure.
For example, a price of $2.59 per gallon of gasoline, an income of $750 in 3 days, and a speed of 60
miles per hour are all rates.
A unit rate is a rate that has a denominator of 1.

Examples of Unit Rates

𝑃𝑟𝑖𝑐𝑒 𝑜𝑓 𝑃𝑎𝑐𝑘𝑎𝑔𝑒
𝑢𝑛𝑖𝑡 𝑝𝑟𝑖𝑐𝑒 =
𝑁𝑢𝑚𝑏𝑒𝑟 𝑜𝑓 𝑈𝑛𝑖𝑡𝑠 𝑖𝑛 𝑡ℎ𝑒 𝑃𝑎𝑐𝑘𝑎𝑔𝑒

𝑁𝑢𝑚𝑏𝑒𝑟 𝑜𝑓 𝑀𝑖𝑙𝑒𝑠 𝑇𝑟𝑎𝑣𝑒𝑙𝑒𝑑


𝐺𝑎𝑠 𝑀𝑖𝑙𝑒𝑎𝑔𝑒 =
𝑁𝑢𝑚𝑏𝑒𝑟 𝑜𝑓 𝐺𝑎𝑙𝑙𝑜𝑛𝑠 𝑜𝑓 𝑔𝑎𝑠 𝑈𝑠𝑒𝑑

𝑁𝑢𝑚𝑏𝑒𝑟 𝑜𝑓 𝑀𝑖𝑙𝑒𝑠 𝑇𝑟𝑎𝑣𝑒𝑙𝑒𝑑


𝑆𝑝𝑒𝑒𝑑 (𝑀𝑖𝑙𝑒𝑠 𝑝𝑒𝑟 𝐻𝑜𝑢𝑟) =
𝑁𝑢𝑚𝑏𝑒𝑟 𝑜𝑓 𝐻𝑜𝑢𝑟𝑠

𝑀𝑎𝑠𝑠
𝐷𝑒𝑛𝑠𝑖𝑡𝑦 =
𝑉𝑜𝑙𝑢𝑚𝑒

B E R A N I B E R J U A N G | 33
K. PERCENTAGE

Percent of Change
The word percent means hundredth or out of every hundred.
To write a decimal or a fraction as a percent, multiply the decimal or the fraction by 100 and add
the % sign. Convert the fraction to decimal.
1
To write a percent as a decimal or a fraction, multiply the percent by , and drop the % sign.
100

Percent of Change.

𝑎𝑚𝑜𝑢𝑛𝑡 𝑜𝑓 𝑖𝑛𝑐𝑟𝑒𝑎𝑠𝑒
𝑝𝑒𝑟𝑐𝑒𝑛𝑡 𝑖𝑛𝑐𝑟𝑒𝑎𝑠𝑒 =
𝑜𝑟𝑖𝑔𝑖𝑛𝑎𝑙 𝑎𝑚𝑜𝑢𝑛𝑡

𝑎𝑚𝑜𝑢𝑛𝑡 𝑜𝑓 𝑑𝑒𝑐𝑟𝑒𝑎𝑠𝑒
𝑝𝑒𝑟𝑐𝑒𝑛𝑡 𝑑𝑒𝑐𝑟𝑒𝑎𝑠𝑒 =
𝑜𝑟𝑖𝑔𝑖𝑛𝑎𝑙 𝑎𝑚𝑜𝑢𝑛𝑡

Percents and Equations


You can solve a percent problem by writing and solving an equation or a proportion. Three types
of percent equations and corresponding verbal phrases are illustrated below

Verbal Phrase Algebraic Expression


𝑛 = 0.15 × 72
1. Finding the part What is 15% of 72? 15 𝑛
=
100 72
𝑛
× 20 = 6
2. Finding the percent What is percent of 20 in 6? 100
𝑛 6
=
100 20
17 = 0.25 × 𝑛
3. Finding the Whole 17 is 25% of what number? 25 17
=
100 𝑛

34 | I U P I T B
L. PROBABILITY

Rules of Probability
An outcome is one of the possible results that can occur as a result of a trial.
The probability of event , denoted 𝑃(𝐴), is:

𝑡ℎ𝑒 𝑛𝑢𝑚𝑏𝑒𝑟 𝑜𝑓 𝑡𝑖𝑚𝑒𝑠 𝑡ℎ𝑒 𝑑𝑒𝑠𝑖𝑟𝑒𝑑 𝑜𝑢𝑡𝑐𝑜𝑚𝑒 𝑜𝑐𝑐𝑢𝑟𝑠


𝑃(𝐴) =
𝑡ℎ𝑒 𝑡𝑜𝑡𝑎𝑙 𝑛𝑢𝑚𝑏𝑒𝑟 𝑜𝑓 𝑜𝑢𝑡𝑐𝑜𝑚𝑒𝑠

If the outcome of one event does not influence the outcome of the second event, then the events
are said to be independent. When drawing at random with replacement, the draws are
independent. Without replacement, the draws are dependent.

If two events 𝐴 and 𝐵 are independent, then the probability that both events will occur is
𝑃(𝐴 𝑎𝑛𝑑 𝐵) = 𝑃(𝐴) ∙ 𝑃(𝐵)

If two events 𝐴 and 𝐵 are dependent, then the probability that both events will occur is
𝑃(𝐴 𝑎𝑛𝑑 𝐵) = 𝑃(𝐴) ∙ 𝑃(𝐵 𝑓𝑜𝑙𝑙𝑜𝑤𝑖𝑛𝑔 𝐴)

If two events 𝐴 and 𝐵 cannot occur at the same time, then the probability that either 𝐴 𝑜𝑟 𝐵 occurs
is
𝑃(𝐴 𝑜𝑟 𝐵) = 𝑃(𝐴) + 𝑃(𝐵)

Categorical Data and Conditional Probabilities


The probability of event 𝐴, given that event 𝐵 occurred, is called the conditional probability of
𝐴 given 𝐵 and is denoted by 𝑃(𝐴|𝐵). [Note: The vertical bar between A and B is read “given.”]

𝑃(𝐴 𝑎𝑛𝑑 𝐵) 𝑝𝑢𝑝𝑜𝑙𝑎𝑡𝑖𝑜𝑛 𝑜𝑓 𝐴 𝑖𝑛 𝐵


𝑃(𝐴|𝐵) = =
𝑃(𝐵) 𝑃𝑜𝑝𝑢𝑎𝑡𝑖𝑜𝑛 𝑜𝑓 𝐵

Two-way contingency table is a type of table in a matrix format that displays relationships between
two or more categorical variables.

B E R A N I B E R J U A N G | 35
M. SURFACE AREAS AND VOLUMES

Prisms

Theorems - Prisms

Lateral Area 𝐿. 𝐴 = 𝑃 ∙ ℎ
Total Surface Area 𝑇. 𝐴 = 𝐿. 𝐴 + 2𝐵
Volume of a Prism 𝑉 =𝐵∙ℎ

Theorems - Cubes
Volume of a cube 𝑠3
Surface area of a cube 6𝑥 2

Cylinders and Spheres

Theorems - Cylinders
Lateral Area of a Cylinder 𝐿. 𝐴 = 2𝜋𝑟ℎ
Total Surface Area of a Cylinder 𝑇. 𝐴 = 2𝜋𝑟ℎ + 2𝜋𝑟 2
Volume of a Cylinder 𝑉 = 𝜋𝑟 2 ℎ

Theorems - Spheres
Surface Area of a Sphere: 𝑆. 𝐴 = 4𝜋𝑟 2
4
Volume of a Sphere 𝑉 = 𝜋𝑟 3
3

36 | I U P I T B
Pyramids and Cones

Theorems – Pyramids and Cones


1
Lateral Area of a Pyramid 𝐿. 𝐴. = 𝑃𝑙
2
Lateral Area of a Cone 𝑆. 𝐴. = 𝐿. 𝐴. +𝐵
1
Surface Area of a Pyramid 𝑉= 𝐵ℎ
3
Surface Area of a Cone 𝐿. 𝐴. = 𝜋𝑟𝑙
Volume of a Pyramid 𝑆. 𝐴. = 𝐿. 𝐴. +𝐵
1 2
Volume of a Cone 𝑉= 𝜋𝑟 ℎ
3

SOLUTION STRATEGIES
#1 STRATEGIES
Apply a general rule or formula to answer the question.
#2 STRATEGIES
Apply basic properties of numbers.
#3 STRATEGIES
Eliminate as many answers as possible so that you can select from a smaller set of answer
choices.
#4 STRATEGIES
Substitute answers into the given question to see which one produces the correct result.
#5 STRATEGIES
Break down the situation into individual steps.

B E R A N I B E R J U A N G | 37
QUANTITATIVE REASONING - SUBJECT

A. QUANTITATIVE COMPARASION
Quantitative Comparisons may either be pure mathematical comparisons, or they may be
posed in real-life settings, or written up as "word problems" that the student must be able to
model or interpret mathematically. They assess a combination of basic math skill,
understanding of elementary math concepts, and the abilities to reason quantitatively, solve
problems with quantitative methods, and model information mathematically.
Quantitative Comparison questions have four possible answers:
A. Quantity A is greater,
B. Quantity B is greater,
C. The two quantities are equal, or
D. Their relationship cannot be determined.

B. DATA-SUFFICIENCY
Data-sufficiency questions have as their focus not finding the solution to the problem, but
determining whether or not there is enough information to solve it. In each item, you are given
a situation and then two statements, and you are asked to determine whether one of the
statements, both of the statements, or neither of the statements provides enough information
to solve the problem.
The answer choices are the same for each question of this type:
A. Statement (1) ALONE is sufficient, but statement (2) is not sufficient.
B. Statement (2) ALONE is sufficient, but statement (1) is not sufficient.
C. BOTH statements TOGETHER are sufficient, but NEITHER statement ALONE is sufficient.
D. EACH statement ALONE is sufficient.
E. Statements (1) and (2) TOGETHER are NOT sufficient.

C. ESSAY
Essay questions are prepared with the same material as quantitative in general with no
answer choices. Participants are expected to be able to answer questions correctly without
any answer choices
SOLUTION STRATEGIES
#1 STRATEGIES
Work only as far as you must to be sure the question has an answer.
#2 STRATEGIES
Be sure to try each statement separately to get an answer to the question.
#3 STRATEGIES
If statement 1 does not yield an answer to the question, check statement 2 to see if it
will yield an answer to the question.
#4 STRATEGIES
If neither statement alone yields an answer to the question, be sure to consider them
together.
#5 STRATEGIES
Remember that when both statements fail to yield an answer individually, they can still
fail to yield an answer when taken together.

38 | I U P I T B
SET-1

Questions 1-10 are based on the following passage.

This passage is adapted from George Eliot, Silas Marner. Originally published In 1861. Silas was a
weaver and a notorious miser, but then the gold he had hoarded was stolen. Shortly after, Silas
adopted a young child, Eppie, the daughter of an impoverished woman who had died suddenly.

Line Unlike the gold which needed nothing, and must be worshipped in close-locked
solitude—which was hidden away from the daylight, was deaf to the song tine of birds,
and Started to no human tones—Eppie was a creature of endless claims and ever-growing
desires, seeking and loving sunshine, and living sounds, and living movement; making trial
5 of everything, with Mist in new joy, and stirring the human kindness in all eyes that looked
on her. The gold had kept his to thoughts in an ever-repeated circle, leading to nothing
beyond itself; but Eppie was an object compacted of changes and hopes that forced his
thoughts onward. and carried them far away from their old eager pacing towards the
same blank is limit—carried them away to the new things that would come with the
10 coming years, when Eppie would have learned to understand how her father Silas cared
for her; and mode him look for images of that time in the ties and charities Jut hound
together the families of his neighbor. The gold had asked that he should sit weaving longer
and longer, deafened and blinded more and more to all things except the monotony of
his loom and the repetition of his web: but Eppie called hint away from his weaving, and
15 made hint think all its pauses a holiday, reawakening his senses with her fresh life, even
to the old winter-flies that came crawling forth in the early spring sunshine, and warming
him into joy because she had joy.
And when the sunshine grew strong and lasting, so that the buttercups were thick in
the meadows, Silas might be seen in the sunny mid-day, or in the late afternoon when the
20 shadows Were lengthening under the hedgerows, strolling out with uncovered 35 head
to carry Eppie beyond the Stone-pits to where the flowers grew. fill they reached some
favorite bank where he could sit down while Eppie toddled to pluck the flowers, and make
remarks to the winged things that murmured happily above the bright as petals, calling
“Dad-dad's” attention continually by bringing him the flowers. Then she would turn her
25 ear to some sudden bird-note, and Silas Inured to please her by making signs of hushed
stillness, that they might listen for the note to come spin: so that as when it c.ae, she
setup her small back and laughed with gurgling triumph, Sitting on the banks in this way.
B E R A N I B E R J U A N G | 39
Silas began to look for the once familiar herbs again: and as the leaves, with their
unchanged outline and markings, lay on his palm, there was a sense of so crowding
30 remembrances from which he turned away timidly, taking refuge in Eppie's little world,
that lay lightly on his enfeebled spirit.
As the child's mind was growing imp knowledge, his mind was growing into memory:
as her his as unfolded, his soul, long stupefied in a cold narrow prison, was unfolding too.
and trembling gradually into full consciousness.
35 It was an influence which must gather force with every new year: the tones that
stirred Silas heap so grew articulate, and called for more distinct answer; shapes and
sounds grew clearer for Eppie’s eyes and ears, and there was more that-“Dad-dad” was
imperatively required to notice and account for, Also, by the time Eppie was three years
old, she as developed a fine capacity for mischief, and for devising ingenious ways of being
40 troublesome, winch found much exercise, not only for Silas' patience, but for his
watchfulness and penetration. Sorely was poor Silas puzzled on such occasions by the
incompatible demands of love.

1. Which choice best describes a major theme 4. The narrator uses the phrase “making trial
of the passage? of everything” (line 7) to present Eppie as
A. The corrupting influence of a A. friendly.
materialistic society B. curious.
B. The moral purity of young children C. disobedient.
C. The bittersweet brevity of childhood D. judgmental.
naïveté
D. The restorative power of parental love 5. According to the narrator, one
consequence of Silas adopting Eppie is that
2. As compared with Silas’s gold, Eppie is he
portrayed as having more A. has renounced all desire for money.
A. vitality. B. better understands his place in nature.
B. durability. C. seems more accepting of help from
C. protection. others.
D. self–sufficiency. D. looks forward to a different kind of
future.
3. Which statement best describes a
technique the narrator uses to represent 6. Which choice provides the best evidence
Silas’s character before he adopted Eppie? for the answer to the previous question?
A. The narrator emphasizes Silas’s former A. Lines 6-7 (“The gold . . . itself”)
obsession with wealth by depicting his B. Lines 7-10 (“but Eppie . . . years”)
gold as requiring certain behaviors on C. Lines 24-26 (“Then . . . stillness”)
his part. D. Lines 36-38 (“shapes . . . for”)
B. The narrator underscores Silas’s 7. What function does the second paragraph
former greed by describing his gold as (lines 18-31) serve in the passage as a
seeming to reproduce on its own. whole?
C. The narrator hints at Silas’s former A. It presents the particular moment at
antisocial attitude by contrasting his which Silas realized that Eppie was
present behavior toward his neighbors changing him.
with his past behavior toward them. B. It highlights Silas’s love for Eppie by
D. The narrator demonstrates Silas’s depicting the sacrifices that he makes
former lack of self-awareness by for her.
implying that he is unable to recall life C. It illustrates the effect that Eppie has
before Eppie. on Silas by describing the interaction
between them.
D. It reveals a significant alteration in the
relationship between Silas and Eppie.
40 | I U P I T B
8. Which choice provides the best evidence B. expanding awareness and Silas’s
for the answer to the previous question? increasing engagement with life.
A. Lines 1-6 (“Unlike . . . her”) C. boundless energy and Silas’s insatiable
B. Lines 18-22 (“And when . . . flowers”) desire for wealth.
C. Lines 27-28 (“Sitting . . . again”) D. physical growth and Silas’s painful
D. Lines 32-34 (“As the . . . perception of his own mortality.
consciousness”)
10. As used in line 39, “fine” most nearly
9. In describing the relationship between means
Eppie and Silas, the narrator draws a A. acceptable.
connection between Eppie’s B. delicate.
A. physical vulnerability and Silas’s C. ornate.
emotional fragility. D. keen.

Questions 11-20 are based on the following passage.

NASA: A Space Program with Down-to-Earth Benefits


The National Aeronautics and Space Administration (NASA) is a US government agency whose
budget is frequently (11) many times contested. Many people think of NASA’s programs as trivial.
In truth, the agency has a widespread positive (12) effect on society by serving as a catalyst for
innovation and scientific understanding, (13) to create jobs, and showing humanity its place within
the universe.

11. 13.
A. NO CHANGE A. NO CHANGE
B. Oftentimes B. creating jobs,
C. Repeatedly C. for job creation,
D. DELETE the underlined portion. D. the creation of jobs,
12.
A. NO CHANGE
B. affect on
C. effect to
D. affects on

In 1958, the program’s first year, very few people believed that it was even possible for a
manned spacecraft to leave the atmosphere and orbit Earth. But by initiating and collaborating on
projects such as the Apollo Moon missions, the space shuttle program, the Hubble Space (14)
Telescope, and unmanned planetary exploration, NASA has continually challenged its scientists and
engineers to do things that were previously thought impossible. All along, these NASA projects have
(15) greatly increased international cooperation. A short list of inventions elaborated by NASA
includes communications satellites, invisible braces, and cordless tools. All these inventions (16)
spawns new industries, and with those industries, jobs. NASA also sponsors the Small Business
Innovation Research and Small Business Technology Transfer programs, which are specifically
designed to support technological development in the private sector.

14. 15. Which choice most effectively sets up the


A. NO CHANGE list of examples that follows in the next
B. Telescope; and sentence?
C. Telescope and;
D. Telescope and,

B E R A N I B E R J U A N G | 41
A. NO CHANGE 16.
B. garnered national publicity for the A. NO CHANGE
agency. B. Spawned
C. generated a steady stream of new C. has spawned
technology. D. spawning
D. made a lot of money for the agency.

[1] A report by the Space Foundation estimated that NASA contributed $180 billion to the
economy in 2005. [2] More than 60 percent of the contribution (17) coming from commercial goods
and services created by companies using space-related technology. [3] This translates as excellent
returns from an agency that received approximately 17.7 billion in tax dollars in 2014. [4] This
investment by taxpayers enhances not only the national economy but also the United States’
competitiveness in the international market. [5] Moreover, the benefits of NASA funding extend
beyond the purely economic, as astrophysicist Neil de Grasse Tyson indicated in his testimony
before the US Senate: “For . . . a penny on a dollar—we can transform the country from a sullen,
dispirited nation, weary of economic struggle, to one where it has reclaimed its twentieth-century
birthright to dream of tomorrow.”

17. 18. To make this paragraph most logical,


A. NO CHANGE sentence 1 should be placed
B. which came A. where it is now.
C. to come B. after sentence 2.
D. came C. after sentence 3.
D. after sentence 4.

Tyson’s expansive vision for the agency hints at another mission of NASA’s, illuminated in this
observation by Apollo 14 astronaut Edgar Mitchell: “You develop an instant global consciousness,
a people orientation, an intense dissatisfaction with the state of the world, and a compulsion to do
something about it.” (19) With world population topping seven billion, humanity is in need of some
perspective. (20) Therefore, we should continue to support NASA not only for practical reasons but
also because it is a necessary vehicle for increasing our awareness of how we can fulfill our
responsibilities to the planet and each other.

19. At this point, the writer is considering C. No, because it undermines the
adding the following sentence. passage’s claim about the economic
In addition, NASA has facilities in benefits of NASA’s work.
Washington, DC, Florida, Texas, California, D. No, because it blurs the paragraph’s
and other states. focus by introducing information that
Should the writer make this addition here? does not support the paragraph’s
A. Yes, because it serves as a claim about the importance of NASA’s
counterargument to the quotation work.
from astrophysicist Neil de Grasse 20.
Tyson. A. NO CHANGE
B. Yes, because it reinforces the B. Instead,
passage’s point about the importance C. For example,
of NASA’s work. D. However,

42 | I U P I T B
Questions 21-25 are based on the following passage.

The Ganga all through its route is filled with innumerable points of interest. Its inspiring beauty, its
spiritual value, its purity has no ---(21)---. But what distinguishes it so much is the ---(22)--- help it
has been in bringing ---(23)---- a sense of unity among the ---(24)--- and thousands of people who
live ---(25)--- it.

21. In each of the following passages, there are A. round


blanks each of which has been umbered. B. in
Find out the appropriate words. C. about
A. opponents D. up
B. enemies
C. adversaries 24. In each of the following passages, there are
D. rivals blanks each of which has been numbered.
Find out the appropriate words.
22. In each of the following passages, there are A. hundreds
blanks each of which has been numbered. B. lakhs
Find out the appropriate words. C. millions
A. tremendous D. dozens
B. copious
C. spacious 25. In each of the following passages, there are
D. voluminous blanks each of which has been numbered.
Find out the appropriate words.
23. In each of the following passages, there are A. alongside
blanks each of which has been numbered. B. at
Find out the appropriate words. C. by
D. amidst

26. Which of the following is most similar in 28. Find the two statements that together
meaning to particular prove:
A. special Sandy is a country music singer.
B. simple 1. Sandy sings in a band.
C. careful 2. Sandy likes country music the best.
D. specific 3. Difficult music such as country and
E. obvious classical require a strongly trained
voice.
27. All of the words in the box below are alike 4. Sandy’s band plays country songs.
in some way. 5. Sandy has attended singing lessons for
Vigilant Precise many years.
Meticulous Particular A. 1 & 3
Which of the following options could not B. 2 & 5
be in the box? C. 3 & 5
A. careful D. 1 & 4
B. thorough E. 3 & 4
C. detailed
D. perfectionistic
E. hasty
B E R A N I B E R J U A N G | 43
29. AIRCRAFT is to TRANSPORT as NEWSPAPER Louisiana, 71.7 years. If a newlywed couple
is to from Louisiana were to begin their family in
A. typing Hawaii, therefore, their children would be
B. publishing expected to live longer than would be the
C. editor case if the family remained in Louisiana
D. newsroom Which of the following, if true, would most
E. media seriously weaken the conclusion drawn in
the passage?
30. In a foreign language A. Insurance company statisticians do not
MISD KUMP GOTH means MERRY believe that moving to Hawaii will
CHRISTMAS DAD. significantly lengthen the average
GOTH WUIM FREK KUMP means DAD IS Louisianan's life.
MERRY TODAY. B. The governor of Louisiana has falsely
KOMP WUIM FREK KUMP means MUM IS alleged that statistics for his state are
MERRY TODAY. inaccurate.
Which word means CHRISTMAS? C. The longevity ascribed to Hawaii's
A. MISD current population is attributable
B. GOTH mostly to genetically determined
C. KUMP factors.
D. WUIM D. Thirty percent of all Louisianans can
E. FREK expect to live longer than 77 years
E. Most of the Hawaiian Islands have
31. Which of the following best completes the levels of air pollution well below the
passage below? national average for the United States.
In a survey of job applicants, two-fifths
admitted to being at least a little dishonest. 33. The average life expectancy for the United
However, the survey may underestimate States population as a whole is 73.9 years,
the proportion of job applicants who are but children born in Hawaii will live an
dishonest, because____. average of 77 years, and those born in
A. some dishonest people taking the Louisiana, 71.7 years. If a newlywed couple
survey might have claimed on the from Louisiana were to begin their family in
survey to be honest Hawaii, therefore, their children would be
B. some generally honest people taking expected to live longer than would be the
the survey might have claimed on the case if the family remained in Louisiana.
survey to be dishonest Which of the following statements, if true,
C. some people who claimed on the would most significantly strengthen the
survey to be at least a little dishonest conclusion drawn in the passage?
may be very dishonest A. As population density increases in
D. some people who claimed on the Hawaii, life expectancy figures for that
survey to be dishonest may have been state are likely to be revised
answering honestly downward.
E. some people who are not job B. Environmental factors tending to favor
applicants are probably at least a little longevity are abundant in Hawaii and
dishonest less numerous in Louisiana.
C. Twenty-five percent of all Louisianans
32. The average life expectancy for the United who move to Hawaii live longer than
States population as a whole is 73.9 years, 77 years.
but children born in Hawaii will live an
average of 77 years, and those born in

44 | I U P I T B
D. Over the last decade, average life 35. A program instituted in a particular state
expectancy has risen at a higher rate allows parents to prepay their children's
for Louisianans than for Hawaiians. future college tuition at current rates. The
E. Studies show that the average life program then pays the tuition annually for
expectancy for Hawaiians who move the child at any of the state's public
permanently to Louisiana is roughly colleges in which the child enrolls. Parents
equal to that of Hawaiians who remain should participate in the program as a
in Hawaii. means of decreasing the cost for their
children's college education.
34. Insurance Company X is considering issuing Which of the following, if true, is the most
a new policy to cover services required by appropriate reason for parents NOT to
elderly people who suffer from diseases participate in the program?
that afflict the elderly. Premiums for the A. the parents are unsure about which
policy must be low enough to attract public college in the state the child will
customers. Therefore, Company X is attend.
concerned that the income from the B. The amount of money accumulated by
policies would not be sufficient to pay for putting the prepayment funds in an
the claims that would be made. interest-bearing account today will be
Which of the following strategies would be greater than the total cost of tuition
most likely to minimize Company X's losses for any of the public colleges when the
on the policies? child enrolls.
A. Attracting middle-aged customers C. The annual cost of tuition at the state's
unlikely to submit claims for benefits public colleges is expected to increase
for many years. at a faster rate than the annual
B. Insuring only those individuals who did increase in the cost of living.
not suffer any serious diseases as D. Some of the state's public colleges are
children contemplating large increases in
C. Including a greater number of services tuition next year.
in the policy than are included in other E. The prepayment plan would not cover
policies of lower cost the cost of room and board at any of
D. Insuring only those individuals who the state's public colleges.
were rejected by other companies for
similar policies
E. Insuring only those individuals who are
wealthy enough to pay for the medical
services

36. Feeding Information for Boarded Pets The table above shows the kinds of foods
Fed both that are fed to the cats and dogs currently
Fed only Total
wet and boarded at a pet care facility. What fraction
dry food
dry food of the dogs are fed only dry food?
16 2
Cats 5 11 A.
41
25 2
Dogs 2 23 B.
25
7
Total 7 34 41 C.
41
2
D.
7

B E R A N I B E R J U A N G | 45
37. A certain package requires 3 centimeters of Questions 42 and 43 refer to the following
tape to be closed securely. What is the information.
maximum number of packages of this type
that can be secured with 6 meters of tape?
(1 meter = 100 cm)
A. 100
B. 150
C. 200
D. 300

38. (𝑥 2 − 3) − (−3𝑥 2 + 5)
Which of the following expressions is
equivalent to the one above ?
A. 4𝑥 2 – 8
B. 4𝑥 2 – 2 The scatterplot above shows the densities of 7
C. −2𝑥 2 – 8 planetoids, in grams per cubic centimeter, with
D. −2𝑥 2 – 2 respect to their average distances from the Sun
in astronomical units (AU). The line of best fit is
39. A market researcher selected 200 people at also shown.
random from a group of people who
indicated that they liked a certain book. 41. According to the scatterplot, which of the
The 200 people were shown a movie based following statements is true about the
on the book and then asked whether they relationship between a planetoid’s average
liked or disliked the movie. Of those distance from the Sun and its density?
surveyed, 95% said they disliked the movie. A. Planetoids that are more distant from
Which of the following inferences can the Sun tend to have lesser densities.
appropriately be drawn from this survey B. Planetoids that are more distant from
result? the Sun tend to have greater densities.
A. At least 95% of people who go see C. The density of a planetoid that is twice
movies will dislike this movie. as far from the Sun as another
B. At least 95% of people who read books planetoid is half the density of that
will dislike this movie. other planetoid.
C. Most people who dislike this book will D. The distance from a planetoid to the
like this movie. Sun is unrelated to its density.
D. Most people who like this book will
dislike this movie. 42. An astronomer has discovered a new
planetoid about 1.2 AU from the Sun.
40. Which of the following ordered pairs (𝑥, 𝑦) According to the line of best fit, which of
satisfies the inequality 5𝑥 − 3𝑦 < 4 ? the following best approximates the
I. (1, 1) density of the planetoid, in grams per cubic
II. (2, 5) centimeter?
III. (3, 2) A. 3.6
A. I only B. 4.1
B. II only C. 4.6
C. I and II only D. 5.5
D. I and III only

46 | I U P I T B
43. In the equation (𝑎𝑥 + 3)2 = 36, a is a 47. A customer paid $53.00 for a jacket after a
constant. If 𝑥 = −3 is one solution to the 6 percent sales tax was added. What was
equation, what is a possible value of 𝑎? the price of the jacket before the sales tax
A. -11 was added?
B. -5 A. $47.60
C. -1 B. $50.00
D. 0 C. $52.60
D. $52.84
44. 9𝑎𝑥 + 9𝑏 − 6 = 21
Based on the equation above, what is the 48. Theresa ran on a treadmill for thirty
value of 𝑎𝑥 + 𝑏 ? minutes, and her time and speed are
A. 3 shown on the graph below. According to
B. 6 the graph, which of the following
C. 8 statements is NOT true concerning
D. 12 Theresa’s run?

45. Lani spent 15% of her 8-hour workday in


meetings. How many minutes of her
workday did she spend in meetings?
A. 1.2
B. 15
C. 48
D. 72

46. A software company is selling a new game


in a standard edition and a collector’s A. Theresa ran at a constant speed for
edition. The box for the standard edition five minutes.
has a volume of 20 cubic inches, and the B. Theresa’s speed was increasing for a
box for the collector’s edition has a volume longer period of time than it was
of 30 cubic inches. The company receives decreasing.
an order for 75 copies of the game, and the C. Theresa’s speed decreased at a
total volume of the order to be shipped is constant rate during the last five
1,870 cubic inches. Which of the following minutes.
systems of equations can be used to D. Theresa’s speed reached its maximum
determine the number of standard edition during the last ten minutes.
games, s, and collector’s edition games, c,
that were ordered? 49. In the figure below, what is the value of 𝑥?
A. 75 − 𝑠 = 𝑐
20𝑠 + 30𝑐 = 1,870
B. 75 − 𝑠 = 𝑐
30𝑠 + 20𝑐 = 1,870
C. 𝑠 − 𝑐 = 75
25(𝑠 + 𝑐) = 1,870
D. 𝑠 − 𝑐 = 75
30𝑠 + 20𝑐 = 1,870
A. 45
B. 90
C. 100
D. 105

B E R A N I B E R J U A N G | 47
50. If 50 one-cent coins were stacked on top of
each other in a column, the column would
7
be approximately 3 inches tall. At this
8
rate, which of the following is closest to the
number of one-cent coins it would take to
make an 8-inch-tall column?
A. 75
B. 100
C. 200
D. 390 A. 10%
B. 7%
𝑏
51. If 𝑎 – 𝑏 = 12 and = 10, what is the C. 4%
2
value of 𝑎 + 𝑏 ? D. 1%
A. 2
B. 12 Questions 55 and 56 refer to the following
C. 32 information.
D. 52
√ℎ𝑤
Mosteller’s formula: 𝐴 =
60
52. 𝑌 = 19.99 + 1.50𝑥 4+𝑤
Current’s formula: 𝐴 =
The equation above models the total cost 30

𝑦, in dollars, that a company charges a


customer to rent a truck for one day and The formulas above are used in medicine to
drive the truck 𝑥 miles. The total cost estimate the body surface area A, in square
consists of a flat fee plus a charge per mile meters, of infants and children whose weight w
driven When the equation is graphed in the ranges between 3 and 30 kilograms and whose
𝑥𝑦- plane, what does the 𝑦-intercept of the height h is measured in centimeters. Percent of
graph represent in terms of the model? total expenses spent on programs
A. A flat fee of $19.99
B. A charge per mile of $1.50 55. Based on Current’s formula, what is 𝑤 in
C. A charge per mile of $19.99 terms of 𝐴?
D. Total daily charges of $21.49 A. 𝑤 = 30𝐴 − 4
B. 𝑤 = 30𝐴 + 4
C. 𝑤 = 30(𝐴 – 4)
53. A circle in the 𝑥𝑦‑plane has equation
D. 𝑤 = 30(𝐴 + 4)
(𝑥 + 3)2 + (𝑦 − 1)2 = 25. Which of
the following points does NOT lie in the
56. If Mosteller’s and Current’s formulas give
interior of the circle?
the same estimate for A, which of the
A. (-7, 3)
following expressions is equivalent to √ℎ𝑤
B. (-3, 1)
?
C. (0, 0) 4+𝑤
D. (3, 2) A.
2
4+𝑤
B.
1800
54. The scatterplot below shows data for ten 4+𝑤
C.
charities along with the line of best fit. For 2
D. 2(4 + 𝑤)
the charity with the greatest percent of
(4+𝑤)2
total expenses spent on programs, which of E.
2
the following is closest to the difference of
the actual percent and the percent
predicted by the line of best fit?

48 | I U P I T B
57. The scatterplot below shows the numbers for all 50 states was 26.95%. What is the
of grams of both total protein and total fat difference between the median percent of
for eight sandwiches on a restaurant menu. residents who earned a bachelor’s degree
The line of best fit for the data is also or higher for these 7 states and the median
shown. According to the line of best fit, for all 50 states?
which of the following is closest to the A. 0.05%
predicted increase in total fat, in grams, for B. 0.95%
every increase of 1 gram in total protein? C. 1.22%
D. 7.45%

59. A cylindrical can containing pieces of fruit is


filled to the top with syrup before being
sealed. The base of the can has an area of
75 𝑐𝑚2 , and the height of the can is 10 𝑐𝑚.
If 110 𝑐𝑚3 of syrup is needed to fill the can
to the top, which of the following is closest
to the total volume of the pieces of fruit in
A. 2.5 the can?
B. 2.0 A. 7.5 𝑐𝑚3
C. 1.5 B. 185 𝑐𝑚3
D. 1.0 C. 640 𝑐𝑚3
D. 750 𝑐𝑚3
58. Percent of Residents Who Earned a
Bachelor’s Degree or Higher 60. ℎ(𝑡) = −16𝑡2 + 110𝑡 + 72
State Percent of residents The function above models the height h, in
A 21.9% feet, of an object above ground t seconds
B 27.9% after being launched straight up in the air.
C 25.9% What does the number 72 represent in the
D 19.5% function?
E 30.1% A. The initial height, in feet, of the object
F 36.4% B. The maximum height, in feet, of the
G 35.5% object
A survey was given to residents of all 50 C. The initial speed, in feet per second, of
states asking if they had earned a the object
bachelor’s degree or higher. The results D. The maximum speed, in feet per
from 7 of the states are given in the table second, of the object
above. The median percent of residents
who earned a bachelor’s degree or higher

61. 𝑸𝒖𝒂𝒏𝒕𝒊𝒕𝒚 𝑨 𝑸𝒖𝒂𝒏𝒕𝒊𝒕𝒚 𝑩


12% 𝑜𝑓 25 25% 𝑜𝑓 (3/8 × 32)

A. Quantity A is greater.
B. Quantity B is greater.
C. The two quantities are equal.
D. The relationship cannot be determined from the information given.

B E R A N I B E R J U A N G | 49
62. 𝒎 > 𝟖
𝑸𝒖𝒂𝒏𝒕𝒊𝒕𝒚 𝑨 𝑸𝒖𝒂𝒏𝒕𝒊𝒕𝒚 𝑩

𝑚 + √51 √64 + 7

A. Quantity A is greater.
B. Quantity B is greater.
C. The two quantities are equal.
D. The relationship cannot be determined from the information given.

63. The side of equilateral triangle 𝑻 is the same as the length of a side of square 𝑺.
𝑸𝒖𝒂𝒏𝒕𝒊𝒕𝒚 𝑨 𝑸𝒖𝒂𝒏𝒕𝒊𝒕𝒚 𝑩

𝑇ℎ𝑒 𝑎𝑟𝑒𝑎 𝑜𝑓 𝑡𝑟𝑖𝑎𝑛𝑔𝑙𝑒 𝑇 𝑇ℎ𝑒 𝑎𝑟𝑒𝑎 𝑜𝑓 𝑠𝑞𝑢𝑎𝑟𝑒 𝑆

A. Quantity A is greater.
B. Quantity B is greater.
C. The two quantities are equal.
D. The relationship cannot be determined from the information given.

64. 𝑸𝒖𝒂𝒏𝒕𝒊𝒕𝒚 𝑨 𝑸𝒖𝒂𝒏𝒕𝒊𝒕𝒚 𝑩


𝑇ℎ𝑒 𝑛𝑢𝑚𝑏𝑒𝑟 𝑜𝑓 𝑑𝑖𝑠𝑡𝑖𝑛𝑐𝑡 𝑝𝑟𝑖𝑚𝑒 𝑇ℎ𝑒 𝑛𝑢𝑚𝑏𝑒𝑟 𝑜𝑓 𝑑𝑖𝑠𝑡𝑖𝑛𝑐𝑡 𝑝𝑟𝑖𝑚𝑒
𝑓𝑎𝑐𝑡𝑜𝑟𝑠 𝑜𝑓 28 𝑓𝑎𝑐𝑡𝑜𝑟𝑠 𝑜𝑓 36

A. Quantity A is greater.
B. Quantity B is greater.
C. The two quantities are equal.
D. The relationship cannot be determined from the information given.

65. 𝒎≥𝟎
𝑸𝒖𝒂𝒏𝒕𝒊𝒕𝒚 𝑨 𝑸𝒖𝒂𝒏𝒕𝒊𝒕𝒚 𝑩

(3)32𝑚 3𝑚+2

A. Quantity A is greater.
B. Quantity B is greater.
C. The two quantities are equal.
D. The relationship cannot be determined from the information given.

66. During a particular period, water enters a partially filled reservoir at a constant rate through a
mountain stream. At the same time, water is pumped out of the reservoir at a constant rate
through an outlet pipe. At what rate, in gallons per minute, is the amount of water in the
reservoir increasing?
(1) The amount of water initially in the reservoir is 1800 gallons.
(2) Water is pumped into the reservoir at a rate of 8 gallons per minute and out of the
reservoir at a rate of 20 gallons every 3 minutes.
A. Statement (1) ALONE is sufficient, but statement (2) alone is not sufficient.
B. Statement (2) ALONE is sufficient, but statement (1) alone is not sufficient.
C. BOTH statements TOGETHER are sufficient, but NEITHER statement ALONE is sufficient.
D. EACH statement ALONE is sufficient.
E. Statements (1) and (2) TOGETHER are NOT sufficient.

50 | I U P I T B
67. 𝑙, 17.2, 12.2, 7.2, 22.2 - What is the value of 𝑙 in the list above?
(1) 𝑙 > 7.2
(2) The median of the numbers in the list is 14.7.
A. Statement (1) ALONE is sufficient, but statement (2) alone is not sufficient.
B. Statement (2) ALONE is sufficient, but statement (1) alone is not sufficient.
C. BOTH statements TOGETHER are sufficient, but NEITHER statement ALONE is sufficient.
D. EACH statement ALONE is sufficient.
E. Statements (1) and (2) TOGETHER are NOT sufficient.

68. If 𝑚 is an integer, is 𝑚 odd?


(1) is an odd integer.
(2) is an even integer.
A. Statement (1) ALONE is sufficient, but statement (2) alone is not sufficient.
B. Statement (2) ALONE is sufficient, but statement (1) alone is not sufficient.
C. BOTH statements TOGETHER are sufficient, but NEITHER statement ALONE is sufficient.
D. EACH statement ALONE is sufficient.
E. Statements (1) and (2) TOGETHER are NOT sufficient.

69. A company offered a training program to all of its 120 employees. How many employees
completed the training?
(1) Of all employees, 30 percent completed more than half of the training.
(2) Of all employees, 30 percent of those who began the training completed it.
A. Statement (1) ALONE is sufficient, but statement (2) alone is not sufficient.
B. Statement (2) ALONE is sufficient, but statement (1) alone is not sufficient.
C. BOTH statements TOGETHER are sufficient, but NEITHER statement ALONE is sufficient.
D. EACH statement ALONE is sufficient.
E. Statements (1) and (2) TOGETHER are NOT sufficient.

70. In 1854, during the California gold rush, each ounce of gold was worth $20, and the largest
known mass of gold found in California was worth $62,400 in that year. What was the weight,
in pounds, of this mass of gold? (16 ounces = 1 pound)

71. Line 𝑡 is shown in the 𝑥𝑦-plane below.

What is the slope of line 𝑡?

B E R A N I B E R J U A N G | 51
72. The score on a trivia game is obtained by subtracting the number of incorrect answers from
twice the number of correct answers. If a player answered 40 questions and obtained a score
of 50, how many questions did the player answer correctly?

73. Point 𝐶 is the center of the circle below. What fraction of the area of the circle is the area of
the shaded region?

74. 𝑦 = 𝑥2 − 4𝑥 + 4
𝑦 = 4 − 𝑥
If the ordered pair (𝑥, 𝑦) satisfies the system of equations above, what is one possible value of
𝑥?

3
75. In the figure below, 𝑡𝑎𝑛 𝐵 = . If 𝐵𝐶 = 15 and 𝐷𝐴 = 4, what is the length of ̅̅̅̅
𝐷𝐸 ?
4

52 | I U P I T B
SET-2

Questions 1-10 are based on the following passage and supplementary material.

This passage is adapted from David Rotman "How Technology Is Destroying Jobs” ©2013 by MIT
Technology Review.

Line MIT business scholars Erik Brynjolfsson and Andrew McAfee have argued that
Impressive advances in computer technology — from improved one industrial robotics
to automated translation services—are largely behind the sluggish employment growth
of the last 10 to 15 years. Even more ominous for workers, they foresee dismal
5 prospects for many types of jobs as these powerful new technologies arc increasingly
adopted not only in manufacturing, clerical, and retail work but in professions suds as
law, financial services, education, and medicine
That robots, automation, and software can replace people might seem obvious to
anyone who's worked in automotive manufacturing or as a travel agent. Brynjolfsson
10 and McAfee's claim is more troubling and controversial. They believe that rapid
technological change has been destroying jobs faster than it is creating them,
contributing to the stagnation of median income and the growth of inequality to the
United States. And, they suspect something similar is happening in other technologically
advanced countries.
15 As evidence Brynjolfsson and McAfee point to a chart that only an economist could
love. In economics, productivity—the amount of economic value created for a given unit
of input, such as an hour of labor—is a crucial indicator of growth and wealth creation.
It is a measure of progress. On the chart Brynjolfsson likes to show, separate lines
represent productivity and IOU employment in the United States. For years after World
20 War II, the two lines closely tracked each other, with increases in jobs corresponding to
increases in productivity. The pattern is clear, as businesses generated more value from
their workers, the country as a whole became richer, which fueled more economic
activity and created even more jobs. Then, beginning in 2000, the lines diverge;
productivity continues to rise robustly, but employment suddenly wilts. By 2011, a
25 significant gap appears between the two lines, showing economic growth with no
parallel increase in job creation. Brynjolfsson and McAfee call it the "great decoupling."
And Brynjolfsson says he confident that technology is behind both the healthy growth
in productivity and the weak growth in jobs
B E R A N I B E R J U A N G | 53
It’s a startling assertion because it threatens the faith that army economists place
30 in technological progress Brynjolfsson and McAfee still believe that technology boosts
productivity and makes societies wealthier, but they think that it can also have a dark
side: technology progress is eliminating the need for many types of jobs and 'raying the
typical worker nurse off than before. Brynjolfsson can point to a second chart indicating
that median income is failing to rise even as the gross domestic product soars. "It’s the
35 great paradox of out era,” he says. "Productivity is at record levels, innovation has never
been faster, and yet at the same time, we have a filling median income and we have
fewer jobs People are falling behind because technology is advancing so fast and our
skills and organizations aren't keeping up”
While technological changes can be painful for workers whose skins no longer
40 match the needs of employer, Lawrence Katz. a Harvard economist says that no
historical pattern show, these shifts Leading to a net decrease in jobs over an extended
period. Katz has done extensive research on how technological advances hare affected
jobs over the last few centuries—describing, for example, how highly skilled artisans in
the mid• 19th century were displaced by lower-skilled workers in factories. While it can
45 take decades for workers so acquire the expertise needed for new types of
employment, he says, “we never have run out of jobs. There is no long-term trend of
eliminating work for people. Over the long term, employment rates are fairly stable.
People have always been able to create new jobs. People come up with new things to
do."
50 Still, Katz doesn’t dismiss the notion that there is something different about today's
digital technologies—something that could affect an even broader range of work. The
question, he says, is whether economic history will serve as a useful guide. Will the job
disruptions caused by technology be temporary as the workforce adapts, or will we see
science-fiction scenario in which automated processes and robots with superhuman
55 skills take over a broad swath of human tasks? Though Katz expects the historical
pattern to bo1d, it is “genuinely a question,” he says. ”If technology disrupts enough,
who knows what will happen?"

54 | I U P I T B
1. The main purpose of the passage is to 5. As used in line 21, “clear” most nearly
A. examine the role of technology in means
workers’ lives during the last century. A. pure.
B. advocate for better technology to B. keen.
enhance workplace conditions. C. untroubled.
C. argue for changes in how technology is D. unmistakable.
deployed in the workplace.
D. assess the impact of advancements in 6. Which of the following best characterizes
technology on overall job growth. Katz’s attitude toward “today’s digital
technologies” (lines 51)?
2. According to Brynjolfsson and McAfee, A. He is alarmed about countries’
advancements in technology since increasing reliance on them.
approximately the year 2000 have resulted B. He is unconcerned about their effect
in on the economy.
A. low job growth in the United States. C. He is uncertain how they might affect
B. global workplace changes. job growth.
C. more skilled laborers in the United D. He is optimistic that they will spur job
States. creation to a degree not seen since the
D. no global creation of new jobs. mid- nineteenth century.

3. Which choice provides the best evidence 7. Which choice provides the best evidence
for the answer to the previous question? for the answer to the previous question?
A. Lines 1-4 (“MIT . . . years”) A. Lines 42-45 (“Katz . . . factories”)
B. Lines 8-9 (“That . . . agent”) B. Lines 45-46 (“While . . . jobs”)
C. Lines 13-14 (“And . . . countries”) C. Line 48-49 (“People come . . . do”)
D. Lines 21-23 (“as businesses . . . jobs”) D. Lines 51-52 (“If . . . happen”)

4. The primary purpose of lines 16-17 (“the 8. According to figure 1, which of the
amount . . . labor”) is to following years showed the widest gap
A. describe a process. between percentages of productivity and
B. highlight a dilemma. employment?
C. clarify a claim. A. 1987
D. explain a term. B. 1997
C. 2007
D. 2013

B E R A N I B E R J U A N G | 55
9. Which statement is supported by figure 2? D. Of the three countries, the United
A. The country with the greatest growth States had the greatest output per
in output per manufacturing worker manufacturing worker for each of the
from 1960 to 1990 was Germany. years shown.
B. Japan experienced its smallest
increase in output per manufacturing 10. As used in line 51, “range” most nearly
worker from 2000 to 2011. means
C. Each of the three countries A. region.
experienced an increase in its output B. scope.
per manufacturing worker from 1960 C. distance.
to 2011. D. position.

Questions 11-15 are based on the following passage and supplementary material.

Professional Development: A Shared Responsibility


New theories, (11) new practices too, and technologies are transforming the twenty-first-
century workplace at lightning speed. To perform their jobs successfully in this dynamic
environment, workers in many (12) fields—from social services to manufacturing, must continually
acquire relevant knowledge and update key skills. This practice of continued education, also known
as professional development, benefits not only employees but also their employers. (13)
Accordingly, meaningful professional development is a shared responsibility: it is the responsibility
of employers to provide useful programs, and it is also the responsibility of employees to take
advantage of the opportunities offered to them.
Critics of employer-provided professional development argue that employees (14) might
consider a popular career path. If employees find themselves falling behind in the workplace, these
critics (15) contend. Then it is the duty of those employees to identify, and even pay

11. 14. Which choice best establishes the


A. NO CHANGE argument that follows?
B. also new practices, A. NO CHANGE
C. in addition to practices, B. should lean heavily on their
D. practices, employers.
12. C. must be in charge of their own careers.
A. NO CHANGE D. will be ready for changes in the job
B. fields market.
C. fields, 15.
D. fields; A. NO CHANGE
13. B. contend; then
A. NO CHANGE C. contend then
B. Nevertheless, D. contend, then
C. Regardless,
D. Similarly,

Questions 16-20 are based on the following passage.

for, appropriate resources to (16) show them how and why they are falling behind and what they
should do about it. This argument ignores research pointing to high employee turnover and training
of new staff as significant costs plaguing employers in many fields. Forward- thinking employers
recognize the importance of investing in the employees they have rather than hiring new staff when
the skills of current workers (17) get old and worn out.

56 | I U P I T B
16. 17.
A. NO CHANGE A. NO CHANGE
B. address their deficiencies. B. are no good anymore.
C. deal with their flaws and C. become obsolete.
shortcomings. D. have lost their charm.
D. allow them to meet their employers’
needs in terms of the knowledge they
are supposed to have.

The most common forms of professional development provided to employees (18) includes
coaching, mentoring, technical assistance, and workshops. Some employers utilize several
approaches simultaneously, developing a framework that suits the particular needs of their
employees. (19) Around the same time, the figure illustrates a simple yet comprehensive
professional-development model created for special education personnel. As the figure suggests,
(20) receiving coaching and consultation is the overarching framework, while the opportunity to
belong to professional networks and participate in activities such as foundation and skill-building
workshops is relatively unimportant.

18. Which choice makes the writer’s 19.


description of the figure most accurate? A. NO CHANGE
A. NO CHANGE B. include
B. participation in foundation and skill- C. including
building workshops is the overarching D. has included
framework within which staff receive 20.
coaching and consultation as well as A. NO CHANGE
the opportunity to belong to a B. Besides that,
professional network. C. Nevertheless,
C. membership in a professional network D. DELETE the underlined portion and
is the overarching framework within begin the sentence with a capital
which staff receive coaching and letter.
consultation as well as the opportunity
to attend foundation and skill-building
workshops.
D. receiving coaching and consultation is
the overarching framework within
which staff have the opportunity to
belong to a professional network as
well as attend foundation and skill-
building workshops.
B E R A N I B E R J U A N G | 57
Questions 21-25 are based on the following passage.

From ancient times, the river has been a bond ---(21)--- the provinces that ---(22)--- on its banks.
The people who lived in the se ---(23)--- spoke different ---(24)--- different customs and ate different
---(25)--- of food. All this should have made them strangers to one another. But it did not.

21. In each of the following passages, there are A. palaces


blanks each of which has been numbered. B. sites
Find out the appropriate words. C. lodgings
A. mixing D. spots
B. uniting
C. covering 24. In each of the following passages, there are
D. merging blanks each of which has been numbered.
Find out the appropriate words.
22. In each of the following passages, there are A. categories
blanks each of which has been numbered. B. forms
Find out the appropriate words. C. sets
A. stand D. types
B. rest
C. stretch 25. In each of the following passages, there are
D. lie blanks each of which has been numbered.
Find out the appropriate words.
23. In each of the following passages, there are A. chased
blanks each of which has been numbered. B. followed
Find out the appropriate words. C. pursued
D. obeyed

26. Jordan’s cat is bigger than Amy’s cat, but is (“was good building”) or “Ern gangro
smaller than Jenna’s cat. Fiona’s cat is the yatju!”
same size as Wendy’s cat, which is bigger If gangro symbolises the opposite of
than Amy’s cat, but smaller than Jordan’s. gengro, what does “Ern gangro yatju”
If bigger cats are faster, but smaller cats are mean?
more obedient, which of the following lists A. was bad building
the fastest and most obedient cat? B. was hard building
A. Fiona & Jenna C. was fun building
B. Amy & Jordan D. was cold building
C. Wendy & Jenna E. was happy building
D. Jenna & Amy
E. None 28. Which of the following words has a similar
meaning to outgoing
27. In ancient Egypt, pyramid workers would A. extraverted
say “gengro yatju” when wishing other B. contained
pyramid workers “good building”. Upon C. outside
the pyramid workers’ return, the children D. exit
would ask “Ern yatju gengro?”, which E. introverted
meant “Was building good”? The pyramid
workers would either reply “Ern gengro”

58 | I U P I T B
29. Four of the following words are alike in which blows on the front windshield, was
some way. turned on full force.
Which is the odd word out? Which of the following, if true, most
A. petrified seriously jeopardizes the validity of the
B. fearful explanation for the speed with which the
C. terrified ice melted?
D. daring A. The side windows had no ice
E. scared condensation on them
B. Even though no attempt was made to
30. All of the words in the box below are alike defrost the back window, the ice there
in some way. melted at the same rate as did the ice
felony offense on the front windshield.
sin wrongdoing C. The speed at which ice on a window
Which of the following options could not go melts increases as the temperature of
in the box? the air blown on the window increases
A. crime D. The warm air from the defrosting vent
B. immoral for the front windshield cools rapidly
C. misdemeanour as it dissipates throughout the rest of
D. prison the car.
E. misconduct E. The defrosting vent operates
efficiently even when the heater,
31. Company Alpha buys free-travel coupons which blows warm air toward the feet
from people who are awarded the coupons or faces of the driver and passengers,
by Bravo Airlines for flying frequently on is on
Bravo airplanes. The coupons are sold to
people who pay les for the coupons than 33. To prevent some conflicts of interest,
they would pay by purchasing tickets from Congress could prohibit high-level
Bravo. This making of coupons results in government officials from accepting
lost revenue for Bravo. positions as lobbyists for three years after
To discourage the buying and selling of such officials leave government service.
free-travel coupons, it would be best for One such official concluded, however, that
Bravo Airlines to restrict the such a prohibition would be unfortunate
A. number of coupons that a person can because it would prevent high-level
be awarded in a particular year government officials from earning a
B. use of the coupons to those who were livelihood for three years.
awarded the coupons and members of The official's conclusion logically depends
their immediate families on which of the following assumptions?
C. days that the coupons can be used to A. Laws should not restrict the behavior
Monday through Friday of former government officials.
D. amount of time that the coupons can B. Lobbyists are typically people who
be used after they are issued have previously been high-level
E. number of routes on which travelers government officials.
can use the coupons C. Low-level government officials do not
often become lobbyists when they
32. The ice on the front windshield of the car leave government service.
had formed when moisture condensed D. High-level government officials who
during the night. The ice melted quickly leave government service are capable
after the car was warmed up the next of earning a livelihood only as
morning because the defrosting vent, lobbyists.

B E R A N I B E R J U A N G | 59
E. High-level government officials who 35. Meteorite explosions in the Earth's
leave government service are atmosphere as large as the one that
currently permitted to act as lobbyists destroyed forests in Siberia, with
for only three years. approximately the force of a twelve-
megaton nuclear blast, occur about once a
34. A conservation group in the United States century.
is trying to change the long-standing image The response of highly automated systems
of bats as frightening creatures. The group controlled by complex computer programs
contends that bats are feared and to unexpected circumstances is
persecuted solely because they are shy unpredictable.
animals that are active only at night. Which of the following conclusions can
Which of the following, if true, would cast most properly be drawn, if the statements
the most serious doubt on the accuracy of above are true, about a highly automated
the group's contention? nuclear-missile defense system controlled
A. Bats are steadily losing natural by a complex computer program?
roosting places such as caves and A. Within a century after its construction,
hollow trees and are thus turning to the system would react
more developed areas for roosting. inappropriately and might accidentally
B. Bats are the chief consumers of start a nuclear war.
nocturnal insects and thus can help B. The system would be destroyed if an
make their hunting territory more explosion of a large meteorite
pleasant for humans. occurred in the Earth's atmosphere.
C. Bats are regarded as frightening C. It would be impossible for the system
creatures not only in the United States to distinguish the explosion of a large
but also in Europe, Africa, and South meteorite from the explosion of a
America. nuclear weapon.
D. Raccoons and owls are shy and active D. Whether the system would respond
only at night; yet they are not inappropriately to the explosion of a
generally feared and persecuted. large meteorite would depend on the
E. People know more about the behavior location of the blast.
of other greatly feared animal species, E. It is not certain what the system's
such as lions, alligators, and greatly response to the explosion of a large
feared animal species, such as lions, meteorite would be, if its designers did
alligators, and snakes, than they do not plan for such a contingency.
about the behavior of bats.

Questions 36 and 37 refer to the following 36. If 𝑥 food calories is equivalent to 𝑘


information. kilojoules, of the following, which best
Energy per Gram of Typical Macronutrients represents the relationship between 𝑥 and
Macronutrient Calories Kilojoules ?
Protein 4.0 16.7 A. 𝑘 = 0.24𝑥
Fat 9.0 37.7 B. 𝑘 = 4.2𝑥
Carbohydrate 4.0 16.7 C. 𝑥 = 4.2𝑘
The table above gives the typical amounts of D. 𝑥𝑘 = 4.2
energy per gram, expressed in both food
calories and kilojoules, of the three
macronutrients in food.
60 | I U P I T B
37. If the 180 food calories in a granola bar 41. The manager of an online news service
come entirely from 𝑝 grams of protein, 𝑓 received the report below on the number
grams of fat, and 𝑐 grams of carbohydrate, of subscriptions sold by the service. The
which of the following expresses 𝑓 in terms manager estimated that the percent
of 𝑝 and 𝑐 ? increase from 2012 to 2013 would be
4
A. 𝑓 = 20 + (𝑝 + 𝑐) double the percent increase from 2013 to
9
4 2014. How many subscriptions did the
B. 𝑓 = 20 – (𝑝 + 𝑐)
9 manager expect would be sold in 2014?
4
C. 𝑓 = 20 – (𝑝 − 𝑐) Year Subscriptions sold
9
9 2012 5600
D. 𝑓 = 20 + (𝑝 + 𝑐)
4
2013 5880
A. 6,020
38. The world’s population has grown at an
B. 6,027
average rate of 1.9 percent per year since
C. 6,440
1945. There were approximately 4 billion
D. 6,468
people in the world in 1975. Which of the
following functions represents the world’s
42. 𝑥 + 𝑦 = 75
population 𝑃, in billions of people, 𝑡 years
The equation above relates the number of
since 1975 ? (1 billion = 1,000,000,000)
minutes, 𝑥, Maria spends running each day
A. 𝑃(𝑡) = 4(1.019)𝑡
B. 𝑃(𝑡) = 4(1.9)𝑡 and the number of minutes, 𝑦, she spends
C. 𝑃(𝑡) = 1.19𝑡 + 4 biking each day. In the equation, what does
D. 𝑃(𝑡) = 1.019𝑡 + 4 the number 75 represent?
A. The number of minutes spent running
39. In the 𝑥𝑦‑plane above, a point (not shown) each day
with coordinates (𝑠, 𝑡) lies on the graph of B. The number of minutes spent biking
the linear function 𝑓. If 𝑠 and 𝑡 are positive each day
integers, what is the ratio of 𝑡 to 𝑠? C. The total number of minutes spent
running and biking each day
D. The number of minutes spent biking
for each minute spent running

43. Which of the following is equivalent to


3(𝑥 + 5) − 6 ?
A. 3𝑥– 3
B. 3𝑥– 1
A. 1 to 3 C. 3𝑥 + 9
B. 1 to 2 D. 15𝑥– 6
C. 2 to 1
D. 3 to 1
44. Which of the following complex numbers is
40. 𝑥 = 𝑦 − 3 equal to (5 + 12𝑖) − (9𝑖 2 − 6𝑖), for
𝑥
+ 2𝑦 = 6 𝑖 = √−1 ?
2
Which ordered pair (𝑥, 𝑦) satisfies the A. −14 − 18𝑖
system of equations shown above? B. −4 − 6𝑖
A. ( −3, 0) C. 4 + 6𝑖
B. (0, 3) D. 14 + 18𝑖
C. (6, −3)
D. (36, −6)

B E R A N I B E R J U A N G | 61
𝑥 2 −6𝑥+3 at least 1 senior director. Which of the
45. If 𝑓(𝑥) = ,, what is 𝑓(−1) ?
𝑥−1
following systems of inequalities
A. −5
represents the conditions described if 𝑥 is
B. −2
the number of junior directors and 𝑦 is the
C. 2
number of senior directors?
D. 5
A. 640𝑥 + 880𝑦 ≥ 9,700
𝑥 + 𝑦 ≤ 10
46. A company that makes wildlife videos
𝑥 ≥ 3
purchases camera equipment for $32,400.
𝑦 ≥ 1
The equipment depreciates in value at a B. 640𝑥 + 880𝑦 ≤ 9,700
constant rate for 12 years, after which it is 𝑥 + 𝑦 ≥ 10
considered to have no monetary value. 𝑥 ≥ 3
How much is the camera equipment worth 𝑦 ≥ 1
4 years after it is purchased? C. 640𝑥 + 880𝑦 ≥ 9,700
A. $10,800 𝑥 + 𝑦 ≥ 10
B. $16,200 𝑥 ≤ 3
C. $21,600 𝑦 ≤ 1
D. $29,700 D. 640𝑥 + 880𝑦 ≤ 9,700
𝑥 + 𝑦 ≤ 10
47. 𝑥 2 + 6𝑥 + 4 𝑥 ≤ 3
Which of the following is equivalent to the 𝑦 ≤ 1
expression above?
A. (𝑥 + 3)2 + 5 50. 𝑎𝑥 3 + 𝑏𝑥 2 + 𝑐𝑥 + 𝑑 = 0
B. (𝑥 + 3)2 − 5 In the equation above, 𝑎, 𝑏, 𝑐, and d are
C. (𝑥 − 3)2 + 5 constants. If the equation has roots −1, −3,
D. (𝑥 − 3)2 – 5 and 5, which of the following is a factor of
𝑎𝑥 3 + 𝑏𝑥 2 + 𝑐𝑥 + 𝑑?
48. Ken is working this summer as part of a A. 𝑥 − 1
crew on a farm. He earned $8 per hour for B. 𝑥 + 1
the first 10 hours he worked this week. C. 𝑥 − 3
Because of his performance, his crew D. 𝑥 + 5
1
leader raised his salary to $10 per hour for 𝑥 −2 𝑦 2
51. The expression 1 , where 𝑥 >
the rest of the week. Ken saves 90% of his 𝑥 3 𝑦 −1
earnings from each week. What is the least 1 𝑎𝑛𝑑 𝑦 > 1, is equivalent to which of
number of hours he must work the rest of the following?
the week to save at least $270 for the √𝑦
A. 3
week? √𝑥 2
𝑦 √𝑦
A. 38 B. 3
√𝑥 2
B. 33 𝑦√𝑦
C. 22 C.
𝑥√𝑥
D. 16 √𝑦
D.
𝑥 2 3√𝑥

49. Marisa needs to hire at least 10 staff 1


members for an upcoming project. The 52. The expression 𝑥 2 − 2 can be rewritten
3
1
staff members will be made up of junior as (𝑥 − 𝑘)(𝑥 + 𝑘), where 𝑘 is a
3
directors, who will be paid $640 per week, positive constant. What is the value of 𝑘?
and senior directors, who will be paid $880 A. 2
per week. Her budget for paying the staff B. 6
members is no more than $9,700 per week. C. √2
She must hire at least 3 junior directors and D. √6
62 | I U P I T B
53. The function 𝑓 is defined by 𝑓(𝑥) = (𝑥 +
3)(𝑥 + 1). The graph of 𝑓 in the 𝑥𝑦-plane
is a parabola. Which of the following
intervals contains the 𝑥-coordinate of the
vertex of the graph of 𝑓?
A. −4 < 𝑥 < −3
B. −3 < 𝑥 < 1
C. 1 < 𝑥 < 3
D. 3 < 𝑥 < 4
On what interval did the number of
54. Which of the following expressions is
𝑥 2 −2𝑥−5
figurines decrease the fastest?
equivalent to ? A. Between 1 and 2 months
𝑥−3
20
A. 𝑥 − 5 − B. Between 2 and 3 months
𝑥−2
10 C. Between 3 and 4 months
B. 𝑥 − 5 −
𝑥−2 D. Between 4 and 5 months
8
C. 𝑥 + 1 −
𝑥−3
2 58. In a random sample of 200 cars of a
D. 𝑥 + 1 −
𝑥−3
particular model, 3 have a manufacturing
55. A shipping service restricts the dimensions defect. At this rate, how many of 10,000
of the boxes it will ship for a certain type of cars of the same model will have a
service. The restriction states that for manufacturing defect?
boxes shaped like rectangular prisms, the A. 150
sum of the perimeter of the base of the box B. 200
and the height of the box cannot exceed C. 250
130 inches. The perimeter of the base is D. 300
determined using the width and length of
the box. If a box has a height of 60 inches 59. The scatterplot below shows data collected
and its length is 2.5 times the width, which on the lengths and widths of Iris setosa
inequality shows the allowable width 𝑥, in petals. A line of best fit for the data is also
inches, of the box? shown. Based on the line of best fit, if the
A. 0 < 𝑥 ≤ 10 width of an Iris setosa petal is 19
2 millimeters, what is the predicted length, in
B. 0 < 𝑥 ≤ 11
3 millimeters, of the petal?
1
C. 0 < 𝑥 ≤ 17
2
D. 0 < 𝑥 ≤ 20

56. One pound of grapes costs $2. At this rate,


how many dollars will 𝑐 pounds of grapes
cost?
A. 2𝑐
B. 2 + 𝑐
2
C.
𝑐
𝑐
D.
2
A. 21.10
57. Tracy collects, sells, and trades figurines, B. 31.73
and she tracks the number of figurines in C. 52.83
her collection on the graph below. D. 55.27

B E R A N I B E R J U A N G | 63
60. In the figure below, lines 𝑙 and 𝑚 are A. 120
parallel, 𝑦 = 20, and 𝑧 = 60. What is the B. 100
value of 𝑥? C. 90
D. 80

61. 𝟎<𝒚<𝟏
𝑸𝒖𝒂𝒏𝒕𝒊𝒕𝒚 𝑨 𝑸𝒖𝒂𝒏𝒕𝒊𝒕𝒚 𝑩
𝑦+3 𝑦2 + 2
+𝑦 +𝑦
𝑦+7 𝑦+7
A. Quantity A is greater.
B. Quantity B is greater.
C. The two quantities are equal.
D. The relationship cannot be determined from the information given.

62. 𝟐𝟎
♪𝒂= + 𝟓 𝒂𝒏𝒅 ☀ 𝒂 = 𝒂𝟐 − 𝟔
𝒂
𝑸𝒖𝒂𝒏𝒕𝒊𝒕𝒚 𝑨 𝑸𝒖𝒂𝒏𝒕𝒊𝒕𝒚 𝑩

☀(♪(−4)) ♪(☀(−4))
A. Quantity A is greater.
B. Quantity B is greater.
C. The two quantities are equal.
D. The relationship cannot be determined from information given.

63. 𝒂 = 𝒃𝒄 𝒄 >𝟎
𝑸𝒖𝒂𝒏𝒕𝒊𝒕𝒚 𝑨 𝑸𝒖𝒂𝒏𝒕𝒊𝒕𝒚 𝑩

𝑎 𝑏
A. Quantity A is greater.
B. Quantity B is greater.
C. The two quantities are equal.
D. The relationship cannot be determined from the information given.

64. 𝒂 >𝟏
𝑸𝒖𝒂𝒏𝒕𝒊𝒕𝒚 𝑨 𝑸𝒖𝒂𝒏𝒕𝒊𝒕𝒚 𝑩

3 + (−9) − (−7) 4𝑎
A. Quantity A is greater.
B. Quantity B is greater.
C. The two quantities are equal.
D. The relationship cannot be determined from the information given.

64 | I U P I T B
65. 𝟏 𝒇𝒐𝒐𝒕 = 𝟏𝟐 𝒊𝒏𝒄𝒉𝒆𝒔
𝑸𝒖𝒂𝒏𝒕𝒊𝒕𝒚 𝑨 𝑸𝒖𝒂𝒏𝒕𝒊𝒕𝒚 𝑩

𝐹𝑒𝑒𝑡 𝐼𝑛𝑐ℎ𝑒𝑠
A. Quantity A is greater.
B. Quantity B is greater.
C. The two quantities are equal.
D. The relationship cannot be determined from the information given.

66. Data Sufficiency: Is 𝑥 3 > 𝑥 2 ?


(1) 𝑥 > 0
(2) 𝑥 < 1
A. Statement (1) ALONE is sufficient, but statement (2) alone is not sufficient.
B. Statement (2) ALONE is sufficient, but statement (1) alone is not sufficient.
C. BOTH statements TOGETHER are sufficient, but NEITHER statement ALONE is sufficient.
D. EACH statement ALONE is sufficient.
E. Statements (1) and (2) TOGETHER are NOT sufficient.

67. Data Sufficiency: Is 𝑥𝑦𝑥𝑦 a terminating decimal?


(1) 𝑥 𝑖𝑠 𝑎 𝑚𝑢𝑙𝑡𝑖𝑝𝑙𝑒 𝑜𝑓 2
(2) 𝑦 𝑖𝑠 𝑎 𝑚𝑢𝑙𝑡𝑖𝑝𝑙𝑒 𝑜𝑓 3
A. Statement (1) ALONE is sufficient, but statement (2) alone is not sufficient.
B. Statement (2) ALONE is sufficient, but statement (1) alone is not sufficient.
C. BOTH statements TOGETHER are sufficient, but NEITHER statement ALONE is sufficient.
D. EACH statement ALONE is sufficient.
E. Statements (1) and (2) TOGETHER are NOT sufficient.

68. Data Sufficiency: Is the positive integer 𝑋 divisible by 21?


(1) When 𝑋 is divided by 14, the remainder is 4
(2) When 𝑋 is divided by 15, the remainder is 5
A. Statement (1) ALONE is sufficient, but statement (2) alone is not sufficient.
B. Statement (2) ALONE is sufficient, but statement (1) alone is not sufficient.
C. BOTH statements TOGETHER are sufficient, but NEITHER statement ALONE is sufficient.
D. EACH statement ALONE is sufficient.
E. Statements (1) and (2) TOGETHER are NOT sufficient.

69. Data Sufficiency: If 𝑥 and 𝑦 are positive integers, is 𝑦 odd?


(1) 𝑥 is odd.
(2) 𝑥𝑦 is odd.
A. Statement (1) ALONE is sufficient, but statement (2) alone is not sufficient.
B. Statement (2) ALONE is sufficient, but statement (1) alone is not sufficient.
C. BOTH statements TOGETHER are sufficient, but NEITHER statement ALONE is sufficient.
D. EACH statement ALONE is sufficient.
E. Statements (1) and (2) TOGETHER are NOT sufficient.

B E R A N I B E R J U A N G | 65
Questions 70 and 71 refer to the following information.

Number of Contestants by Score and Day


5 Out 4 Out 3 Out 2 Out 1 Out 0 Out
Total
of 5 of 4 of 3 of 2 of 1 of 0
Day 1 2 3 4 6 2 3 20
Day 2 2 3 5 5 4 1 20
Day 3 3 3 4 5 3 2 20
Total 7 9 13 16 9 6 60
The same 20 contestants, on each of 3 days, answered 5 questions in order to win a prize. Each
contestant received 1 point for each correct answer. The number of contestants receiving a given
score on each day is shown in the table above.

70. What was the mean score of the contestants on Day 1?

71. No contestant received the same score on two different days. If a contestant is selected at
random, what is the probability that the selected contestant received a score of 5 on Day 2 or
Day 3, given that the contestant received a score of 5 on one of the three days?

72. If 2𝑥 + 8 = 16, what is the value of 𝑥 + 4 ?

̅̅̅̅̅ and 𝑁𝑅
73. In the figure below, 𝑀𝑄 ̅̅̅̅ at point 𝑃, 𝑁𝑃 = 𝑄𝑃, and 𝑀𝑃 = 𝑃𝑅. What is the measure,
in degrees, of ∠𝑄𝑀𝑅 ? (Disregard the degree symbol when gridding your answer.)

74. The number of radians in a 720-degree angle can be written as 𝑎𝜋, where a is a constant. What
is the value of 𝑎?

75. The graph of a line in the 𝑥𝑦-plane passes through the point (1, 4) and crosses the 𝑥- axis at
the point (2, 0). The line crosses the 𝑦 axis at the point (0, 𝑏). What is the value of 𝑏?

66 | I U P I T B
SET-3

Questions 1-10 are based on the following passage.

This passage is adapted from Patricia Waldron. “Why bids Fly in a V Formation.” C2014 by American
Association for the Advancement of Science.

Line Anyone watching the autumn sky knows that migrating birds fly in a V formation,
but scientists have long debated why. A new study of ibises finds taw that these big-
winged birds carefully position their s wingtips and sync their flapping, presumably to
catch the preceding bird's updraft—and save energy during flight.
5 There are two reasons birds might fly in a V formation: It may make flight easier,
or they're to simply following the leader. Squadrons of planes can save fuel by flying in
a V formation, and many scientists suspect that migrating birds do the same. Models
that treated flapping birds like fixed-wing airplanes estimate that they save energy by
drafting is off each other, but currents created by airplanes are far more stable than
10 the oscillating eddies coming off of a bird. "Air gets pretty unpredictable behind a
flapping wing." says James Usherwood, a locomotor biomechanist at the Royal
Veterinary College at the University of London In Hatfield, where the research took
place.
The study, published in Nature, took advantage of an existing project to
15 reintroduce endangered northern bald ibises (Geronticus eremita) to Europe. Scientists
used a microlight plane to show hand-raised birds their ancestral migration route from
Austria to Italy. A flock of 14 juveniles carried data loggers specially built by Usherwood
and his lab. The device's GPS determined each bird's flight 30 position to within 30 cm,
and an accelerometer showed the tinting of the wing flaps.
20 Just as aerodynamic estimates 'would predict, the birds positioned themselves to
fly just behind and to the side of the bird in front, timing their wing beats n to catch the
uplifting eddies. When a bird flew directly behind another, the timing of the flapping
reversed so that it could minimize the effects of the downdraft coming off the back of
the bird's body. “We didn't think this was possible." Usherwood says, considering that
25 the feat requires careful flight and incredible awareness of one's neighbors. “Perhaps
these big V formation birds can be thought of quite like an airplane with wings that go
up and down.”

B E R A N I B E R J U A N G | 67
The findings likely apply to other long-winged birds, such as pelicans, storks, and
geese, Usherwood says. Smaller birds create more complex wakes that would make
30 drafting too difficult. The researchers did not attempt to calculate the bird's energy
savings because the necessary physiological measurements would be too invasive for
an endangered specks. Previous studies estimate that birds can use 20 percent to 30
percent less energy while flying in a V.
"From a behavioral perspective it's really a breakthrough,” says David Lentink. a
35 mechanical engineer at Stanford University in Pak Alto, California, who was not
involved in the work. "Showing that birds care about syncing their wing as beats is
definitely an important insight that we didn't have before."
Scientists do not know how the birds find that aerodynamic sweet spot, but they
suspect that the animals align themselves either by sight or by sensing air currents
40 through their feathers. Alternatively, they may move around until they find the location
with the least resistance. In future studies, the researchers will switch to more common
birds, such as pigeons or geese. They plan to 70 investigate how the animals decide
who sets the course and the pace, and whether a mistake made by the leader can ripple
through the rest of the flock to cause traffic Jams.
45 "It's a pretty impressive piece of work as it is, but it does suggest that there's a lot
more to learn." says Ty Hedrick, a biologist at the University of North Carolina, Chapel
Hill, who studies flight aerodynamics in birds and insects. However they do it, he says,
”birds are awfully good hang-glider as pilots."

1. The main purpose of the passage is to C. The ibises have a body design that is
A. describe how squadrons of planes can similar to that of a modern airplane.
save fuel by flying in a V formation. D. The ibises were easily accessible for
B. discuss the effects of downdrafts on Usherwood and his team to track and
birds and airplanes. observe.
C. explain research conducted to study
why some birds fly in a V formation. 4. Which choice provides the best evidence
D. illustrate how birds sense air currents for the answer to the previous question?
through their feathers. A. Lines 2-4 (“A new . . . flight”)
B. Lines 6-7 (“Squadrons . . . same”)
2. The author includes the quotation “Air gets C. Lines 14-15 (“The study . . . Europe”)
pretty unpredictable behind a flapping D. Lines 18-19 (“The device’s . . . flaps”)
wing” (lines 10-11) to
A. explain that the current created by a 5. What is the most likely reason the author
bird differs from that of an airplane. includes the 30 cm measurement in line
B. stress the amount of control exerted 19?
by birds flying in a V formation. A. To demonstrate the accuracy with
C. indicate that wind movement is which the data loggers collected the
continuously changing. data
D. emphasize that the flapping of a bird’s B. To present recorded data about how
wings is powerful. far an ibis flies between successive
wing flaps
3. What can reasonably be inferred about the C. To provide the wingspan length of a
reason Usherwood used northern bald juvenile ibis
ibises as the subjects of his study? D. To show how far behind the microlight
A. The ibises were well acquainted with plane each ibis flew
their migration route.
B. Usherwood knew the ibises were
familiar with carrying data loggers
during migration.

68 | I U P I T B
6. What does the author imply about C. Future research will help scientists to
pelicans, storks, and geese flying in a V better understand V formations.
formation? D. Long-winged birds watch the lead bird
A. They communicate with each other in closely to keep a V formation intact.
the same way as do ibises.
B. They have the same migration routes 9. The author uses the phrase “aerodynamic
as those of ibises. sweet spot” in line 38 most likely to
C. They create a similar wake to that of A. describe how the proper structural
ibises. design of an airplane helps to save
D. They expend more energy than do fuel.
ibises. B. show that flying can be an exhilarating
experience.
7. Which choice provides the best evidence C. describe the birds’ synchronized wing
for the answer to the previous question? movement.
A. Lines 22-24 (“When . . . body”) D. suggest that a certain position in a V
B. Lines 29-30 (“Smaller . . . difficult”) formation has the least amount of
C. Lines 32-33 (“Previous . . . a V”) wind resistance.
D. Lines 40-41 (“Alternatively . . .
resistance”) 10. As used in line 44, “ripple” most nearly
means
8. What is a main idea of the seventh A. fluctuate.
paragraph (lines 38-44)? B. spread.
A. Different types of hierarchies exist in C. wave.
each flock of birds. D. undulate.
B. Mistakes can happen when long-
winged birds create a V formation.

A recent trend in professional development that has provided advantages to both employers
and employees is online instruction. From an employer perspective, the first and perhaps most
obvious advantage is the lower cost of online professional development compared with that of in-
person workshops and training. Employers can also (11) identify, which employees have
successfully completed instructional modules and which need to be offered additional training. For
employees, online professional development provides the opportunity to receive instruction at
their own pace and interact with other professionals online. This exciting trend has the potential to
make the shared responsibility of professional development less burdensome for both employers
and employees.

11.
A. NO CHANGE
B. identify:
C. identify
D. identify

Questions 12-15 are based on the following passage.


The Evolution of Slow Food
In 1986, McDonald’s caused a stir in Italy when it opened a restaurant next to Rome’s historic
Spanish Steps. Young, on-the-go eaters were thrilled; (12) specifically, those who prized regional
foods and Italy’s convivial culture built on cooking and long meals feared that the restaurant
signaled the death of a way of life. To counter the rise of fast food and fast (13) life, a cohort of
chefs, journalists, and sociologists spearheaded a Slow Food movement, declaring loyalty to
unhurried enjoyment. (14)

B E R A N I B E R J U A N G | 69
From its beginning, the movement (15) had opposed the standardization of taste that fast food
chains promote. For example, a McDonald’s hamburger made in Boston tastes more or less the
same as one made in Beijing. This consistency is made possible by industrial mass production. Slow
Food supporters, by contrast, back methods of growing and preparing food based on regional
culinary traditions. When produced using traditional methods, goat cheese made in France tastes
different from goat cheese made in Vermont.

12. A. Yes, because it explains the primary


A. NO CHANGE belief that led to the development of
B. for example, the Slow Food movement.
C. however, B. Yes, because it reinforces a claim that
D. in fact, the writer makes earlier in the
paragraph.
13. C. No, because it blurs the paragraph’s
A. NO CHANGE focus by introducing a new idea that is
B. life; a not clearly explained.
C. life: a D. No, because it distracts from the
D. life. A paragraph’s emphasis on the Slow
Food movement’s origins and beliefs.
14. At this point, the writer is considering
adding the following sentence. 15.
The group’s philosophy was connected to A. NO CHANGE
the tale of the hare and the tortoise, in B. opposes
which the tortoise wins the race. C. will oppose
Should the writer make this addition here? D. has opposed

A goat ingests the vegetation particular to the meadow in which it grazes, which, along with
other environmental (16) factors such as altitude and weather shapes the cheese’s taste and
texture. If all foods were produced under the industrial model, (17) we would have meals that are
not very flavorful.
During (18) their early years, the movement also focused on the value of (19) spending lots of
time with friends and family during long meals. It emphasized the importance of preserving these
“easygoing, slow pleasures.”

16. 18.
A. NO CHANGE A. NO CHANGE
B. factors, such as altitude and weather, B. there
C. factors such as, altitude and weather, C. Its
D. factors, such as altitude and weather D. it’s

17. Which choice most effectively supports the 19.


central point of the paragraph? A. NO CHANGE
A. NO CHANGE B. leisurely meals with friends and family.
B. the public would not be interested in C. eating slowly and in the company of
learning about traditional foods. loved ones such as friends and family.
C. people would not be able to determine D. joining friends as well as family for
how a particular food was made. time-consuming meals.
D. consumers would lose this diversity of
flavors.

70 | I U P I T B
As the movement grew beyond Italy’s borders—today Slow Food International boasts more
than 100,000 members in 150 countries—this emphasis on pleasure (20) pictured criticism for
being elitist. Critics have also asked if growing food using traditional methods, as opposed to mass
production, (21) can adequately and affordably feed the world? Given the hectic pace of modern
life, who among us has the time and resources for elaborate meals? Such questions, in addition to
environmental concerns, are at the heart of perennial debates about food production.
Over time, Slow Food has broadened its mission to focus on food that is good, clean, and fair
for all. Members assert that food should be flavorful, carrying the properties of a particular region;
it should be raised using environmentally sustainable practices that preserve biodiversity; and it
should be accessible to all without exploiting the labors of those who produced it. In short, Slow
Food runs programs that support small-scale producers in marketing regional foods in a world
where food corporations threaten to drive them out of the marketplace and homogenize food
choices.

20. 21.
A. NO CHANGE A. NO CHANGE
B. portrayed B. adequately and affordably can feed
C. drew the world?
D. sketched C. can adequately and affordably feed
the world.
D. adequately and affordably can feed
the world.

Questions 22-25 are based on the following passage.

People talk of ---(22)--- to him in statues of bronze or marble or pillars and thus they ---(23)--- him
and ---(24)--- his message. What tributes shall we --- (25)--- to him that he would have appreciated?

22. In each of the following passages, there are 24. In each of the following passages, there are
blanks each of which has been numbered. blanks each of which has been numbered.
Find out the appropriate words. Find out the appropriate words.
A. memorials A. decry
B. praises B. berate
C. epitaphs C. refute
D. tributes D. belie

23. In each of the following passages, there are 25. In each of the following passages, there are
blanks each of which has been numbered. blanks each of which has been numbered.
Find out the appropriate words. Find out the appropriate words.
A. mock A. pay
B. overthrow B. give
C. depreciate C. offer
D. ridicule D. extend

B E R A N I B E R J U A N G | 71
26. BIRDS are to FLY as HORSE is to: C. heart
A. speed D. emotional
B. grass E. emotive
C. legs
D. paddock 31. The fewer restrictions there are on the
E. trot advertising of legal services, the more
lawyers there are who advertise their
27. Find the two statements that together services, and the lawyers who advertise a
prove: specific service usually charge less for that
The football player wearing a blue jersey service than lawyers who do not advertise.
was in the team that won the Grand Final. Therefore, if the state removes any of its
2. The football player on the losing side current restrictions, such as the one against
wore purple. advertisements that do not specify fee
3. John is a football player and he wears arrangements, overall consumer legal costs
a red jersey. will be lower than if the state retains its
4. John and his team were runners up in current restrictions.
the Grand Final. If the statements in the passage are true,
5. A football player in a blue jersey beat which of the following must be true?
John’s team in the Grand Final. A. Some lawyers who now advertise will
6. John’s shorts are also white. charge more for specific services if
A. 1 & 3 they do not have to specify fee
B. 3 & 4 arrangements in the advertisements.
C. 2 & 4 B. More consumers will use legal services
D. 3 & 5 if there are fewer restrictions on the
E. 1 & 5 advertising of legal service.
C. If the restriction against
28. Assume that all mils are hils, some hils are advertisements that do not specify fee
jils, and some tils are rils. arrangements is removed, more
Therefore it makes sense that: lawyers will advertise their services.
A. all mils are jils D. If more lawyers advertise lower prices
B. all hils are mils for specific services, some lawyers
C. all tils are hils who do not advertise will also charge
D. some jils are hils less than they currently charge for
E. some mils are hils those services.
E. If the only restrictions on the
29. Assume that some huys are knis, all jiks are advertising of legal services were
suds, and some aws are huys. those that apply to every type of
Therefore it makes sense that: advertising, most lawyers would
A. some knis may also be aws advertise their services.
B. all huys are knis
C. all jiks are aws 32. The fewer restrictions there are on the
D. some suds are knis advertising of legal services, the more
E. all jiks are huys lawyers there are who advertise their
services, and the lawyers who advertise a
30. MOTION is to RUN as EMOTION is to: specific service usually charge less for that
A. furious service than lawyers who do not advertise.
B. feeling Therefore, if the state removes any of its
72 | I U P I T B
current restrictions, such as the one against C. The machine-tool industry
advertisements that do not specify fee encountered difficulty in obtaining
arrangements, overall consumer legal costs governmental protection against
will be lower than if the state retains its imports on grounds other than
current restrictions. defense.
Which of the following, if true, would most D. A few weapons important for defense
seriously weaken the argument concerning consist of parts that do not require
overall consumer legal costs? extensive machining.
A. The state has recently removed some E. Several federal government programs
other restrictions that had limited the have been designed which will enable
advertising of legal services. domestic machine-tool manufacturing
B. The state is unlikely to remove all of firms to compete successfully with
the restrictions that apply solely to the foreign toolmakers.
advertising of legal services.
C. Lawyers who do not advertise 34. Opponents of laws that require automobile
generally provide legal services of the drivers and passengers to wear seat belts
same quality as those provided by argue that in a free society people have the
lawyers who do advertise. right to take risks as long as the people do
D. Most lawyers who now specify fee not harm other as a result of taking the
arrangements in their advertisements risks. As a result, they conclude that it
would continue to do so even if the should be each person's decision whether
specification were not required. or not to wear a seat belt.
E. Most lawyers who advertise specific Which of the following, if true, most
services do not lower their fees for seriously weakens the conclusion drawn
those services when they begin to above?
advertise. A. Many new cars are built with seat belts
that automatically fasten when
33. Defense Department analysts worry that someone sits in the front seat.
the ability of the United States to wage a B. Automobile insurance rates for all
prolonged war would be seriously automobile owners are higher
endangered if the machine-tool because of the need to pay for the
manufacturing base shrinks further. Before increased injuries or deaths of people
the Defense Department publicly not wearing seat belts.
connected this security issue with the C. Passengers in airplanes are required to
import quota issue, however, the machine- wear seat belts during takeoffs and
tool industry raised the national security landings.
issue in its petition for import quotas. D. The rate of automobile fatalities in
Which of the following, if true, contributes states that do not have mandatory
most to an explanation of the machine-tool seat belt laws is greater than the rate
industry's raising the issue above regarding of fatalities in states that do have such
national security? laws.
A. When the aircraft industries retooled, E. In automobile accidents, a greater
they provided a large amount of work number of passengers who do not
for too builders. wear seat belts are injured than are
B. The Defense Department is only passengers who do wear seat belts.
marginally concerned with the effects
of foreign competition on the
machine-tool industry.

B E R A N I B E R J U A N G | 73
35. The cost of producing radios in Country Q C. the tariff on a radio imported from
is ten percent less than the cost of Country Q to Country Y is less than ten
producing radios in Country Y. even after percent of the cost of manufacturing
transportation fees and tariff charges are the radio in Country Y.
added, it is still cheaper for a company to D. the fee for transporting a radio from
import radios from Country Q to Country Y Country Q to Country Y is more than
than to produce radios in Country Y. ten percent of the cost of
The statements above, if true, best support manufacturing the radio in Country Q.
which of the following assertions? E. it takes ten percent less time to
A. labor costs in Country Q are ten manufacture a radios in Country Q
percent below those in Country Y. than it does in Country Y.
B. importing radios from Country Q to
Country Y will eliminate ten percent of
the manufacturing jobs in Country Y.

2
36. Two types of tickets were sold for a concert 38. 𝑥 + 1 =
𝑥+1
held at an amphitheater. Tickets to sit on a In the equation above, which of the
bench during the concert cost $75 each, following is a possible value of + 1 ?
and tickets to sit on the lawn during the
A. 1 − √2
concert cost $40 each. Organizers of the
B. √2
concert announced that 350 tickets had
C. 2
been sold and that $19,250 had been
D. 4
raised through ticket sales alone. Which of
the following systems of equations could 39. Roberto is an insurance agent who sells
be used to find the number of tickets for two types of policies: a $50,000 policy and
bench seats, B, and the number of tickets a $100,000 policy. Last month, his goal was
for lawn seats, L, that were sold for the to sell at least 57 insurance policies. While
concert? he did not meet his goal, the total value of
A. (75𝐵)(40𝐿) = 1,950
the policies he sold was over $3,000,000.
𝐵 + 𝐿 = 350
Which of the following systems of
B. 40𝐵 + 75𝐿 = 19,250
inequalities describes 𝑥, the possible
𝐵 + 𝐿 = 350
number of $50,000 policies, and y, the
C. 75𝐵 + 40𝐿 = 350
possible number of $100,000 policies, that
𝐵 + 𝐿 = 19,250
Roberto sold last month?
D. 75𝐵 + 40𝐿 = 19,250
𝐵 + 𝐿 = 350 A. 𝑥 + 𝑦 < 57
50,000𝑥 + 100,000𝑦 < 3,000,000
B. 𝑥 + 𝑦 > 57
37. In the 𝑥𝑦-plane, the graph of which of the
50,000𝑥 + 100,000𝑦 > 3,000,000
following equations is a line with a slope of
C. 𝑥 + 𝑦 < 57
3?
1 50,000𝑥 + 100,000𝑦 > 3,000,000
A. 𝑦 = 𝑥 D. 𝑥 + 𝑦 > 57
3
B. 𝑦 = 𝑥 − 3 50,000𝑥 + 100,000𝑦 < 3,000,000
C. 𝑦 = 3𝑥 + 2
D. 𝑦 = 6𝑥 + 3

74 | I U P I T B
Questions 40-42 refer to the following
information.

D.

42. Jenny has a pitcher that contains 1 gallon


of water. How many times could Jenny
completely fill the glass with 1 gallon of
water? (1 gallon = 128 fluid ounces)
The glass pictured above can hold a maximum A. 16
volume of 473 cubic centimeters, which is B. 8
approximately 16 fluid ounces. C. 4
D. 3
40. What is the value of 𝑘, in centimeters ?
A. 2.52 43. A granite block in the shape of a right
B. 7.67 rectangular prism has dimensions 30
C. 7.79 centimeters by 40 centimeters by 50
D. 10.11 centimeters. The block has a density of 2.8
grams per cubic centimeter. What is the
41. Water pours into the glass slowly and at a mass of the block, in grams? (Density is
constant rate. Which of the following mass per unit volume.)
graphs best illustrates the height of the A. 336
water level in the glass as it fills? B. 3,360
C. 16,800
D. 168,000

44. The table shows the results of a research


study that investigated the therapeutic
value of vitamin C in preventing colds. A
random sample of 300 adults received
A.
either a vitamin C pill or a sugar pill each
day during a 2-week period, and the adults
reported whether they contracted a cold
during that time period. What proportion
of adults who received a sugar pill reported
contracting a cold?
B. Cold No Cold Total
Vitamin 21 129 150
C
Sugar Pill 33 117 150
Total 54 246 300
11
A.
18
11
B.
50
C. 9
C.
50
11
D.
100

B E R A N I B E R J U A N G | 75
45. Ages of 20 Students Enrolled in a College 47. The range of the polynomial function 𝑓 is
Class the set of real numbers less than or equal
Age Frequency to 4. If the zeros of 𝑓 are −3 and 1, which
18 6 of the following could be the graph of 𝑦 =
19 5 𝑓(𝑥) in the 𝑥𝑦-plane?
20 4
21 2
22 1
23 1
30 1
The table above shows the distribution of
ages of the 20 students enrolled in a A.
college class. Which of the following gives
the correct order of the mean, median, and
mode of the ages?
A. mode < median < mean
B. mode < mean < median
C. median < mode < mean
D. mean < mode < median B.

46. The figure below shows the relationship


between the percent of leaf litter mass
remaining after decomposing for 3 years
and the mean annual temperature, in
degrees Celsius (°C), in 18 forests in C.
Canada. A line of best fit is also shown.

D.
1
48. If 𝑎 −2 = 𝑥, where 𝑎 > 0, what is a in
terms of 𝑥 ?
A particular forest in Canada, whose data is A. √𝑥
not included in the figure, had a mean B. −√𝑥
1
annual temperature of −2°C. Based on the C.
𝑥2
line of best fit, which of the following is 1
D. –
closest to the predicted percent of leaf 𝑥2

litter mass remaining in this particular


49. Which of the following is a value of 𝑥 for
forest after decomposing for 3 years? −3
A. 50% which the expression is
𝑥 2 +3𝑥−10
B. 63% undefined?
C. 70% A. −3
D. 82% B. −2
C. 0
D. 2

76 | I U P I T B
50. The average annual energy cost for a D. The average annual increase, in
certain home is $4,334. The homeowner billions of pounds, of plastic produced
plans to spend $25,000 to install a per year in the United States from
geothermal heating system. The 1985 to 2003
homeowner estimates that the average
annual energy cost will then be $2,712. 52. Which of the following is closest to the
Which of the following inequalities can be percent increase in the billions of pounds
solved to find t, the number of years after of plastic produced in the United States
installation at which the total amount of from 2000 to 2003?
energy cost savings will exceed the A. 10%
installation cost? B. 44%
A. 25,000 > (4,334 − 2,712)𝑡 C. 77%
B. 25,000 < (4,334 − 2,712)𝑡 D. 110%
C. 25,000 − 4,334 > 2,712𝑡
4,332
D. 25,000 > 𝑡 53. 𝑀 = 1,800(1.02)𝑡
2,712
The equation above models the number of
Questions 51 and 52 refer to the following members, 𝑀, of a gym 𝑡 years after the
information. gym opens. Of the following, which
Between 1985 and 2003, data were collected equation models the number of members
every three years on the amount of plastic of the gym 𝑞 quarter years after the gym
produced annually in the United States, in opens?
𝑞
billions of pounds. The graph below shows the A. 𝑀 = 1,800(1.02)4
data and a line of best fit. The equation of the B. 𝑀 = 1,800(1.02)4𝑞
line of best fit is 𝑦 = 3.39𝑥 + 46.89, where x C. 𝑀 = 1,800(1.005)4𝑞
is the number of years since 1985 and 𝑦 is the D. 𝑀 = 1,800(1.082)𝑞
amount of plastic produced annually, in billions
of pounds. 54. To determine the mean number of children
per household in a community, Tabitha
surveyed 20 families at a playground. For
the 20 families surveyed, the mean number
of children per household was 2.4. Which
of the following statements must be true?
A. The mean number of children per
household in the community is 2.4.
B. A determination about the mean
number of children per household in
51. Which of the following is the best the community should not be made
interpretation of the number 3.39 in the because the sample size is too small.
context of the problem? C. The sampling method is flawed and
A. The amount of plastic, in billions of may produce a biased estimate of the
pounds, produced in the United States mean number of children per
during the year 1985 household in the community.
B. The number of years it took the United D. The sampling method is not flawed
States to produce 1 billion pounds of and is likely to produce an unbiased
plastic estimate of the mean number of
C. The average annual plastic production, children per household in the
in billions of pounds, in the United community.
States from 1985 to 2003

B E R A N I B E R J U A N G | 77
55. For the finale of a TV show, viewers could 58. The 22 students in a health class conducted
use either social media or a text message an experiment in which they each recorded
to vote for their favorite of two their pulse rates, in beats per minute,
contestants. The contestant receiving before and after completing a light exercise
more than 50% of the vote won. An routine. The dot plots below display the
estimated 10% of the viewers voted, and results.
30% of the votes were cast on social media.
Contestant 2 earned 70% of the votes cast
using social media and 40% of the votes
cast using a text message. Based on this
information, which of the following is an
accurate conclusion?
A. If all viewers had voted, Contestant 2
would have won.
B. Viewers voting by social media were
likely to be younger than viewers
voting by text message. Let 𝑠1 and 𝑟1 be the standard deviation
C. If all viewers who voted had voted by and range, respectively, of the data before
social media instead of by text exercise, and let 𝑠2 and 𝑟2 be the standard
message, Contestant 2 would have deviation and range, respectively, of the
won. data after exercise. Which of the following
D. Viewers voting by social media were is true?
more likely to prefer Contestant 2 than A. 𝑠1 = 𝑠2 𝑎𝑛𝑑 𝑟1 = 𝑟2
were viewers voting by text message. B. 𝑠1 < 𝑠2 𝑎𝑛𝑑 𝑟1 < 𝑟2
C. 𝑠1 > 𝑠2 𝑎𝑛𝑑 𝑟1 > 𝑟2
56. Population of Greenleaf, Idaho D. 𝑠1 ≠ 𝑠2 𝑎𝑛𝑑 𝑟1 = 𝑟2
Year Population
2000 862 59. The complete graph of the function 𝑓 and
2010 846 a table of values for the function 𝑔 are
The table above shows the population of shown below. The maximum value of 𝑓 is
Greenleaf, Idaho, for the years 2000 and 𝑘. What is the value of (𝑘) ?
2010. If the relationship between 𝑥 𝑔(𝑥)
population and year is linear, which of the -2 1
following functions 𝑃 models the -1 2
population of Greenleaf 𝑡 years after 2000? 0 3
A. 𝑃(𝑡) = 862 − 1.6𝑡 1 4
B. 𝑃(𝑡) = 862 − 16𝑡 2 5
C. 𝑃(𝑡) = 862 + 16(𝑡 – 2,000) 3 6
D. 𝑃(𝑡) = 862 – 1,6(𝑡 – 2,000) 4 7

57. In the 𝑥𝑦-plane, the point (𝑝, 𝑟) lies on the


line with equation 𝑦 = 𝑥 + 𝑏, where 𝑏 is
a constant. The point with coordinates
(2𝑝, 5𝑟) lies on the line with equation 𝑦 =
𝑟
2𝑥 + 𝑏. If 𝑝 ≠ 0, what is the value of ?
𝑝
A. 25 A. 7
B. 34 B. 6
C. 43 C. 3
D. 52 D. 0
78 | I U P I T B
60. A photocopy machine is initially loaded minutes after the machine started
with 5,000 sheets of paper. The machine printing?
starts a large job and copies at a constant A. 𝑝 = 5,000 − 20𝑚
rate. After 20 minutes, it has used 30% of B. 𝑝 = 5,000 − 75𝑚
𝑚
the paper. Which of the following C. 𝑝 = 5,000(0.3)20
equations models the number of sheets of 𝑚
D. 𝑝 = 5,000(0.7)20
paper, 𝑝, remaining in the machine 𝑚

61. 𝑸𝒖𝒂𝒏𝒕𝒊𝒕𝒚 𝑨 𝑸𝒖𝒂𝒏𝒕𝒊𝒕𝒚 𝑩


𝑇ℎ𝑒 𝑎𝑟𝑒𝑎 𝑜𝑓 𝑎 𝑠𝑒𝑚𝑖𝑐𝑖𝑟𝑐𝑙𝑒 𝑇ℎ𝑒 𝑎𝑟𝑒𝑎 𝑜𝑓 𝑎 𝑟𝑒𝑐𝑡𝑎𝑛𝑔𝑙𝑒
𝑤𝑖𝑡ℎ 𝑟𝑎𝑑𝑖𝑢𝑠 𝑜𝑓 6 𝑤𝑖𝑡ℎ 𝑎 𝑙𝑒𝑛𝑔𝑡ℎ 𝑜𝑓 9 𝑎𝑛𝑑 𝑎 𝑤𝑖𝑑𝑡ℎ 𝑜𝑓 6
A. Quantity A is greater.
B. Quantity B is greater.
C. The two quantities are equal.
D. The relationship cannot be determined from the information given.

62. 𝟑𝒂 + 𝟐𝒃
𝒂&&𝒃 =
𝒃
𝒙 = 𝟑 𝒂𝒏𝒅 𝒚 = 𝟒
𝑸𝒖𝒂𝒏𝒕𝒊𝒕𝒚 𝑨 𝑸𝒖𝒂𝒏𝒕𝒊𝒕𝒚 𝑩

𝑦&&𝑥 − 𝑥&&𝑦 2
A. Quantity A is greater.
B. Quantity B is greater.
C. The two quantities are equal.
D. The relationship cannot be determined from the information given.

63. 𝟐𝟏𝟒𝒛 > (𝟐𝟏𝟐𝒛 )(𝟖𝟐 )


𝑸𝒖𝒂𝒏𝒕𝒊𝒕𝒚 𝑨 𝑸𝒖𝒂𝒏𝒕𝒊𝒕𝒚 𝑩

𝑧 3
A. Quantity A is greater.
B. Quantity B is greater.
C. The two quantities are equal.
D. The relationship cannot be determined from the information given.

64. Set S consists of the even integers from 2 through 12, inclusive.

𝑸𝒖𝒂𝒏𝒕𝒊𝒕𝒚 𝑨 𝑸𝒖𝒂𝒏𝒕𝒊𝒕𝒚 𝑩

𝑇ℎ𝑒 𝑝𝑟𝑜𝑏𝑎𝑏𝑖𝑙𝑖𝑡𝑦 𝑡ℎ𝑎𝑡 𝑡𝑤𝑜 𝑑𝑖𝑠𝑡𝑖𝑛𝑐𝑡 𝑖𝑛𝑡𝑒𝑔𝑒𝑟𝑠 1


𝑠𝑒𝑙𝑒𝑐𝑡𝑒𝑑 𝑎𝑡 𝑟𝑎𝑛𝑑𝑜𝑚 𝑓𝑟𝑜𝑚 𝑆 𝑤𝑖𝑙𝑙 ℎ𝑎𝑣𝑒 𝑎 6
𝑑𝑖𝑓𝑓𝑒𝑟𝑒𝑛𝑐𝑒 𝑜𝑓 6 𝑜𝑟 𝑔𝑟𝑒𝑎𝑡𝑒𝑟
A. A is greater.
B. Quantity B is greater.
C. The two quantities are equal.
D. The relationship cannot be determined from the information given.

B E R A N I B E R J U A N G | 79
65. (𝟓, 𝟐), (𝒎, 𝟒), (𝟑, 𝟓), (𝒏, 𝟕) 𝐚𝐫𝐞 𝐩𝐨𝐢𝐧𝐭𝐬 𝐨𝐧 𝐭𝐡𝐞 𝐬𝐚𝐦𝐞 𝐥𝐢𝐧𝐞.

𝑸𝒖𝒂𝒏𝒕𝒊𝒕𝒚 𝑨 𝑸𝒖𝒂𝒏𝒕𝒊𝒕𝒚 𝑩

𝑚 𝑛
A. Quantity A is greater.
B. Quantity B is greater.
C. The two quantities are equal.
D. The relationship cannot be determined from the information given.

66. Data Sufficiency: Is 𝑥𝑦 < 0?


(1) 5|𝑥| + |𝑦| = 0
(2) |𝑥| + 5|𝑦| = 0
A. Statement (1) ALONE is sufficient, but statement (2) alone is not sufficient.
B. Statement (2) ALONE is sufficient, but statement (1) alone is not sufficient.
C. BOTH statements TOGETHER are sufficient, but NEITHER statement ALONE is sufficient.
D. EACH statement ALONE is sufficient.
E. Statements (1) and (2) TOGETHER are NOT sufficient.

67. Data Sufficiency: When a positive integer ′𝑥′ is divided by a divisor ′𝑑′, the remainder is 24.
What is 𝑑?
(1) When 2𝑥 is divided by 𝑑, the remainder is 23.
(2) When 3𝑥 is divided by 𝑑, the remainder is 22.
A. Statement (1) ALONE is sufficient, but statement (2) alone is not sufficient.
B. Statement (2) ALONE is sufficient, but statement (1) alone is not sufficient.
C. BOTH statements TOGETHER are sufficient, but NEITHER statement ALONE is sufficient.
D. EACH statement ALONE is sufficient.
E. Statements (1) and (2) TOGETHER are NOT sufficient.

68. Data Sufficiency: How many of the numbers 𝑥, 𝑦, and 𝑧 are positive if each of these numbers
is less than 10?
(1) 𝑥 + 𝑦 + 𝑧 = 20
(2) 𝑥 + 𝑦 = 14
A. Statement (1) ALONE is sufficient, but statement (2) alone is not sufficient.
B. Statement (2) ALONE is sufficient, but statement (1) alone is not sufficient.
C. BOTH statements TOGETHER are sufficient, but NEITHER statement ALONE is sufficient.
D. EACH statement ALONE is sufficient.
E. Statements (1) and (2) TOGETHER are NOT sufficient.

69. Data Sufficiency: 𝐴 set 𝑆 contains the following elements: {7, 11, 15, 19, 23, 𝑥}. What is the
value of 𝑥?
(1) The elements are in arithmetic progression
(2) 𝑥 is prime
A. Statement (1) ALONE is sufficient, but statement (2) alone is not sufficient.
B. Statement (2) ALONE is sufficient, but statement (1) alone is not sufficient.
C. BOTH statements TOGETHER are sufficient, but NEITHER statement ALONE is sufficient.
D. EACH statement ALONE is sufficient.
E. Statements (1) and (2) TOGETHER are NOT sufficient.

80 | I U P I T B
70. (7532 + 10𝑦 2 ) + 10(10𝑦 2 − 110)The expression above can be written in the form 𝑎𝑦 2 +
𝑏, where 𝑎 and 𝑏 are constants. What is the value of 𝑎 + 𝑏?

71. There are two atoms of hydrogen and one atom of oxygen in one molecule of water. How many
atoms of hydrogen are there in 51 molecules of water?

1
72. 𝑥 − 𝑎 = 0
2
If 𝑥 = 1 in the equation above, what is the value of 𝑎?

73. In the 𝑥𝑦-plane, the equation 𝑥 + 2𝑦 = 10 and 3𝑥 + 6𝑦 = 𝑐 represent the same line for
some constant 𝑐. what is the value of 𝑐?

74. On April 18, 1775, Paul Revere set off on his midnight ride from Charlestown to Lexington. If
he had ridden straight to Lexington without stopping, he would have traveled 11 miles in 26
minutes. In such a ride, what would the average speed of his horse have been, to the nearest
tenth of a mile per hour?

1
75. The graph of the function 𝑓, defined by 𝑓(𝑥) = − (𝑥 − 4)2 + 10, is shown in the 𝑥𝑦- plane
2
below. If the function 𝑔 (not shown) is defined by 𝑔(𝑥) = −𝑥 + 10, what is one possible
value of a such that 𝑓(𝑎) = 𝑔(𝑎) ?

B E R A N I B E R J U A N G | 81
SET-4

Questions 1-10 are based on the following passages.

Passage 1 is adapted from Alexis de Tocqueville, Democracy in America, Volume 2. Originally


Published in 1840. Passage 2 is adapted from Harriet Taylor Mill, ”Enfranchisement of Women”
Originally Published in 1851.As United States and European Societies grew increasingly democratic
during the nineteenth century, debates arose about whether freedoms enjoyed by men should be
extended to women as well
Line Passage 1
I have shown how democracy destroy or modifies the different Inequalities which
originate in society, but Is this all? or does it not ultimately affect that great inequality of
man and woman which has seemed, up to the present day, to be eternally based
inhuman nature? 1 believe that the social changes %thick bring nearer to the same level
5 the father and son, the master and servant, and superiors and inferiors generally
speaking, will raise woman and nuke her more and more the equal of man. But here,
more than ever, l feel the necessity of making myself clearly understood for there is no
subject on which the coarse and lawless fancies of our age have taken a freer range.
There are people in Europe who, confounding together the different characteristics
10 of the sexes, would make of nun and woman beings not only equal but alike. They would
give to both the same functions, impose on both the ante duties, and grant to both the
same rights: they would mix them in all things—their occupations, their pleasures, their
business. It may readily be conceived, that by thus attempting to make one sex equal to
the other, both are degraded: and from so preposterous a medley of the works of nature
15 nothing could ever result but weak men and disorderly women.
It is not thus that the Americans understand that species of democratic equality
which may be established between the sexes. They admit, that as nature has appointed
such wide differences between the physical and moral constitution of nun and woman,
her manifest design was to give a distinct employment to their various faculties and they
20 hold that Improvement does not consist in making beings so dissimilar do pretty nearly
the same things, but in getting each of them to fulfill their respective tasks in the best
possible manner. The Americans have applied to the sexes the great principle of political
economy which governs the manufactures of our age, by carefully dividing the duties of
man from thou of woman. In order that the great work of society may be the better
25 earned on.
82 | I U P I T B
Passage 2
As society was constituted until the last few generations, inequality was its very
basis; association grounded on equal rights scarcely existed to be equals was to be
enemies; two persons could hardly cooperate in anything, or meet in any amicable
relation, without the law’s appointing that one of them should be the superior of the
30 other. Mankind have outgrown this state, and all things now tend to substitute, as the
general principle of human relations, a just equality instead of the dominion of the
strongest. But old relations, that between men and women, being the nearest and most
intimate, and connected with the greatest number of strong emotions, was sure to be
the last to throw off the old rule, and receive the new: for, in proportion to the strength
35 of a feeling is the tenacity with which it clings to the forms and circumstances with which
it has even accidentally become associated....
... The proper sphere for all human beings is the largest and highest which they are
able to attain to. What this is, cannot be ascertained without complete liberty of choice....
Let every occupation be open to all, without favor or discouragement to any, and
40 employments will fall into the hands of those men or women who are found by
experience to be most capable of worthily exercising them. There need be no fear that
women will take out of the hands of men any occupation which men perform better than
they. Each individual will prove his or her capacities, in the only way in which capacities
can be proved,—by trial: and the works will have the benefit of the best faculties of all
45 its inhabitants. But to interfere beforehand by an arbitrary limit, and declare that
whatever be the genius, talent, energy, or force of mind, of an individual of a certain sex
or class, those faculties shall not be exerted, or shall be mated only in some few of the
many modes In which others are permitted to use theirs. is not only an injustice to the
individual, and a detriment to society, which loses what it can ill spare, but is also the
50 moss effectual way of providing that, in the sex or class so fettered the qualities which
are not permitted to be exercised shall net exit

1. As used in line 6, “raise” most nearly means A. omnipotence.


A. increase. B. supremacy.
B. cultivate. C. ownership.
C. nurture. D. territory.
D. elevate.
5. In Passage 2, Mill most strongly suggests
2. In Passage 1, Tocqueville implies that that gender roles are resistant to change
treatment of men and women as identical because they
in nature would have which consequence? A. have long served as the basis for the
A. Neither sex would feel oppressed. formal organization of society.
B. Both sexes would be greatly harmed. B. are matters of deeply entrenched
C. Men would try to reclaim their lost tradition.
authority. C. can be influenced by legislative
D. Men and women would have reforms only indirectly.
privileges they do not need. D. benefit the groups and institutions
currently in power.
3. Which choice provides the best evidence
for the answer to the previous question? 6. Which choice provides the best evidence
A. Lines 9-10 (“There . . . alike”) for the answer to the previous question?
B. Lines 10-12 (“They . . . rights”) A. Lines 26-27 (“As society . . . basis”)
C. Lines 13-14 (“It may . . . degraded”) B. Lines 28-30 (“two . . . other”)
D. Lines 16-17 (“It is . . . sexes”) C. Lines 34-36 (“in proportion . . .
associated”)
4. As used in line 31, “dominion” most nearly D. Lines 40-41 (“employments . . . them”)
means
B E R A N I B E R J U A N G | 83
7. Both authors would most likely agree that B. Tocqueville believes that an
the changes in gender roles that they individual’s economic class should
describe would be determine that individual’s position,
A. part of a broad social shift toward while Mill believes that class is not a
greater equality. legitimate consideration.
B. unlikely to provide benefits that C. Tocqueville believes that an
outweigh their costs. individual’s temperament should
C. inevitable given the economic determine that individual’s position,
advantages of gender equality. while Mill believes that temperament
D. at odds with the principles of American should not be a factor in an individual’s
democracy. position.
D. Tocqueville believes that an
8. Tocqueville in Passage 1 would most likely individual’s position should be
characterize the position taken by Mill in determined by what is most beneficial
lines 39-41 in Passage 2 (“Let . . . them”) as to society, while Mill believes it should
A. less radical about gender roles than it be determined by what an individual
might initially seem. finds most rewarding.
B. persuasive in the abstract but difficult
to implement in practice. 10. Based on Passage 2, Mill would most likely
C. ill-advised but consistent with a view say that the application of the “great
held by some other advocates of principle of political economy” (lines 22-23,
gender equality. Passage 1) to gender roles has which
D. compatible with economic progress in effect?
the United States but not in Europe. A. It prevents many men and women
from developing to their full potential.
9. Which choice best describes the ways that B. It makes it difficult for men and
the two authors conceive of the women to sympathize with each
individual’s proper position in society? other.
A. Tocqueville believes that an C. It unintentionally furthers the cause of
individual’s position should be defined gender equality.
in important ways by that individual’s D. It guarantees that women take
sex, while Mill believes that an occupations that men are better
individual’s abilities should be the suited to perform.
determining factor.

Questions 11-19 are based on the following passage.

Was the Hoax a Hoax?


For an hour on the evening of October 30, 1938, Orson Welles and other performers from the
Mercury Theatre flooded the airwaves with alarming “news bulletins” about a Martian invasion
supposedly occurring in Grover’s Mill, New Jersey. They were performing a radio play adapted from
The War of the Worlds, a science fiction novel by H. G. Wells. The next day, a front-page (11)
headline in the New York Times declared, “Radio Listeners in Panic, Taking War Drama as Fact.”
(12) The Times article claimed that people had fled their homes and that police stations had been
swamped with calls. This version of events persisted, and the legend became that Welles’s
broadcast had as many as twelve million people (13) who feared that Martians had invaded Earth.
Recently, however, scholars have questioned the accuracy of this legend, suggesting the
degree of public hysteria has been grossly exaggerated. The authors of an article published in
October 2013 go (14) so far to assign blame for the distortion to the newspaper industry.

84 | I U P I T B
11. C. In 2013, many newspapers and
A. NO CHANGE magazines featured articles about the
B. headline in the New York Times, seventy‑fifth anniversary of the
declared broadcast.
C. headline, in the New York Times D. The Times was then and is now one of
declared, the United States’ most popular news
D. headline, in the New York Times, sources.
declared 13.
A. NO CHANGE
12. The writer wants to add a supporting detail B. that feared
to indicate that the story was widely C. fearing
reported. Which choice best accomplishes D. to fear
this goal? 14.
A. NO CHANGE A. NO CHANGE
B. Other newspapers also ran stories B. as far
claiming that the broadcast had incited C. as far and
mass hysteria. D. so far as

(15) At this time, Jefferson Pooley and Michael Socolow, both professors of communication
studies, argue that the newspaper industry sought to discredit the newly emerging technology of
radio, which was cutting into newspapers’ (16) profits. The newspaper industry tried to do this by
portraying the new medium as irresponsible.
[1] Proof of ulterior motives is scarce, (17) consequently weakening Pooley and Socolow’s
argument. [2] For instance, the C. E. Hooper ratings indicate that a mere 2 percent of households
had tuned in to the broadcast. [3] Pooley and Socolow also call into question the validity of an oft-
cited report that was based on a survey conducted six weeks after the broadcast. [4] Just because
some people found the broadcast unsettling, the authors contend, doesn’t mean they believed it
and reacted with real terror. [5] According to this report, one million people indicated that they
had been “frightened” by the broadcast. [6] Ratings, however, reveal that (18) far fewer than a
million people had been

15. 17. Which choice best establishes the main


A. NO CHANGE idea of the paragraph?
B. On one hand, A. NO CHANGE
C. In the article, B. but evidence does suggest that reports
D. Next, of panic have been overblown.
C. yet Pooley and Socolow maintain that
16. Which choice most effectively combines the newspaper industry intentionally
the sentences at the underlined portion? distorted the story.
A. profits, which is what the newspaper D. making it difficult to determine what
industry tried to do when it portrayed really happened in 1938.
B. profits, by which the newspaper
industry portrayed 18.
C. profits and tried to do this by A. NO CHANGE
portraying B. many less than
D. profits, by portraying C. much less then
D. much fewer then

B E R A N I B E R J U A N G | 85
listening to the broadcast. [7] Furthermore, Pooley and Socolow note that this survey “conflated
being ‘frightened,’ ‘disturbed,’ or ‘excited’ by the program with being ‘panicked.’” (19)
Pooley and Socolow describe a more likely scenario: most people who heard the broadcast
understood they were listening to a piece of fiction, but some being influenced by the
sensationalized news coverage afterward, later “remembered” being more afraid than they had
been. The researchers also suggest that, not unlike people who got caught up in the excitement of
the story when reading about it in the newspaper, the American public may have been willing to
embrace the legend because of its appeal to the imagination.

19. To make this paragraph most logical, sentence 4 should be placed


A. where it is now.
B. after sentence 2.
C. after sentence 5.
D. after sentence 7.

Questions 20-25 are based on the following passage.

He has shown us the way to live and the way to die and if we have not ---(20)--- that lessons, it
would be better that we ---(21)--- no memorial to him, for the only fit memorial is to follow ---(22)-
-- in the path he showed us. To him India was dear because she had ---(23)--- throughout the ---
(24)--- certain ---(25)--- truths.
20. In each of the following passages, there are 23. In each of the following passages, there are
blanks each of which has been numbered. blanks each of which has been numbered.
Find out the appropriate words. Find out the appropriate words.
A. followed A. followed
B. realised B. offered
C. accepted C. represented
D. understood D. adhered

21. In each of the following passages, there are 24. In each of the following passages, there are
blanks each of which has been numbered. blanks each of which has been numbered.
Find out the appropriate words. Find out the appropriate words.
A. raised A. country
B. erected B. world
C. constructed C. period
D. built D. ages

22. In each of the following passages, there are 25. In each of the following passages, there are
blanks each of which has been numbered. blanks each of which has been numbered.
Find out the appropriat e words. Find out the appropriate words.
A. reverently A. immutable
B. strictly B. immortal
C. willingly C. higher
D. readily D. durable

86 | I U P I T B
26. Phil is a personal trainer. He has clients 29. Which of the following is most similar in
booked in for Monday through Friday next meaning to obtain:
week. He is seeing Claire two days before A. busy
Mary. Sam is booked in two days before B. interrogate
Bec, who is one day after Mary and one day C. serious question
before Amanda. Finally, Pauline is D. acquittal
scheduled in four days after Claire. E. acquire
If Claire is coming in on Monday, who has
Phil double booked? 30. Assume that:
A. Claire & Bec All HIFS are PIFS
B. Bec & Sam Some SIFS are RIFS
C. Claire & Mary All ZIFS are SIFS
D. Mary & Sam All PIFS are ZIFS
E. Amanda & Pauline Which of the following do we know is
correct?
27. Five trucks travel the primary route every A. All ZIFS are RIFS
day. Truck A arrives back at the depot B. Some HIFS are ZIFS
before truck C. Truck E is after truck B who C. All RIFS are SIFS
is 2 after truck D and 3 after truck A. Truck D. All HIFS are RIFS
D arrives before truck C but after truck A. E. Some ZIFS are PIFS
Which truck is the last to arrive back at the
depot? 31. During the Second World War, about
A. truck A 375,000 civilians died in the United States
B. truck B and about 408,000 members of the United
C. truck C States armed forces died overseas. On the
D. truck D basis the those figures, it can be concluded
E. truck E that it was not much more dangerous to be
overseas in the armed forces during the
28. Find the two statements that together Second World War than it was to stay at
prove: home as a civilian.
Emily attends acrobatic classes. Which of the following would reveal most
2. Emily’s school teaches gymnastics and clearly the absurdity of the conclusion
acrobatics. drawn above?
3. Laura has acrobatic lessons with A. Counting deaths among members of
Jeremy. the armed forces who served in the
4. Emily’s friend Jeremy attends United State in addition to deaths
gymnastics and acrobatics classes. among members of the armed forces
5. Jeremy and Emily are in the same serving overseas
classes. B. Expressing the difference between the
6. Jeremy’s sister Laura attends numbers of deaths among civilians and
gymnastics classes with Emily. members of the armed forces as a
A. 1 & 2 percentage of the total number of
B. 2 & 3 deaths
C. 3 & 4 C. Separating deaths caused by accidents
D. 2 & 5 during service in the armed forces
E. 1 & 5 from deaths caused by combat injuries

B E R A N I B E R J U A N G | 87
D. Comparing death rates per thousand A. most individuals have no formal
members of each group rather than training in how to extinguish fires.
comparing total numbers of deaths B. Since new homes are only a tiny
E. Comparing deaths caused by accidents percentage of available housing in the
in the United States to deaths caused city, the new ordinance would be
by combat in the armed forces extremely narrow in scope.
C. The installation of smoke detectors in
32. Toughened hiring standards have not been new residences costs significantly less
the primary cause of the present staffing than the installation of sprinklers.
shortage in public schools. The shortage of D. In the city where the ordinance was
teachers is primarily caused by the fact that proposed, the average time required
in recent years teachers have not by the fire department to respond to a
experienced any improvements in working fire was less than the national average.
conditions and their salaries have not kept E. The largest proportion of property
pace with salaries in other professions. damage that results from residential
Which of the following, if true, would most fires is caused by fires that start when
support the claims above? no household member is present.
A. Many teachers already in the
profession would not have been hired 34. Even though most universities retain the
under the new hiring standards. royalties from faculty members'
B. Today more teachers are entering the inventions, the faculty members retain the
profession with a higher educational royalties from books and articles they
level than in the past. write. Therefore, faculty members should
C. Some teachers have cited higher retain the royalties from the educational
standards for hiring as a reason for the computer software they develop.
current staffing shortage. The conclusion above would be more
D. Many teachers have cited low pay and reasonably drawn if which of the following
lack of professional freedom as were inserted into the argument as an
reasons for their leaving the additional premise?
profession. A. Royalties from inventions are higher
E. Many prospective teachers have cited than royalties from educational
the new hiring standards as a reason software programs.
for not entering the profession. B. Faculty members are more likely to
produce educational software
33. A proposed ordinance requires the programs than inventions.
installation in new homes of sprinklers C. Inventions bring more prestige to
automatically triggered by the presence of universities that do books and articles.
a fire. However, a home builder argued D. In the experience of most universities,
that because more than ninety percent of educational software programs are
residential fires are extinguished by a more marketable that are books and
household member, residential sprinklers articles.
would only marginally decrease property E. In terms of the criteria used to award
damage caused by residential fires. royalties, educational software
Which of the following, if true, would most programs are more nearly comparable
seriously weaken the home builder's to books and articles than to
argument? inventions.

88 | I U P I T B
35. Increase in the level of high-density B. Individuals who do not exercise
lipoprotein (HDL) in the human regularly have a high risk of developing
bloodstream lower bloodstream- high levels of cholesterol in the
cholesterol levels by increasing the body's bloodstream late in life.
capacity to rid itself of excess cholesterol. C. Exercise and weight reduction are the
Levels of HDL in the bloodstream of some most effective methods of lowering
individuals are significantly increased by a bloodstream cholesterol levels in
program of regular exercise and weight humans.
reduction. D. A program of regular exercise and
Which of the following can be correctly weight reduction lowers cholesterol
inferred from the statements above? levels in the bloodstream of some
A. Individuals who are underweight do individuals.
not run any risk of developing high E. Only regular exercise is necessary to
levels of cholesterol in the decrease cholesterol levels in the
bloodstream. bloodstream of individuals of average
weight.

36. 3𝑥 + 𝑥 + 𝑥 + 𝑥 − 3 − 2 = 7 + 38. The formula below is often used by project


𝑥 + 𝑥 managers to compute 𝐸, the estimated
In the equation above, what is the value of time to complete a job, where 𝑂 is the
𝑥? shortest completion time, 𝑃 is the longest
5
A. − completion time, and 𝑀 is the most likely
7
B. 1 completion time.
12 𝑂 + 4𝑀 + 𝑃
C. 𝐸 =
7 6
D. 3 Which of the following correctly gives 𝑃 in
terms of 𝐸, 𝑂, 𝑎𝑛𝑑 𝑀 ?
37. The graph below shows the distance A. 𝑃 = 6𝐸 − 𝑂 − 4𝑀
traveled 𝑑, in feet, by a product on a B. 𝑃 = −6𝐸 + 𝑂 + 4𝑀
conveyor belt m minutes after the product 𝑂+4𝑀+𝐸
C. 𝑃 =
6
is placed on the belt. Which of the 𝑂+4𝑀−𝐸
following equations correctly relates 𝑑 and D. 𝑃 =
6
𝑚?
39. In the figure below, 𝑅𝑇 = 𝑇𝑈. What is the
value of 𝑥 ?

A. 72
B. 66
A. 𝑑 = 2𝑚 C. 64
1
B. 𝑑 = 𝑚 D. 58
2
C. 𝑑 = 𝑚 + 2
D. 𝑑 = 2𝑚 + 2
B E R A N I B E R J U A N G | 89
40. The width of a rectangular dance floor is 𝑤 45. A group of 202 people went on an
feet. The length of the floor is 6 feet longer overnight camping trip, taking 60 tents
than its width. Which of the following with them. Some of the tents held 2 people
expresses the perimeter, in feet, of the each, and the rest held 4 people each.
dance floor in terms of 𝑤 ? Assuming all the tents were filled to
A. 2𝑤 + 6 capacity and every person got to sleep in a
B. 4𝑤 + 12 tent, exactly how many of the tents were 2-
C. 𝑤 2 + 6 person tents?
D. 𝑤 2 + 6𝑤 A. 30
B. 20
41. 𝑦 > 2𝑥 − 1 C. 19
2𝑥 > 5 D. 18
Which of the following consists of the 𝑦-
coordinates of all the points that satisfy the 46. Which of the following could be the
system of inequalities above? equation of the graph below?
A. 𝑦 > 6
B. 𝑦 > 4
C. 𝑦 > 52
D. 𝑦 > 32

42. √2𝑥 + 6 + 4 = 𝑥 + 3
What is the solution set of the equation
above? A. 𝑦 = 𝑥(𝑥 − 2)(𝑥 + 3)
A. {1} B. 𝑦 = 𝑥 2 (𝑥 − 2)(𝑥 + 3)
B. {5} C. 𝑦 = 𝑥(𝑥 + 2)(𝑥 − 3)
C. {−1. 5} D. 𝑦 = 𝑥 2 (𝑥 + 2)(𝑥 − 3)
D. {0, −1,5}
2𝑎 𝑏
3 47. If = 12, what is the value of ?
43. 𝑓(𝑥) = 𝑥 − 9𝑥 𝑏 𝑎
1
𝑔(𝑥) = 𝑥 2 − 2𝑥 − 3 A.
8
Which of the following expressions is 1
B.
𝑓(𝑥) 4
equivalent to , for 𝑥 > 3 ? C. 2
𝑔(𝑥)

A.
1 D. 4
𝑥+1
𝑥+3
B. 48. What value of 𝑥 satisfies the equation
𝑥+1
C. 𝑥(𝑥 − 3)𝑥 + 1 3𝑥 + 3 = 27 ?
D. 𝑥(𝑥 + 3)𝑥 + 1 A. 3
B. 8
44. (𝑥 − 6)2 + (𝑦 + 5)2 = 16 C. 10
In the 𝑥𝑦-plane, the graph of the equation D. 27
above is a circle. Point 𝑃 is on the circle and
has coordinates (10, −5). If ̅̅̅̅ 𝑃𝑄 is a 49. 𝑦 = 𝑥 2 + 3𝑥 − 7
diameter of the circle, what are the 𝑦 − 5𝑥 + 8 = 0
coordinates of point 𝑄 ? How many solutions are there to the
A. (2, −5) system of equations above?
B. 6, −1) A. There are exactly 4 solutions.
C. (6, −5) B. There are exactly 2 solutions.
D. (6, −9) C. There is exactly 1 solutions.
D. There are no solutions.

90 | I U P I T B
50. 𝑔(𝑥) = 2𝑥 − 1 Questions 55 and 56 refer to the following
ℎ(𝑥) = 1 − 𝑔(𝑥) information.
The functions 𝑔 and ℎ are defined above.
What is the value of ℎ (0) ?
A. −2
B. 0
C. 1
D. 2

51. Oil and gas production in a certain area


dropped from 4 million barrels in 2000 to
1.9 million barrels in 2013. Assuming that
the oil and gas production decreased at a
constant rate, which of the following linear
functions 𝑓 best models the production, in In an experiment, a heated cup of coffee is
millions of barrels, 𝑡 years after the year removed from a heat source, and the cup of
2000? coffee is then left in a room that is kept at a
21
A. 𝑓(𝑡) = 𝑡 + 4 constant temperature. The graph above shows
130
19 the temperature, in degrees Fahrenheit (°F), of
B. 𝑓(𝑡) = 𝑡 + 4
130
21 the coffee immediately after being removed
C. 𝑓(𝑡) = − 𝑡 + 4 from the heat source and at 10-minute intervals
130
19
D. 𝑓(𝑡) = − 𝑡 + 4 thereafter
130

52. Two units of length used in ancient Egypt 55. Of the following, which best approximates
were cubits and palms, where 1 cubit is the temperature, in degrees Fahrenheit, of
equivalent to 7 palms. The Great Sphinx the coffee when it is first removed from the
statue in Giza is approximately 140 cubits heat source?
long. Which of the following best A. 75
approximates the length, in palms, of the B. 100
Great Sphinx statue? C. 155
A. 0.05 D. 195
B. 20
C. 140 56. During which of the following 10-minute
D. 980 intervals does the temperature of the
coffee decrease at the greatest average
2𝑛 rate?
53. If = 10, what is the value of 2𝑛 − 1 ?
5 A. Between 0 and 10 minutes
A. 24
B. Between 30 and 40 minutes
B. 49
C. Between 50 and 60 minutes
C. 50
D. Between 90 and 100 minutes
D. 99
̅̅̅̅ intersects 𝐵𝐸
57. In the figure below, 𝐴𝐷 ̅̅̅̅ at 𝐶.
54. √𝑥 2 = 𝑥 If 𝑥 = 100, what is the value of 𝑦 ?
Which of the following values of 𝑥 is NOT a
solution to the equation above?
A. −4
B. 0
C. 1
D. 3

B E R A N I B E R J U A N G | 91
A. 100 customer selected at random on that day
B. 90 did not purchase gasoline?
C. 80
D. 60

58. The line graphed in the 𝑥𝑦-plane below


models the total cost, in dollars, for a cab
ride, 𝑦, in a certain city during nonpeak
hours based on the number of miles
traveled, 𝑥. According to the graph, what is
15
the cost for each additional mile traveled, A.
50
in dollars, of a cab ride? 15
B.
40
35
C.
50
50
D.
135

60. Washington High School randomly selected


freshman, sophomore, junior, and senior
students for a survey about potential
changes to next year’s schedule. Of
1
students selected for the survey , were
4
1
freshmen and were sophomores. Half of
3
the remaining selected students were
juniors. If 336 students were selected for
A. $2.00 the survey, how many were seniors?
B. $2.60 A. 240
C. $3.00 B. 140
D. $5.00 C. 120
D. 70
59. On Tuesday, a local gas station had 135
customers. The table above summarizes
whether or not the customers on Tuesday
purchased gasoline, a beverage, both, or
neither. Based on the data in the table,
what is the probability that a gas station

61. 𝟑𝟔𝟎 = 𝟐𝒙 𝟑𝒚𝟓𝒛


𝑸𝒖𝒂𝒏𝒕𝒊𝒕𝒚 𝑨 𝑸𝒖𝒂𝒏𝒕𝒊𝒕𝒚 𝑩

𝑥 𝑦+𝑧
A. Quantity A is greater.
B. Quantity B is greater.
C. The two quantities are equal.
D. The relationship cannot be determined from the information given.

92 | I U P I T B
62. A vase contains 16 tulips, of which 8 are red, 3 are purple, and 5 are white.

𝑸𝒖𝒂𝒏𝒕𝒊𝒕𝒚 𝑨 𝑸𝒖𝒂𝒏𝒕𝒊𝒕𝒚 𝑩

𝑇ℎ𝑒 𝑝𝑟𝑜𝑏𝑎𝑏𝑖𝑙𝑖𝑡𝑦 𝑜𝑓 𝑠𝑒𝑙𝑒𝑐𝑡𝑖𝑛𝑔 𝑡𝑤𝑜 𝑇ℎ𝑒 𝑝𝑟𝑜𝑏𝑎𝑏𝑖𝑙𝑖𝑡𝑦 𝑜𝑓 𝑠𝑒𝑙𝑒𝑐𝑡𝑖𝑛𝑔 𝑜𝑛𝑒 𝑝𝑢𝑟𝑝𝑙𝑒


𝑐𝑜𝑛𝑠𝑒𝑐𝑢𝑡𝑖𝑣𝑒 𝑟𝑒𝑑 𝑡𝑢𝑙𝑖𝑝𝑠 𝑎𝑡 𝑟𝑎𝑛𝑑𝑜𝑚 𝑖𝑓 𝑡𝑢𝑙𝑖𝑝 𝑎𝑡 𝑟𝑎𝑛𝑑𝑜𝑚
𝑡ℎ𝑒 𝑓𝑖𝑟𝑠𝑡 𝑡𝑢𝑙𝑖𝑝 𝑖𝑠 𝑛𝑜𝑡 𝑟𝑒𝑝𝑙𝑎𝑐𝑒𝑑
A. Quantity A is greater.
B. Quantity B is greater.
C. The two quantities are equal.
D. The relationship cannot be determined from the information given.

63. 𝒇(𝒙) = 𝒙𝟑 + 𝒙𝟐 − 𝟏𝟔𝒙 + 𝟑


𝑸𝒖𝒂𝒏𝒕𝒊𝒕𝒚 𝑨 𝑸𝒖𝒂𝒏𝒕𝒊𝒕𝒚 𝑩

𝑓(−4) 𝑓(4)
A. Quantity A is greater.
B. Quantity B is greater.
C. The two quantities are equal.
D. The relationship cannot be determined from the information given.

64. A bag contains two 0.50-carat diamonds and two 0.75-carat diamonds and no others.
Three diamonds are drawn at random from the bag without replacement.

𝑸𝒖𝒂𝒏𝒕𝒊𝒕𝒚 𝑨 𝑸𝒖𝒂𝒏𝒕𝒊𝒕𝒚 𝑩

𝑇ℎ𝑒 𝑝𝑟𝑜𝑏𝑎𝑏𝑖𝑙𝑖𝑡𝑦 𝑜𝑓 𝑑𝑟𝑎𝑤𝑖𝑛𝑔 𝑜𝑛𝑒 𝑇ℎ𝑒 𝑝𝑟𝑜𝑏𝑎𝑏𝑖𝑙𝑖𝑡𝑦 𝑜𝑓 𝑑𝑟𝑎𝑤𝑖𝑛𝑔 𝑜𝑛𝑒


0.50 − 𝑐𝑎𝑟𝑎𝑡 𝑎𝑛𝑑 𝑡𝑤𝑜 0.75 − 𝑐𝑎𝑟𝑎𝑡 0.75 − 𝑐𝑎𝑟𝑎𝑡 𝑎𝑛𝑑 𝑡𝑤𝑜 0.50 − 𝑐𝑎𝑟𝑎𝑡
𝑑𝑖𝑎𝑚𝑜𝑛𝑑𝑠 𝑑𝑖𝑎𝑚𝑜𝑛𝑑𝑠
A. Quantity A is greater.
B. Quantity B is greater.
C. The two quantities are equal.
D. The relationship cannot be determined from the information given.

65. 𝒂 𝟑
=
𝒃 𝟒
𝑸𝒖𝒂𝒏𝒕𝒊𝒕𝒚 𝑨 𝑸𝒖𝒂𝒏𝒕𝒊𝒕𝒚 𝑩
2𝑎 − 𝑏 𝑎
𝑏 𝑎 − 𝑏
A. Quantity A is greater.
B. Quantity B is greater.
C. The two quantities are equal.
D. The relationship cannot be determined from the information given.

66. Data Sufficiency: What is the 6th term of the Arithmetic sequence?
(1) The sum of the 6th to the 12th term of the sequence is 77.
(2) The sum of the 2nd to the 10th term of the sequence is 108.
A. Statement (1) ALONE is sufficient, but statement (2) alone is not sufficient.
B. Statement (2) ALONE is sufficient, but statement (1) alone is not sufficient.
C. BOTH statements TOGETHER are sufficient, but NEITHER statement ALONE is sufficient.
D. EACH statement ALONE is sufficient.
E. Statements (1) and (2) TOGETHER are NOT sufficient.

B E R A N I B E R J U A N G | 93
67. Data Sufficiency: What is the value of 𝑋, if 𝑋 and 𝑌 are two distinct integers and their product
is 30?
(1) 𝑋 is an odd integer
(2) 𝑋 > 𝑌
A. Statement (1) ALONE is sufficient, but statement (2) alone is not sufficient.
B. Statement (2) ALONE is sufficient, but statement (1) alone is not sufficient.
C. BOTH statements TOGETHER are sufficient, but NEITHER statement ALONE is sufficient.
D. EACH statement ALONE is sufficient.
E. Statements (1) and (2) TOGETHER are NOT sufficient.

68. Data Sufficiency: Is 𝑦 an integer?


(1) 𝑦 3 is an integer
(2) 3𝑦 is an integer
A. Statement (1) ALONE is sufficient, but statement (2) alone is not sufficient.
B. Statement (2) ALONE is sufficient, but statement (1) alone is not sufficient.
C. BOTH statements TOGETHER are sufficient, but NEITHER statement ALONE is sufficient.
D. EACH statement ALONE is sufficient.
E. Statements (1) and (2) TOGETHER are NOT sufficient.

69. Data Sufficiency: Is the positive integer ′𝑥′ divisible by 12?


(1) 𝑥 is divisible by 6
(2) 𝑥 is divisible by 8
A. Statement (1) ALONE is sufficient, but statement (2) alone is not sufficient.
B. Statement (2) ALONE is sufficient, but statement (1) alone is not sufficient.
C. BOTH statements TOGETHER are sufficient, but NEITHER statement ALONE is sufficient.
D. EACH statement ALONE is sufficient.
E. Statements (1) and (2) TOGETHER are NOT sufficient.

̅̅̅̅. What is the value of


70. In triangle 𝑅𝑆𝑇 below, point 𝑊 (not shown) lies on 𝑅𝑇
𝑐𝑜𝑠(∠𝑅𝑆𝑊) – 𝑠𝑖𝑛 (∠𝑊𝑆𝑇) ?

71. 𝑥 2 + 𝑥 − 12 = 0
If 𝑎 is a solution of the equation above and 𝑎 > 0 what is the value of 𝑎?

72. The sum of −2𝑥 2 + 𝑥 + 31 and 3𝑥 2 + 7𝑥 − 8 can be written in the form 𝑎𝑥 2 + 𝑏𝑥 + 𝑐,


where 𝑎, 𝑏, and 𝑐 are constants. What is the value of 𝑎 + 𝑏 + 𝑐 ?

73. −𝑥 + 𝑦 = −3.5
𝑥 + 3𝑦 = 9.5
If (𝑥, 𝑦) satisfies the system of equations above, what is the value of 𝑦?

94 | I U P I T B
Questions 74 and 75 refer to the following information.

When a patient receives a penicillin injection, the kidneys begin removing the penicillin from the
body. The table and graph above show the penicillin concentration in a patient’s bloodstream at 5-
minute intervals for the 20 minutes immediately following a one-time penicillin injection.

74. According to the table, how many more micrograms of penicillin are present in 10 milliliters of
blood drawn from the patient 5 minutes after the injection than are present in 8 milliliters of
blood drawn 10 minutes after the injection?

75. The penicillin concentration, in micrograms per milliliter, in the patient’s bloodstream 𝑡
𝑡
minutes after the penicillin injection is modeled by the function 𝑃 defined by 𝑃(𝑡) = 200𝑏 5 .
If 𝑃 approximates the values in the table to within 10 micrograms per milliliter, what is the
value of 𝑏, rounded to the nearest tenth?

B E R A N I B E R J U A N G | 95
SET-5

Questions 1-11 are based on the following passage and supplementary material.

This passage is adapted from Brian Greene. 'How the Higgs Boson Was Found? 02013 by
Smithsonian Institution. The Higgs boson is an elementary particle associated with the Higgs field.
Experiments conducted in 2012-2013 tentatively confirmed the existence of the Higgs boson and
thus of the Higgs field.

Line Nearly a half-century ago. Peter Hi s and a handful of other physicists were trying to
understand the origin of a basic physical feature: mass. You can think of mass as an
object's heft or, a little more precisely, as the resistance it offers to having its motion
changed. Push on a freight train (or a feather) to increase its speed, and the resistance
5 you feel reflects its mass. At a microscopic level, the freight train's mass comes from its
constituent molecules and atoms, which are themselves built from fundamental
particles, electrons and quarks. But where do the masses of these and other fundamental
particles come from?
When physicists in the 1960s modeled the behavior of these particles using
10 equations rooted in quantum physics, they encountered a puzzle. If they imagined that
the particles were all massless, then each term in the equations clicked into a perfectly
symmetric pattern, like the tips of a perfect snowflake. And this symmetry was not just
mathematically elegant. It explained patterns evident in the experimental data. But—
and here's the puzzle—physicists knew that the particles did have mass, and when they
15 modified the equations to account for this fact, the mathematical harmony was spoiled.
The equations became complex and unwieldy and. worse still, inconsistent.
What to do? Here's the idea put forward by Higgs. Don't shove the particles' masses
down the throat of the beautiful equations. Instead, keep the equations pristine and
symmetric, but consider them operating within a peculiar environment. Imagine that all
20 of space is uniformly filled with an invisible substance—now called the Higgs field—that
exerts a drag force on particles when they accelerate through it. Push on a fundamental
particle in an effort to increase its speed and, according to His, you would feel this drag
force as a resistance. Justifiably, you would interpret the resistance as the particle's mass.
For a mental toehold, think of a ping-pong ball submerged in water. When you push on
25 the ping-pong ball, it will feel much more massive than it does outside of water. Its

96 | I U P I T B
interaction with the watery environment has the effect of endowing it with mass. So with
particles submerged in the Higgs field.
In 1964, Higgs submitted a paper to a prominent physics journal in which he
formulated this idea mathematically. The paper was rejected. Not because it contained
30 a technical error, but because the premise of an invisible something permeating space,
interacting with particles to provide their mass, well, it all just seemed like heaps of
overwrought speculation. The editors of the journal deemed it 'of no obvious relevance
to physics."
But Higgs persevered (and his revised paper appeared later that year in another
35 journal), and physicists who took the time to study the proposal gradually realized that
his idea was a stroke of genius, one that allowed them to have their cake and eat It too.
In Higgs's scheme, the fundamental equations can retain their pristine form because the
dirty work of providing the particles masses is relegated to the environment.
While I wasn't around to witness the initial rejection of His s's proposal in 1964 (well,
40 I was around, but only barely). I can attest that by the mid-1980s, the assessment had
changed. The physics community had, for the most part, fully bought into the idea that
there was a Higgs field permeating space. In fact, in a graduate course I took that covered
what's known as the Standard Model of Particle Physics (the quantum equations
physicists have assembled to describe the particles of matter and the dominant forces
45 by which they influence each other), the professor presented the Higgs field with such
certainty that for a long while I had no idea it had yet to be established experimentally.
On occasion, that happens in physics. Mathematical equations can sometimes tell such
a convincing tale, they can seemingly radiate reality so strongly, that they become
entrenched in the vernacular of working physicists, even before there's data to confirm
50 them.

1. Over the course of the passage, the main D. an analysis of the Higgs field to a
focus shifts from suggestion of future discoveries that
A. a technical account of the Higgs field to might build upon it.
a description of it aimed at a broad
audience. 2. The main purpose of the analogy of the
B. a review of Higgs’s work to a ping-pong ball (line 25) is to
contextualization of that work within A. popularize a little-known fact.
Higgs’s era. B. contrast competing scientific theories.
C. an explanation of the Higgs field to a C. criticize a widely accepted
discussion of the response to Higgs’s explanation.
theory. D. clarify an abstract concept.
B E R A N I B E R J U A N G | 97
3. The author most strongly suggests that the D. He contrasts the status of Higgs’s
reason the scientific community initially theory at two time periods to reveal
rejected Higgs’s idea was that the idea how the details of the theory evolved.
A. addressed a problem unnoticed by
other physicists. 8. What purpose does the graph serve in
B. only worked if the equations were relation to the passage as a whole?
flawless.
A. It indicates that the scientific
C. rendered accepted theories in physics
community’s quick acceptance of the
obsolete.
D. appeared to have little empirical basis. Higgs boson was typical.
B. It places the discussion of the
4. Which choice provides the best evidence reception of the Higgs boson into a
for the answer to the previous question? broader scientific context.
A. Lines 18-19 (“Instead . . . C. It demonstrates that the Higgs boson
environment”) was regarded differently than were
B. Lines 28-29 (“In 1964 . . . other hypothetical particles.
mathematically”) D. It clarifies the ways in which the Higgs
C. Lines 29-32 (“Not . . . speculation”) boson represented a major discovery.
D. Lines 42-43 (“The physics . . . space”) 9. Which statement is best supported by the
data presented in the graph?
5. The author notes that one reason Higgs’s
A. The W boson and the Z boson were
theory gained acceptance was that it
proposed and experimentally
A. let scientists accept two conditions
that had previously seemed confirmed at about the same time.
irreconcilable. B. The Higgs boson was experimentally
B. introduced an innovative approach confirmed more quickly than were
that could be applied to additional most other particles.
problems. C. The tau neutrino was experimentally
C. answered a question that earlier confirmed at about the same time as
scientists had not even raised. the tau.
D. explained why two distinct D. The muon neutrino took longer to
phenomena were being experimentally confirm than did the
misinterpreted as one phenomenon. electron neutrino.
10. Based on the graph, the author’s depiction
6. Which choice provides the best evidence
of Higgs’s theory in the mid-1980s is most
for the answer to the previous question?
A. Lines 21-24 (“Push . . . mass”) analogous to which hypothetical situation?
B. Lines 26-27 (“Its interaction . . . field”) A. The muon neutrino was widely
C. Lines 35-39 (“But . . . environment”) disputed until being confirmed in the
D. Lines 48-51 (“On occasion . . . them”) early 1960s.
B. Few physicists in 2012 doubted the
7. Which statement best describes the reality of the tau neutrino.
technique the author uses to advance the C. No physicists prior to 1960 considered
main point of the last paragraph? the possibility of the W or Z boson.
A. He recounts a personal experience to D. Most physicists in 1940 believed in the
illustrate a characteristic of the existence of the electron neutrino.
discipline of physics.
11. As used in line 47, “established” most
B. He describes his own education to
nearly means
show how physics has changed during
A. validated.
his career.
B. founded.
C. He provides autobiographical details
C. introduced.
to demonstrate how Higgs’s theory
D. enacted.
was confirmed.
98 | I U P I T B
Questions 12–21 are based on the following passage.

NEH: A Human-Centered Agency


The National Endowment for the Humanities (NEH) is an independent federal agency that was
created in 1965 to provide grants to humanities projects throughout the United States. Funding for
the agency has been the (12) protagonist of debate for many years. Some critics (13) think the
money that goes to the NEH would be better spent on infrastructure or job creation, while others
object to the nature of some of the projects that receive funding. (14) Therefore, the agency
provides important services in all fifty states and has had a notable impact on American culture
over the last fifty years.

12. 14.
A. NO CHANGE A. NO CHANGE
B. significance B. Likewise,
C. discipline C. However,
D. subject D. For instance,
13.
A. NO CHANGE
B. Thought
C. Thinking
D. would think

The NEH was founded as a direct response to an explosion in scientific research in the middle
years of the 20th century. There was concern, especially among those in the fields of arts and
humanities, that non-scientific pursuits were in danger of getting left behind or (15) overlooked.
The NEH addresses this concern by distributing grant money in seven areas, including preservation,
research, education, and digital humanities.
Among the most notable projects that have been funded by the NEH over the last 50 years are
the Ken Burns documentary The Civil War, the blockbuster Metropolitan Museum of Art exhibition
“Treasures of (16) Tutankhamen”; and sixteen Pulitzer Prize-winning books.

15. 16.
A. NO CHANGE A. NO CHANGE
B. in being overlooked. B. Tutankhamen” and;
C. of oversight. C. Tutankhamen,” and
D. to be overlooked. D. Tutankhamen” and,

Although some critics of the NEH argue that the agency’s spending is frivolous in an age when
our country is in desperate need of spending on more concrete things like infrastructure, NEH grant
money (17) which has a positive impact on local economies. Grants that support construction or
renovation of facilities employ local construction workers, and the construction or expansion of a
museum creates permanent jobs for staff. Preservation funds (18) likewise similarly create jobs for
archivists and technicians. One striking example is the NEH-funded excavation of Historic
Jamestown— the first permanent English colony in America— which has resulted in the creation of
an entirely new local tourist industry. (19)

17. 18.
A. NO CHANGE A. NO CHANGE
B. Has B. in the same way
C. Having C. comparably
D. to have D. DELETE the underlined portion.

B E R A N I B E R J U A N G | 99
19. At this point, the writer is considering B. Yes, because it reinforces the
adding the following sentence. Jamestown importance of the NEH grant on the
was thought to be lost for hundreds of region’s economy.
years— scholars knew where it should be, C. No, because it blurs the paragraph’s
but found no evidence of it. Should the focus on the far- reaching economic
writer make this addition here? effects NEH grants can have.
A. Yes, because it further explains why D. No, because it undermines the idea
the discovery of Jamestown was that tourist attractions can contribute
important. to the local economy.

[1] As federal funding for scientific research and the military continues to increase, we should
not forget Seaborg’s words. [2] In the early 1960s, Glenn Seaborg, then head of the Atomic Energy
Commission, expressed his support for establishing the NEH by (20) cautioning against an over-
reliance on technology: “Science and technology are providing us with the means to travel swiftly.
But what course do we take? This is the question that no computer can answer.” [3] The issue
should not be (21) weather to fund the NEH but how much. [4] NEH grants help inform the kind of
cultural awareness that is vital to our roles as good citizens in a global community. 9. Which choice
most effectively sets up the quote that follows in this sentence?

20. 21.
A. NO CHANGE A. NO CHANGE
B. emphasizing the importance of B. weather too
science over humanities: C. whether to
C. lobbying for increased funding for D. whether too
computer research:
D. arguing for the importance of public
art:

Questions 22-25 are based on the following passage and supplementary material.

It looks like the trail that leads to a magical land. The path climbs out of the forest ---(22)--- white
and yellow butterflies flutter ---(23)--- the hanging creepers. It emerges on a ---(24)--- meadow
basking in the sun ---(25)--- this is no fairy tale country.

22. In each of the following passages, there are 24. In each of the following passages, there are
blanks each of which has been numbered. blanks each of which has been numbered.
Find out the appropriate words. Find out the appropriate words.
A. where A. dry
B. which B. gaudy
C. when C. beautiful
D. that D. stony

23. In each of the following passages, there are 25. In each of the following passages, there are
blanks each of which has been numbered. blanks each of which has been numbered.
Find out the appropriate words. Find out the appropriate words.
A. on A. so
B. in B. as
C. at C. and
D. among D. but

100 | I U P I T B
26. Assume that some pings are lings, all hings A thin, red-haired baby wearing a bonnet is
are fings, and some tings are pings. the oldest of the group. Who can it be?
Therefore it makes sense that: A. Baby 1
A. some lings may also be tings B. Baby 2
B. all pings are lings C. Baby 3
C. all hings are tings D. Baby 4
D. some fings are lings E. Baby 5
E. all hings are pings
31. Which of the following best completes the
27. Four of the following words are alike in passage below?
some way. People buy prestige when they buy a
Which is the odd word out? premium product. They want to be
A. conventional associated with something special. Mass-
B. peculiar marketing techniques and price-reduction
C. conservative strategies should not be used
D. typical because____.
E. traditional A. affluent purchasers currently
represent a shrinking portion of the
28. Which of the following words is most population of all purchasers
similar in meaning to perplexing: B. continued sales depend directly on the
A. curious maintenance of an aura of exclusivity
B. ambiguous C. purchasers of premium products are
C. complex concerned with the quality as well as
D. incomprehensible with the price of the products
E. confucious D. expansion of the market niche to
include a broader spectrum of
29. All of the words in the box below are alike consumers will increase profits
in some way. E. manufacturing a premium brand is not
feeling altruistic necessarily more costly than
human volunteer manufacturing a standard brand of the
Which of the following options could not go same product
in the box?
A. malevolent 32. When limitations were in effect on nuclear-
B. selfless arms testing, people tended to save more
C. humanitarian of their money, but when nuclear-arms
D. unselfish testing increased, people tended to spend
E. generous more of their money. The perceived threat
of nuclear catastrophe, therefore,
30. In a hospital maternity ward there are only decreases the willingness of people to
five babies. Baby 1 is heavier, with red hair. postpone consumption for the sake of
Baby 2 is male and thin, with the same saving money.
colour hair as Baby 3, who is blonde and The argument above assumes that
wears a bonnet. Baby 4 is one of the three A. the perceived threat of nuclear
female babies and Baby 5 wears sunglasses catastrophe has increased over the
and is heavier. All babies wear bonnets and years.
at least one of the females wears B. most people supported the
sunglasses. development of nuclear arms
B E R A N I B E R J U A N G | 101
C. people's perception of the threat of such countries import all or none of their
nuclear catastrophe depends on the oil.If the statement in the passage
amount of nuclear-arms testing being concerning oil-supply disruptions is true,
done which of the following policies in an open-
D. the people who saved the most money market nation is most likely to reduce the
when nuclear-arms testing was limited long-term economic impact on that nation
were the ones who supported such of sharp and unexpected increases in
limitations international oil prices?
E. there are more consumer goods A. Maintaining the quantity of oil
available when nuclear-arms testing imported at constant yearly levels
increases B. Increasing the number of oil tankers in
its fleet
33. A cost-effective solution to the problem of C. Suspending diplomatic relations with
airport congestion is to provide high-speed major oil-producing nations
ground transportation between major D. Decreasing oil consumption through
cities lying 200 to 500 miles apart. The conservation
successful implementation of this plan E. Decreasing domestic production of oil
would cost far less than expanding existing
airports and would also reduce the number 35. If there is an oil-supply disruption resulting
of airplanes clogging both airports and in higher international oil prices, domestic
airways. oil prices in open-market countries such as
Which of the following, if true, could be the United States will rise as well, whether
proponents of the plan above most such countries import all or none of their
appropriately cite as a piece of evidence for oil. Which of the following conclusions is
the soundness of their plan? best supported by the statement in the
A. An effective high-speed ground- passage?
transportation system would require A. Domestic producers of oil in open-
major repairs to many highways and market countries are excluded from
mass-transit improvements. the international oil market when
B. One-half of all departing flights in the there is a disruption in the
nation's busiest airport head for a international oil supply.
destination in a major city 225 miles B. International oil-supply disruptions
away. have little, if any, effect on the price of
C. The majority of travelers departing domestic oil as long as an open-market
from rural airports are flying to country has domestic supplies capable
destinations in cities over 600 miles of meeting domestic demand.
away. C. The oil market in an open-market
D. Many new airports are being built in country is actually part of the
areas that are presently served by international oil market, even if most
high-speed ground-transportation of that country's domestic oil is usually
systems. sold to consumers within its borders.
E. A large proportion of air travelers are D. Open-market countries that export
vacationers who are taking long- little or none of their oil can maintain
distance flights. stable domestic oil prices even when
international oil prices rise sharply.
34. If there is an oil-supply disruption resulting E. If international oil prices rise, domestic
in higher international oil prices, domestic distributors of oil in open-market
oil prices in open-market countries such as countries will begin to import more oil
the United States will rise as well, whether than they export

102 | I U P I T B
36. Plant 𝐴 is currently 20 centimeters tall, and C. 𝑣 = 120,000 + 9,000𝑡
Plant 𝐵 is currently 12 centimeters tall. The D. 𝑣 = 120,000 − 30,000𝑡
ratio of the heights of Plant 𝐴 to Plant 𝐵 is
equal to the ratio of the heights of Plant 𝐶 40. Line 𝑚 in the 𝑥𝑦-plane contains the points
to Plant D. If Plant 𝐶 is 54 centimeters tall, (2, 4) and (0, 1). Which of the following is
what is the height of Plant 𝐷, in an equation of line 𝑚?
centimeters? A. 𝑦 = 2𝑥 + 3
A. 32.4 B. 𝑦 = 2𝑥 + 4
3
B. 44.0 C. 𝑦 = 𝑥 + 3
2
C. 62.0 3
D. 𝑦 = 𝑥 + 1
D. 90.0 2

37. Biologists found a new species of pale 41. (4𝑥 + 4)(𝑎𝑥 − 1) − 𝑥 2 + 4


shrimp at the world’s deepest undersea In the expression above, 𝑎 is a constant. If
vent, the Beebe Vent Field. The vent is 3.1 the expression is equivalent to 𝑏𝑥, where 𝑏
miles below the sea’s surface. is a constant, what is the value of 𝑏 ?
Approximately how many kilometers A. −5
below the sea’s surface is the vent? (1 B. −3
kilometer ≈ 0.6214 miles) C. 0
A. 2 D. 12
B. 3
C. 4 42. If 2𝑤 + 4𝑡 = 14 and 4𝑤 + 5𝑡 = 25,
D. 5 what is the value of 2𝑤 + 3𝑡 ?
A. 6
38. A cargo helicopter delivers only 100-pound B. 10
packages and 120-pound packages. For C. 13
each delivery trip, the helicopter must D. 17
carry at least 10 packages, and the total
weight of the packages can be at most Questions 43-45 refer to the following
1,100 pounds. What is the maximum information.
number of 120-pound packages that the
helicopter can carry per trip? Jennifer bought a box of Crunchy Grain cereal.
A. 2 The nutrition facts on the box state that a
3
B. 4 serving size of the cereal is cup and provides
4
C. 5 210 calories, 50 of which are calories from fat.
D. 6 In addition, each serving of the cereal provides
180 milligrams of potassium, which is 5% of the
39. A company purchased a machine valued at daily allowance for adults
$120,000. The value of the machine
depreciates by the same amount each year 43. If 𝑝 percent of an adult’s daily allowance of
so that after 10 years the value will be potassium is provided by 𝑥 servings of
$30,000. Which of the following equations Crunchy Grain cereal per day, which of the
gives the value, 𝑣, of the machine, in following expresses 𝑝 in terms of 𝑥?
dollars, 𝑡 years after it was purchased for A. 𝑝 = 0.5𝑥
0 ≤ 𝑡 ≤ 10 ? B. 𝑝 = 5𝑥
A. 𝑣 = 30,000 − 9,000𝑡 C. 𝑝 = (0.05)𝑥
B. 𝑣 = 120, 000 − 9, 000𝑡 D. 𝑝 = (1.05)𝑥

B E R A N I B E R J U A N G | 103
44. On Tuesday, Jennifer will mix Crunchy A. ℎ(𝑥) = −3(𝑑) 𝑥
Grain cereal with Super Grain cereal for her B. ℎ(𝑥) = 3(𝑥)𝑑
breakfast Super Grain cereal provides 240 C. ℎ(𝑥) = 𝑑(−𝑥)3
calories per cup. If the total number of D. ℎ(𝑥) = −𝑑(3) 𝑥
calories in one cup of Jennifer’s mixture is
270, how much Super Grain cereal is in one 47. The weights, in pounds, for 15 horses in a
cup of the mixture? stable were reported, and the mean,
1 median, range, and standard deviation for
A. 𝑐𝑢𝑝
8
1 the data were found. The horse with the
B. 𝑐𝑢𝑝 lowest reported weight was found to
4
1
C. 𝑐𝑢𝑝 actually weigh 10 pounds less than its
3
1 reported weight. What value remains
D. 𝑐𝑢𝑝
2
unchanged if the four values are reported
using the corrected weight?
45. Which of the following could be the graph
A. Mean
of the number of calories from fat in
B. Median
Crunchy Grain cereal as a function of the
3 C. Range
number servings of the cereal? D. Standard deviation
4

48. Near the end of a US cable news show, the


host invited viewers to respond to a poll on
the show’s website that asked, “Do you
support the new federal policy discussed
during the show?” At the end of the show,
A.
the host reported that 28% responded
“Yes,” and 70% responded “No.” Which of
the following best explains why the results
are unlikely to represent the sentiments of
the population of the United States?
A. The percentages do not add up to
B. 100%, so any possible conclusions
from the poll are invalid.
B. Those who responded to the poll were
not a random sample of the
population of the United States.
C. There were not 50% “Yes” responses
C. and 50% “No” responses.
D. The show did not allow viewers
enough time to respond to the poll.

49. If 𝑓(𝑥) = 5𝑥 2 − 3 and 𝑓(𝑥) = 5𝑥 2 +


30𝑥 + 42, what is the value of 𝑎 ?
A. -30
D. B. -3
C. 3
46. The graph of the exponential function ℎ in
D. 30
the 𝑥𝑦-plane, where 𝑦 = ℎ(𝑥), has a 𝑦-
intercept of 𝑑, where 𝑑 is a positive
constant. Which of the following could
define the function ℎ?
104 | I U P I T B
50. If 𝑠𝑖𝑛 𝑥° = 𝑎, which of the following must
be true for all values of 𝑥 ?
A. 𝑐𝑜𝑠 𝑥° = 𝑎
B. 𝑠𝑖𝑛(90° − 𝑥°) = 𝑎
C. 𝑐𝑜𝑠(90° − 𝑥°) = 𝑎
D. 𝑠𝑖𝑛 (𝑥 2 )° = 𝑎2

51. ℎ(𝑥) = −16𝑥 2 + 100𝑥 + 10


The quadratic function above models the
height above the ground ℎ, in feet, of a
projectile 𝑥 seconds after it had been A. Each year between 1940 and 2010, the
launched vertically. If 𝑦 = ℎ(𝑥) is graphed
average increase in minimum wage
in the 𝑥𝑦- plane, which of the following
was 0.096 dollars.
represents the real-life meaning of the B. Each year between 1940 and 2010, the
positive 𝑥-intercept of the graph? average increase in minimum wage
A. The initial height of the projectile was 0.49 dollars.
B. The maximum height of the projectile
C. Every 10 years between 1940 and
C. The time at which the projectile
2010, the average increase in
reaches its maximum height minimum wage was 0.096 dollars.
D. The time at which the projectile hits D. Every 10 years between 1940 and
the ground
2010, the average increase in
minimum wage was 0.488 dollars.
52. In the 𝑥𝑦-plane, the graph of the
polynomial function 𝑓 crosses the 𝑥-axis at 55. The scatterplot above shows a company’s
exactly two points, (𝑎, 0) and (𝑏, 0), where ice cream sales 𝑑, in dollars, and the high
𝑎 and 𝑏 are both positive. Which of the
temperature 𝑡, in degrees Celsius (°C), on
following could define 𝑓?
12 different days. A line of best fit for the
A. 𝑓 (𝑥) = (𝑥 − 𝑎)(𝑥 − 𝑏)
data is also shown. Which of the following
B. 𝑓 (𝑥) = (𝑥 + 𝑎)(𝑥 + 𝑏)
could be an equation of the line of best fit?
C. 𝑓 (𝑥) = (𝑥 − 𝑎)(𝑥 + 𝑏)
D. 𝑓 (𝑥) = 𝑥(𝑥 − 𝑎)(𝑥 − 𝑏)

53. If 𝑦 = 3𝑥 2 + 6𝑥 + 2 is graphed in the


𝑥𝑦-plane, which of the following
characteristics of the graph is displayed as
a constant or coefficient in the equation?
A. 𝑦-coordinate of the vertex
B. 𝑥-intercept(s)
C. 𝑦-intercept A. 𝑑 = 0.03𝑡 + 402
D. 𝑥-intercept of the line of symmetry B. 𝑑 = 10𝑡 + 402
C. 𝑑 = 33𝑡 + 300
54. The scatterplot above shows the federal- D. 𝑑 = 33𝑡 + 84
mandated minimum wage every 10 years
between 1940 and 2010. A line of best fit is 56. Which of the following is equivalent to
shown, and its equation is 𝑦 = 0.096𝑥 − 2(𝑥 2 − 𝑥) + 3(𝑥 2 − 𝑥)?
0.488. What does the line of best fit A. 5𝑥 2 − 5𝑥
predict about the increase in the minimum B. 5𝑥 2 + 5𝑥
wage over the 70-year period? C. 5𝑥
D. 5𝑥 2

B E R A N I B E R J U A N G | 105
57. 2𝑥 − 𝑦 = 8 rises at a constant rate of 8 feet per second,
𝑥 + 2𝑦 = 4 which of the following equations gives the
For the system of equations above, what is height h, in feet, of the front of the roller-
the value of 𝑥 + 𝑦 ? coaster car s seconds after it starts up the
A. –1 hill?
B. 4 A. ℎ = 8𝑠 + 15
C. 5 335
B. ℎ = 15𝑠 +
8
D. 20 335
C. ℎ = 8𝑠 +
8
58. Which of the following statements is true D. ℎ = 15𝑠 + 8
about the graph of the equation 2𝑦 −
3𝑥 = −4 in the 𝑥𝑦-plane? 60. 𝐶 = 75ℎ + 125
A. It has a negative slope and a positive 𝑦- The equation above gives the amount 𝐶, in
intercept. dollars, an electrician charges for a job that
B. It has a negative slope and a negative takes ℎ hours. Ms. Sanchez and Mr. Roland
𝑦-intercept. each hired this electrician. The electrician
C. It has a positive slope and a positive 𝑦- worked 2 hours longer on Ms. Sanchez’s
intercept. job than on Mr. Roland’s job. How much
D. It has a positive slope and a negative 𝑦- more did the electrician charge Ms.
intercept. Sanchez than Mr. Ronald?
A. 75$
59. The front of a roller-coaster car is at the B. 125$
bottom of a hill and is 15 feet above the C. 150$
ground. If the front of the roller-coaster car D. 275$

61. 𝟏
In the 𝒙𝒚-plane, the equation of line k is 𝒙 + 𝟏 = 𝟓𝒚.
𝟐
𝑸𝒖𝒂𝒏𝒕𝒊𝒕𝒚 𝑨 𝑸𝒖𝒂𝒏𝒕𝒊𝒕𝒚 𝑩

𝑇ℎ𝑒 𝑠𝑙𝑜𝑝𝑒 𝑜𝑓 𝑙𝑖𝑛𝑒 𝑘 𝑇ℎ𝑒 𝑦 − 𝑖𝑛𝑡𝑒𝑟𝑐𝑒𝑝𝑡 𝑜𝑓 𝑙𝑖𝑛𝑒 𝑘


A. Quantity A is greater.
B. Quantity B is greater.
C. The two quantities are equal.
D. The relationship cannot be determined from the information given.

62. 𝒙 > 𝒚 > 𝒛


𝒙 𝒂𝒏𝒅 𝒚 𝒂𝒓𝒆 𝒐𝒅𝒅, 𝒂𝒏𝒅 𝒛 𝒊𝒔 𝒆𝒗𝒆𝒏
𝑸𝒖𝒂𝒏𝒕𝒊𝒕𝒚 𝑨 𝑸𝒖𝒂𝒏𝒕𝒊𝒕𝒚 𝑩

(−1)𝑥𝑦 − 𝑧 (−1)2𝑥𝑦 − 𝑧
A. Quantity A is greater.
B. Quantity B is greater.
C. The two quantities are equal.
D. The relationship cannot be determined from the information given.

106 | I U P I T B
63. 𝑳𝒊𝒔𝒕 𝑺 = 𝟏𝟏, 𝟐, 𝟒, 𝟏𝟑, 𝟏, 𝟐, 𝟏𝟎, 𝟖, 𝟑
𝑸𝒖𝒂𝒏𝒕𝒊𝒕𝒚 𝑨 𝑸𝒖𝒂𝒏𝒕𝒊𝒕𝒚 𝑩

𝑇ℎ𝑒 𝑚𝑒𝑑𝑖𝑎𝑛 𝑝𝑙𝑢𝑠 𝑡ℎ𝑒 𝑚𝑜𝑑𝑒 𝑜𝑓 𝑆 𝑇ℎ𝑒 𝑚𝑒𝑎𝑛 𝑜𝑓 𝑆


A. Quantity A is greater.
B. Quantity B is greater.
C. The two quantities are equal.
D. The relationship cannot be determined from the information given.

64. √𝒙𝟐 − 𝟔𝒙 + 𝟐𝟓 = 𝟒

𝑸𝒖𝒂𝒏𝒕𝒊𝒕𝒚 𝑨 𝑸𝒖𝒂𝒏𝒕𝒊𝒕𝒚 𝑩
𝑥 4
A. Quantity A is greater.
B. Quantity B is greater.
C. The two quantities are equal.
D. The relationship cannot be determined from the information given.

65. 𝑸𝒖𝒂𝒏𝒕𝒊𝒕𝒚 𝑨 𝑸𝒖𝒂𝒏𝒕𝒊𝒕𝒚 𝑩


𝑇ℎ𝑒 𝑠𝑢𝑚 𝑜𝑓 𝑡ℎ𝑒 𝑐𝑜𝑜𝑟𝑑𝑖𝑛𝑎𝑡𝑒𝑠 𝑇ℎ𝑒 𝑝𝑟𝑜𝑑𝑢𝑐𝑡 𝑜𝑓 𝑡ℎ𝑒 𝑐𝑜𝑜𝑟𝑑𝑖𝑛𝑎𝑡𝑒𝑠
𝑜𝑓 𝑎 𝑝𝑜𝑖𝑛𝑡 𝑖𝑛 𝑡ℎ𝑒 𝑓𝑜𝑢𝑟𝑡ℎ 𝑞𝑢𝑎𝑑𝑟𝑎𝑛𝑡 𝑜𝑓 𝑎 𝑝𝑜𝑖𝑛𝑡 𝑖𝑛 𝑡ℎ𝑒 𝑓𝑖𝑟𝑠𝑡 𝑞𝑢𝑎𝑑𝑟𝑎𝑛𝑡
𝑜𝑓 𝑎𝑛 𝑥𝑦 − 𝑐𝑜𝑜𝑟𝑑𝑖𝑛𝑎𝑡𝑒 𝑝𝑙𝑎𝑛𝑒 𝑜𝑓 𝑎𝑛 𝑥𝑦 − 𝑐𝑜𝑜𝑟𝑑𝑖𝑛𝑎𝑡𝑒 𝑝𝑙𝑎𝑛𝑒
A. Quantity A is greater.
B. Quantity B is greater.
C. The two quantities are equal.
D. The relationship cannot be determined from the information given.

66. Data Sufficiency: Is 𝑎𝑏 positive?


(1) (𝑎 + 𝑏)2 < (𝑎 − 𝑏)2
(2) 𝑎 = 𝑏
A. Statement (1) ALONE is sufficient, but statement (2) alone is not sufficient.
B. Statement (2) ALONE is sufficient, but statement (1) alone is not sufficient.
C. BOTH statements TOGETHER are sufficient, but NEITHER statement ALONE is sufficient.
D. EACH statement ALONE is sufficient.
E. Statements (1) and (2) TOGETHER are NOT sufficient.

67. Data Sufficiency: When the positive integer 𝑌 is divided by 2, is the remainder 1?
(1) (−1)(𝑌 + 2) = −1
(2) 𝑌 is prime
A. Statement (1) ALONE is sufficient, but statement (2) alone is not sufficient.
B. Statement (2) ALONE is sufficient, but statement (1) alone is not sufficient.
C. BOTH statements TOGETHER are sufficient, but NEITHER statement ALONE is sufficient.
D. EACH statement ALONE is sufficient.
E. Statements (1) and (2) TOGETHER are NOT sufficient.

B E R A N I B E R J U A N G | 107
68. Data Sufficiency: Is the two digit positive integer 𝑃 a prime number?
(1) (𝑃 + 2) and (𝑃 − 2) are prime.
(2) (𝑃 − 4) and (𝑃 + 4) are prime.
A. Statement (1) ALONE is sufficient, but statement (2) alone is not sufficient.
B. Statement (2) ALONE is sufficient, but statement (1) alone is not sufficient.
C. BOTH statements TOGETHER are sufficient, but NEITHER statement ALONE is sufficient.
D. EACH statement ALONE is sufficient.
E. Statements (1) and (2) TOGETHER are NOT sufficient.

69. Data Sufficiemcy: Is 𝑎 < 𝑏 ?


(1) 𝑎𝑏 < 𝑏 𝑎
𝑎
(2) > 1
𝑏
A. Statement (1) ALONE is sufficient, but statement (2) alone is not sufficient.
B. Statement (2) ALONE is sufficient, but statement (1) alone is not sufficient.
C. BOTH statements TOGETHER are sufficient, but NEITHER statement ALONE is sufficient.
D. EACH statement ALONE is sufficient.
E. Statements (1) and (2) TOGETHER are NOT sufficient.

70. A start-up company opened with 8 employees. The company’s growth plan assumes that 2
new employees will be hired each quarter (every 3 months) for the first 5 years. If an equation
is written in the form 𝑦 = 𝑎𝑥 + 𝑏 to represent the number of employees, 𝑦, employed by
the company 𝑥 quarters after the company opened, what is the value of 𝑏 ?

̂ is 25 of the circumference of
71. In the circle below, point 𝐴 is the center and the length of arc 𝐵𝐶
the circle. What is the value of 𝑥?

72. In the 𝑥𝑦-plane below, the circle has center (ℎ, 𝑘) and radius 10. What is the value of 𝑘?

73. In the 𝑥𝑦-plane, line 𝑙 has a 𝑦-intercept of −13 and is perpendicular to the line with equation
2
𝑦 = − 𝑥. If the point (10, 𝑏) is on line 𝑙, what is the value of 𝑏?
3

108 | I U P I T B
74. Human blood can be classified into four common blood types—A, B, AB, and O. It is also
characterized by the presence (+) or absence (–) of the rhesus factor. The table above shows
the distribution of blood type and rhesus factor for a group of people. If one of these people
who is rhesus negative (–) is chosen at random, the probability that the person has blood type
1
B is What is the value of 𝑥 ?
9
Blood Type
Rhesus Factor A B AB O
+ 33 9 3 37
- 7 2 1 𝑥

75. Based on the graph, in how many of the games played did the soccer team score goals equal
to the median number of goals for the 29 games?

B E R A N I B E R J U A N G | 109
SET-6

Questions 1–10 are based on the following passage.

Line And the words sang about him, filled the air with a mad pulsing of intolerable joy,
made him desire to fling himself in passionate humility at her feet, to weep hot tears, to
cry to her in insane worship. He thought her beautiful beyond anything his heart had
imagined; her warm gold hair was the rapture of his eyes and of his reverent hand.
5 Though slenderly fashioned, she was an gloriously strong. Not a day of illness in her life:
said Mrs. Yule, and one could readily believe it.
She spoke with such a sweet decision. Her 'I love you!' was a bond with eternity. In
the simplest as in the greatest things she saw his with and acted frankly upon it. No pretty
petulance, no affectation of silly-sweet languishing, none of the weaknesses of woman.
10 And so exquisitely fresh in her twenty years of maidenhood, with bright young eyes that
seemed to bid defiance to all the years to come.
He went about like one dazzled with excessive light. He talked as he had never talked
before, recklessly, exultantly, insolently—in the nobler sense. He made friends on every
hand; he welcomed all the world to his bosom; he felt the benevolence of a god.
15 'I love you!' It breathed like music at his ears when he fell asleep in weariness of joy;
it awakened him on the morrow as with a glorious ringing summons to renewed life.
Delay? Why should there be delay? Amy wished nothing but to become his wife. Idle to
think of his doing any more work until he sat down in the home of which she was
mistress. His brain burned with visions of the books he would henceforth write, but his
20 hand was incapable of anything but a love-letter. And what letters! Reardon never
published anything equal to those. 'I have received your poem: Amy replied to one of
them. And she was right not a letter, but a poem he had sent her, with every word on
fire.
The hours of talk! It enraptured him to find how much she had read, and with what
25 clearness of understanding. Latin and Greek, no. All! but she should learn them both,
that them might be nothing wanting in the communion between his thought and hers.
For he loved the old writers with all his heart they had been oath strength to him in his
days of misery
They would go together to the charmed lands of the South. No, not now for their
30 marriage holiday—Amy said that would be an imprudent expense; but as soon as he had

110 | I U P I T B
got a good price for a book. Will not the publishers be kind? If they knew what happiness
lurked in embryo within their foolish cheque-books!
He woke of a sudden in the early hours of one morning, a week before the wedding-
day You know that kind of awaking, so complete in an instant, caused by the pressure of
35 some troublesome thought upon the dreaming brain. 'Suppose I should not succeed
henceforth? Suppose I could never get mom than this poor hundred pounds for one of
the long books which cost me to much labour? I shall perhaps have children to support:
and Amy—how would Amy bear poverty?'
He knew what poverty means. Tile chilling of brain and heart, the unnerving of the
40 hands, the slow gathering about one of fear and shame and impotent wrath, the dread
feeling of helplessness, of the world's base indifference. Poverty! Poverty!

1. Which choice best describes a major theme A. The narrator describes in detail her
of the passage? youthful enthusiasm as a major
A. The internal battle between true love motivating factor.
and self-doubt B. The narrator asks a hypothetical
B. The unequivocal joy of wedded marital question that is immediately refuted.
bliss C. The narrator applauds her decision as
C. The destructive power of encroaching a reflection of an inner strength that is
poverty unparalleled.
D. The fear of never reaching one’s D. The narrator stresses her sincerity as
ultimate potential proof of a dedication fostered by her
maidenhood.
2. According to the narrator, when the
woman he loved learned of his feelings for 6. What function does the last paragraph
her, she (lines 39–41) serve in the passage as a
A. Pledged her undying affection in whole?
return. A. It expands upon the overall theme of
B. dedicated herself to her maidenhood. the narrator’s never- ending love for
C. reconsidered her prior refusal of his Amy.
advances. B. It lists the long-term effects that
D. wrote her own book of poetry. poverty could have on the narrator’s
relationship.
3. Which choice provides the best evidence C. It provides evidence that the narrator
for the answer to the previous question? cannot support his wife by revealing
A. Lines 5 (“Though slenderly...strong”) his hysteria.
B. Lines 8 (“She spoke...eternity”) D. It intensifies the narrator’s growing
C. Lines 17 (“Amy wished...wife”) concerns about his ability to support
D. Lines 21–23 (“Amy replied...fire”) his marriage.

4. In describing the relationship between 7. Which choice provides the best evidence
Amy and the narrator, the narrator for the answer to the previous question?
highlights a distinction between Amy’s A. Lines 3–4 (“He thought...hand”)
A. desire for marriage and his readiness. B. Lines 7–8 (“In the...it”)
B. beauty and his common appeal. C. Lines 25–26 (“Latin and...hers”)
C. conventional education and his love of D. Lines 29–31 (“No, not...book”)
language.
D. distaste for spending and his ability to 8. As used in line 26, “wanting” most nearly
write. means
A. lacking.
5. Which statement best describes a B. requesting.
technique used to represent Amy’s desire C. pleasing.
to marry the narrator? D. desiring.
B E R A N I B E R J U A N G | 111
9. As compared with his love letters, the 10. The narrator uses the phrase “what
narrator’s book writing is portrayed as happiness lurked in embryo” (lines 31–32)
being to present the publishers as
A. agreeable. A. kind.
B. stalled. B. wealthy.
C. fiery. C. influential.
D. imaginative. D. foolish.

Questions 11–19 are based on the following passage and supplementary material.

Tuition Reimbursement: A Mutual Benefit


All employers want a well-educated workforce, and one way to accomplish that goal is to
provide tuition assistance benefits to employees. Tuition assistance programs are (11)
commonplace; a 2013 study showed that 61% of U.S. employers offered undergraduate tuition
assistance as a benefit—but their goals and guidelines vary widely. Companies that offer tuition
assistance (12) see it as a benefit both to employees and to the company. Whether employees are
taking classes that are related to their careers or not, tuition assistance programs result in improved
morale,(13) also better job performance, and retention.

11. 13.
A. NO CHANGE A. NO CHANGE
B. commonplace, B. job performance,
C. commonplace C. job performance too,
D. commonplace— D. additionally job performance,
12.
A. NO CHANGE
B. seeing
C. have seen
D. sees

Those who are skeptical of tuition assistance programs argue that (14) the programs cost a lot
and provide very little return on the investment. Some firms have instituted a requirement that
employees receiving tuition benefits must stay with the company for a specific amount of time
after completing their educations. (15) In any case, such clauses are hard to enforce, and research
shows that they aren’t necessary. Over 80% of workers who receive tuition benefits from their
employers feel an increased sense of loyalty stemming from the investment, and they are in fact
less likely to (16) leave than the average employee is.

14. Which choice best establishes the 15.


argument that follows: A. NO CHANGE
A. NO CHANGE B. Consequently,
B. companies should place restrictions on C. However,
the types of courses employees can be D. Additionally,
reimbursed for.
C. taking classes while working spreads 16.
employees too thin, resulting in lower A. NO CHANGE
productivity. B. leave;
D. an employee may use the benefit to C. leave,
seek a position at a different company. D. leave

112 | I U P I T B
Increased employee loyalty is not the only way that employers benefit from offering tuition
assistance. The programs can also be powerful recruiting tools. Employers from the U.S. Armed
Forces to Starbucks are able to attract workers who are interested in earning an education while
they earn money, without having to (17) take out loans. Most companies require employees to
earn a minimum grade in their classes, but student employees often find it easier to maintain their
grades when they feel a responsibility to the company paying for their education, not just to
themselves. Managers can also use tuition assistance programs to evaluate their employees. If an
employee takes advantage of the optional benefit, the thinking (18) goes then he or she is likely to
be a highly motivated and productive worker.
17. 18.
A. NO CHANGE A. NO CHANGE
B. borrow money through educational B. goes, then he
loan programs. C. goes; then he
C. go into debt by financing their D. goes. Then he
educations with student loans.
D. take on the risky proposition of
borrowing loan money that would
leave them with a significant debt
burden.

Some companies are increasing the value of their investment in tuition assistance programs by
restricting the benefit to classes that will provide their employees with necessary or helpful job
skills. (19) In addition, some employers have even gone so far as to create custom degree or
certification programs. The figure shows the ways in which a company can leverage its tuition
assistance program to make a killing on the company’s investment in the program. As shown in the
figure, the most effective programs will start with a strategy to manage the workforce necessary
for a particular field, which will lead to a secure workforce, will provide a reliable source of talent,
and will ultimately make talent development proceed productively.
Benefits of Employee Tuition Assistance Programs

19.
A. NO CHANGE
B. By contrast,
C. In other words,
D. DELETE the underlined portion and begin the sentence with a capital letter.

Questions 20-25 are based on the following passage and supplementary material.

The ---(20)--- inhabitants of the meadows ---(21)--- barely living human skeletons, with sunken,
fevered ---(22)--- and clothed in filthy and ---(23)--- rags. They are the refugees who have ---(24)---
to flee deeper into Thailand, and have ---(25)--- instead to stay close to their Cambodian homeland,
inspite of all the dangers and hardships they face everyday.

B E R A N I B E R J U A N G | 113
20. In each of the following passages, there are 23. In each of the following passages, there are
blanks each of which has been numbered. blanks each of which has been numbered.
Find out the appropriate words. Find out the appropriate words.
A. cheerful A. battered
B. erstwhile B. tattered
C. only C. cluttered
D. happy D. fluttered

21. In each of the following passages, there are 24. In each of the following passages, there are
blanks each of which has been numbered. blanks each of which has been numbered.
Find out the appropriate words. Find out the appropriate words.
A. being A. refused
B. are B. wanted
C. were C. asked
D. have D. denied

22. In each of the following passages, there are 25. In each of the following passages, there are
blanks each of which has been numbered. blanks each of which has been numbered.
Find out the appropriate words. Find out the appropriate words.
A. hand A. chosen
B. feet B. needed
C. ears C. refused
D. eyes D. wondered

26. FIXED is to VARIABLE as SOLID is to: 29. ALOOF is to SOCIABLE as ORDINARY is to:
A. glass A. quiet
B. malleable B. hidden
C. hard C. splendid
D. rotate D. addition
E. None of these E. same

27. Assume that some paks are saks, all naks 30. All of the words in the box below are alike
are paks, and some zaks are kaks. in some way.
Therefore it makes sense that: entourage associates
A. no paks are naks followers supporters
B. all paks are naks Which of the following options could also
C. some kaks are naks be in the box?
D. some naks may be saks A. employer
E. all paks are kaks B. staff
C. enemies
28. Which of the following words has a similar D. competition
meaning to terminate: E. hierarchy
A. use-by
B. cease
C. uncontinue
D. maintain
E. discount
114 | I U P I T B
31. Red blood cells in which the malarial-fever B. The total time allotted to the average
parasite resides are eliminated from a cluster of consecutive television
person's body after 120 days. Because the commercials is decreasing.
parasite cannot travel to a new generation C. The average number of hours per day
of red blood cells, any fever that develops that people spend watching television
in a person more than 120 days after that is decreasing.
person has moved to a malaria-free region D. The average number of clusters of
is not due to the malarial parasite. consecutive commercials per hour of
Which is the following, if true, most television is increasing.
seriously weakens the conclusion above? E. The average number of television
A. The fever caused by the malarial commercials in a cluster of
parasite may resemble the fever consecutive commercials is increasing.
caused by flu viruses.
B. The anopheles mosquito, which is the 33. The number of people diagnosed as having
principal insect carrier of the malarial a certain intestinal disease has dropped
parasite, has been eradicated in many significantly in a rural county this year, as
parts of the world. compared to last year. Health officials
C. Many malarial symptoms other than attribute this decrease entirely to
the fever, which can be suppressed improved sanitary conditions at water-
with anti-malarial medication, can treatment plants, which made for cleaner
reappear within 120 days after the water this year and thus reduced the
medication is discontinued. incidence of the disease.
D. In some cases, the parasite that causes Which of the following, if true, would most
malarial fever travels to cells of the seriously weaken the health officials'
spleen, which are less frequently explanation for the lower incidence of the
eliminated from a person's body than disease?
are red blood cells. A. Many new water-treatment plants
E. In any region infested with malaria- have been built in the last five years in
carrying mosquitoes, there are the rural county.
individuals who appear to be immune B. Bottled spring water has not been
to malaria. consumed in significantly different
quantities by people diagnosed as
32. Fact 1: Television advertising is becoming having the intestinal disease, as
less effective: the proportion of brand compared to people who did not
names promoted on television that viewers contract the disease.
of the advertising can recall is slowly C. Because of a new diagnostic
decreasing. technique, many people who until this
Fact 2: Television viewers recall year would have been diagnosed as
commercials aired first or last in a cluster of having the intestinal disease are now
consecutive commercials far better than correctly diagnosed as suffering from
they recall commercials aired somewhere intestinal ulcers.
in the middle. D. Because of medical advances this year,
Fact 2 would be most likely to contribute to far fewer people who contract the
an explanation of fact 1 if which of the intestinal disease will develop severe
following were also true? cases of the disease.
A. The average television viewer E. The water in the rural county was
currently recalls fewer than half the brought up to the sanitary standards
brand names promoted in of the water in neighboring counties
commercials he or she saw. ten years ago.

B E R A N I B E R J U A N G | 115
34. The average normal infant born in the Historical costing allows contractors to
United States weighs between twelve and protect their profits by adding a
fourteen pounds at the age of three percentage increase, based on the current
months. Therefore, if a three-month-old rate of inflation, to the previous year's
child weighs only ten pounds, its weight contractual price.
gain has been below the United States Which of the following statements, if true,
average. is the best basis for a criticism of historical
Which of the following indicates a flaw in costing as an economically sound pricing
the reasoning above? method for military contracts?
A. Weight is only one measure of normal A. The government might continue to pay
infant development. for past inefficient use of funds.
B. Some three-month-old children weigh B. The rate of inflation has varied
as much as seventeen pounds. considerably over the past twenty
C. It is possible for a normal child to years.
weigh ten pounds at birth. C. The contractual price will be greatly
D. The phrase "below average" does not affected by the cost of materials used
necessarily mean insufficient. for the products.
E. Average weight gain is not the same as D. Many taxpayers question the amount
average weight. of money the government spends on
military contracts.
35. The price the government pays for E. The pricing method based on historical
standard weapons purchased from military costing might not encourage the
contractors is determined by a pricing development of innovative weapons.
method called "historical costing."

36. The circle has center O, the length of arc 38. 2𝑎𝑥 − 15 = 3(𝑥 + 5) + 5(𝑥 − 1)
̂ is 5𝜋, and 𝑥 = 100. What is the
𝐴𝐷𝐶 In the equation above, 𝑎 is a constant. If no
̂?
length of arc 𝐴𝐵𝐶 value of 𝑥 satisfies the equation, what is
the value of 𝑎 ?
A. 1
B. 2
C. 4
D. 8

A. 9𝜋 39. A certain homeowner uses a gas edger to


B. 13𝜋 clean up his lawn every time he mows. If
C. 18𝜋 the edger uses 160 milliliters of fuel each
13 time, what is the maximum number of
D. 𝜋
2
times the homeowner can edge his lawn
8 with 8 liters of fuel? (1 liter = 1,000
37. If = 60 what is the value of 𝑥 ? milliliters)
𝑥
A. 1,280 A. 5
B. 80 B. 50
C. 20 C. 100
D. 0.05 D. 1,000

116 | I U P I T B
40. (𝑎𝑥 + 3)(5𝑥 2 − 𝑏𝑥 + 4) = 20𝑥 3 − 45. 𝑦 ≥ 𝑥 + 2
9𝑥 2 − 2𝑥 + 12 2𝑥 + 3𝑦 ≤ 6
The equation above is true for all 𝑥, where In which of the following does the shaded
𝑎 and 𝑏 are constants. What is the value of region represent the solution set in the 𝑥𝑦-
𝑎𝑏 ? plane to the system of inequalities above?
A. 18
B. 20
C. 24
D. 40

41. 12𝑥 + 1 + 5
Which of the following is equivalent to the A.
expression above for 𝑥 > 0 ?
2𝑥+5
A.
2𝑥+1
2𝑥+6
B.
2𝑥+1
10𝑥+5
C.
2𝑥+1
10𝑥+6
D. B.
2𝑥+1

42. A system of three equations is graphed in


the 𝑥𝑦-plane below. How many solutions
does the system have?

C.

A. None
D.
B. One
C. Two
46. The graph of the function 𝑓 in the 𝑥𝑦-plane
D. Three
below is a parabola. Which of the following
𝑥 2𝑥
defines 𝑓 ?
43. =
𝑥−3 2
A. 0 and 2
B. 0 and 4
C. -4 and 4
D. 4

44. What is the set of all solutions to the


equation √𝑥 + 2 = −𝑥 ?
A. {−1, 2}
B. {−1} A. 𝑓 (𝑥) = 4(𝑥 − 3)2 + 1
C. {2} B. 𝑓 (𝑥) = 4(𝑥 + 3)2 + 1
C. 𝑓 (𝑥) = (𝑥 − 3)2 + 1
D. There are no solutions to the given
D. 𝑓 (𝑥) = 3(𝑥 + 3)2 + 1
equation

B E R A N I B E R J U A N G | 117
47. The table below shows the number of A. Few people who prefer working alone
students who chose to be graded on lab will be unhappy doing this task.
reports only or on lab reports and final B. Few people who do not prefer working
exams in Dr. Soper’s and Mr. Coelho’s alone will be happy doing this task.
physics classes. What fraction of the C. Less than 5% of people will be happy
students in Dr. Soper’s class chose to be doing this task if they do not work
graded on the lab report and final exam? alone
D. Less than 5% of people will be unhappy
doing this task if they work alone

Questions 51 and 52 refer to the following


information.
The scatterplot below shows the 𝑝𝐻 of seven
well water samples in West Texas with respect
3
a.
43
to the bicarbonate concentration in 𝑝𝑝𝑚 (parts
5 per million). The line of best fit is also shown.
b.
43
3
c.
20
3
d.
5

48. (4 − 𝑎2 ) − (2𝑎2 − 6)
Which of the following expressions is
equivalent to the one above?
A. 𝑎2 – 2
B. 𝑎2 + 10
C. −3𝑎2 – 2
D. −3𝑎2 + 10 51. According to the scatterplot, which of the
following statements about the
49. The ordered pair (3, −1) satisfies which of
relationship between a well’s pH and its
the following inequalities?
bicarbonate concentration is true?
I. 𝑥 + 3𝑦 > 0
A. A well with half as much bicarbonate
II. 2𝑥 + 3𝑦 > 2
as another well will have a pH twice
III. 𝑥 + 𝑦 < 0
that of the other well.
A. I only
B. Wells that have more bicarbonate
B. II only
tend to have higher 𝑝𝐻.
C. I and III only
C. Wells that have more bicarbonate
D. II and III only
tend to have lower 𝑝𝐻.
D. The bicarbonate concentration of the
50. A psychology student randomly selected
well water is unrelated to its 𝑝𝐻.
300 people from a group of people who
indicated that they preferred to work
52. A new well is discovered in West Texas with
alone. Those 300 people were given a task
a bicarbonate concentration of 225 ppm.
to work on individually and then asked
According to the line of best fit, which of
whether they were happy or unhappy
the following best approximates the pH of
while doing the task. Of those surveyed, 5%
the well water?
stated they were unhappy while doing the
A. 7.1
task. Which of the following inferences can
B. 7.3
appropriately be drawn from this survey
C. 7.4
result?
D. 8.4
118 | I U P I T B
53. 25 = (𝑘𝑦 − 1)3 A. 15
In the equation above, 𝑦 = −2 is one B. 20
solution. If 𝑘 is a constant, what is a C. 30
possible value of 𝑘 ? D. 40
A. -13
B. -3 57. George recorded his distance from home
C. 0 over a five-hour period; his distance and
D. 5 time are shown in the graph below.
According to the graph, which of the
54. Andrew works out for 30 minutes every following is NOT true about the five-hour
other day. If he spends 35% of his workout period?
time one day waiting for the weight rack,
how many seconds of that day’s workout
did he spend waiting for the weight rack?
A. 630
B. 35
C. 21
D. 10.5

55. A backpacker is packing survival rations


that consist of granola bars and packets of
peanut butter. A granola bar has 470 food
calories, and a packet of peanut butter has
90 food calories. The backpacker makes the
survival rations using a total of 10 granola
bars and packets of peanut butter A. George’s distance from home
combined, and the granola bars and increased at a constant rate during the
packets of peanut butter have a total of first hour of the five-hour period.
1,660 food calories. Which of the following B. George’s distance from home reached
systems of equations can be used to its maximum during the first hour.
determine the number of granola bars, g, C. George remained a constant distance
and packets of peanut butter, p, that are in from his home for one hour.
the survival rations? D. George was moving further from his
a. 280(𝑔 + 𝑝) = 1,660 home for a longer period of time than
𝑔 − 𝑝 = 10 he was moving closer to his home.
b. 90𝑔 + 470𝑝 = 1,660
𝑔 − 𝑝 = 10 58. In the figure below, what is the value of 𝑎 ?
c. 90𝑔 + 470𝑝 = 1,660
𝑔 = 10 − 𝑝
d. 470𝑔 + 90𝑝 = 1,660
𝑔 = 10 − 𝑝

56. Ten floorboards with equal widths laid


down side-to-side cover a width of
approximately feet. At this rate, which of A. 40
the following is the closest to the number B. 60
of boards laid side-to-side needed to cover C. 100
a width of 32 feet? D. 130

B E R A N I B E R J U A N G | 119
59. If 8𝑥 – 8𝑦𝑧 + 2 = 74, what is the value 60. A chef trimmed fat off a steak and was left
of 𝑥 – 𝑦𝑧 ? with a steak weighing 8.80 ounces. If the
A. 2 weight of the fat was equal to 12 percent
B. 6 of the original weight, what was the
C. 9 original weight, in ounces, of the steak?
D. 16 A. 8.92
B. 9.20
C. 10.00
D. 11.20

61. 𝟔 < 𝟑𝒙 < 𝟏𝟐


𝟏𝟎 < 𝟓𝒚 < 𝟐𝟎
𝑸𝒖𝒂𝒏𝒕𝒊𝒕𝒚 𝑨 𝑸𝒖𝒂𝒏𝒕𝒊𝒕𝒚 𝑩
𝑥 𝑦
A. Quantity A is greater.
B. Quantity B is greater.
C. The two quantities are equal.
D. The relationship cannot be determined from the information given.

62. The length of a rectangular canvas is increased by 𝒙 percent, and the width of the
canvas is decreased by 𝒙 percent.
𝑸𝒖𝒂𝒏𝒕𝒊𝒕𝒚 𝑨 𝑸𝒖𝒂𝒏𝒕𝒊𝒕𝒚 𝑩
𝑇ℎ𝑒 𝑎𝑟𝑒𝑎 𝑜𝑓 𝑡ℎ𝑒 𝑛𝑒𝑤 𝑐𝑎𝑛𝑣𝑎𝑠 𝑇ℎ𝑒 𝑎𝑟𝑒𝑎 𝑜𝑓 𝑡ℎ𝑒 𝑛𝑒𝑤 𝑐𝑎𝑛𝑣𝑎𝑠
𝑖𝑓 𝑥 = 20 𝑖𝑓 𝑥 = 40
A. Quantity A is greater.
B. Quantity B is greater.
C. The two quantities are equal.
D. The relationship cannot be determined from the information given.

63. |𝒓𝟑 | = 𝟐𝟕
|𝒓𝟐 − 𝟗| = 𝟎

𝑸𝒖𝒂𝒏𝒕𝒊𝒕𝒚 𝑨 𝑸𝒖𝒂𝒏𝒕𝒊𝒕𝒚 𝑩
𝑟 −3
A. Quantity A is greater.
B. Quantity B is greater.
C. The two quantities are equal.
D. The relationship cannot be determined from the information given.

64. 𝑸𝒖𝒂𝒏𝒕𝒊𝒕𝒚 𝑨 𝑸𝒖𝒂𝒏𝒕𝒊𝒕𝒚 𝑩


𝑇ℎ𝑒 𝑛𝑢𝑚𝑏𝑒𝑟 𝑜𝑓 𝑚𝑖𝑙𝑒𝑠 𝑑𝑟𝑖𝑣𝑒𝑛 𝑎𝑡 𝑇ℎ𝑒 𝑠ℎ𝑜𝑟𝑡𝑒𝑠𝑡 𝑑𝑖𝑠𝑡𝑎𝑛𝑐𝑒 𝑏𝑒𝑡𝑤𝑒𝑒𝑛
50 𝑚𝑖𝑙𝑒𝑠 𝑝𝑒𝑟 ℎ𝑜𝑢𝑟 𝑓𝑜𝑟 2 ℎ𝑜𝑢𝑟𝑠 𝑡ℎ𝑒 𝑠𝑡𝑎𝑟𝑡𝑖𝑛𝑔 𝑝𝑜𝑖𝑛𝑡 𝑎𝑛𝑑 𝑡ℎ𝑒 𝑒𝑛𝑑𝑖𝑛𝑔
𝑝𝑜𝑖𝑛𝑡 𝑜𝑓 𝑎 𝑡𝑟𝑖𝑝 𝑖𝑓 𝑎 𝑣𝑒ℎ𝑖𝑐𝑙𝑒 𝑖𝑠 𝑑𝑟𝑖𝑣𝑒𝑛
60 𝑚𝑖𝑙𝑒𝑠 𝑛𝑜𝑟𝑡ℎ 𝑎𝑛𝑑 80 𝑚𝑖𝑙𝑒𝑠 𝑒𝑎𝑠𝑡
A. Quantity A is greater.
B. Quantity B is greater.
C. The two quantities are equal.
D. The relationship cannot be determined from the information given.

120 | I U P I T B
65. The area of equilateral triangle 𝑨𝑩𝑪 is
𝑸𝒖𝒂𝒏𝒕𝒊𝒕𝒚 𝑨 𝑸𝒖𝒂𝒏𝒕𝒊𝒕𝒚 𝑩
𝑇ℎ𝑒 𝑝𝑒𝑟𝑖𝑚𝑒𝑡𝑒𝑟 𝑜𝑓 ∆𝐴𝐵𝐶 32
A. Quantity A is greater.
B. Quantity B is greater.
C. The two quantities are equal.
D. The relationship cannot be determined from the information given.

66. Data Sufficiemcy: Is |𝑎| > |𝑏| ?


1 1
(1) >
(𝑎−𝑏) (𝑏−𝑎)
(2) 𝑎+𝑏 <0
A. Statement (1) ALONE is sufficient, but statement (2) alone is not sufficient.
B. Statement (2) ALONE is sufficient, but statement (1) alone is not sufficient.
C. BOTH statements TOGETHER are sufficient, but NEITHER statement ALONE is sufficient.
D. EACH statement ALONE is sufficient.
E. Statements (1) and (2) TOGETHER are NOT sufficient.

67. Data Sufficiency: Is 𝑎3 > 𝑎2 ?


1
(1) >𝑎
𝑎
(2) 𝑎 > 𝑎3
5

A. Statement (1) ALONE is sufficient, but statement (2) alone is not sufficient.
B. Statement (2) ALONE is sufficient, but statement (1) alone is not sufficient.
C. BOTH statements TOGETHER are sufficient, but NEITHER statement ALONE is sufficient.
D. EACH statement ALONE is sufficient.
E. Statements (1) and (2) TOGETHER are NOT sufficient.

68. Data Sufficiency: Is |𝑎| > 𝑎 ?


(1) 𝑎2 < 𝑎
𝑎 2
(2) ( ) > ( )
2 𝑎
A. Statement (1) ALONE is sufficient, but statement (2) alone is not sufficient.
B. Statement (2) ALONE is sufficient, but statement (1) alone is not sufficient.
C. BOTH statements TOGETHER are sufficient, but NEITHER statement ALONE is sufficient.
D. EACH statement ALONE is sufficient.
E. Statements (1) and (2) TOGETHER are NOT sufficient.

69. Data Sufficiency :Is ′𝑎′ positive?


(1) 𝑎 − 𝑏 > 0
(2) 2𝑎 − 𝑏 > 0
A. Statement (1) ALONE is sufficient, but statement (2) alone is not sufficient.
B. Statement (2) ALONE is sufficient, but statement (1) alone is not sufficient.
C. BOTH statements TOGETHER are sufficient, but NEITHER statement ALONE is sufficient.
D. EACH statement ALONE is sufficient.
E. Statements (1) and (2) TOGETHER are NOT sufficient.

70. Gisela would owe $15,500 in taxes each year if she were not eligible for any tax deductions.
This year, Gisela is eligible for tax deductions that reduce the amount of taxes she owes by
$2,325.00. If these tax deductions reduce the taxes Gisela owes this year by 𝑑%, what is the
value of 𝑑?

B E R A N I B E R J U A N G | 121
3 1
71. 𝑥 − 𝑦 = 12
4 2
𝑎𝑥 − 𝑏𝑦 = 9
The system of equations above has no solutions. If 𝑎 and 𝑏 are constants, what is the value of
𝑎
?
𝑏

Questions 72 and 73 refer to the following information.

The table above shows the number of international tourist arrivals, rounded to the nearest tenth
of a million, to the top nine tourist destinations in both 2012 and 2013.

72. Based on the information given in the table, how much greater, in millions, was the median
number of international tourist arrivals to the top nine tourist destinations in 2013 than the
median number in 2012, to the nearest tenth of a million?

73. The number of international tourist arrivals in Russia in 2012 was 13.5% greater than in 2011.
The number of international tourist arrivals in Russia was 𝑘 million more in 2012 than in 2011.
What is the value of 𝑘 to the nearest integer?

74. What is the volume, in cubic centimeters, of a right rectangular prism that has a length of 4
centimeters, a width of 9 centimeters, and a height of 10 centimeters?

75. 4𝑥 + 2 = 4
If 𝑥 satisfies the equation above, what is the value of 2𝑥 + 1 ?

122 | I U P I T B
SET-7

Questions 1–10 are based on the following passage and supplementary material.

This passage is excerpted from Dan Glass, “What Happens if GPS Fails?” © 2016 by The Atlantic.

Line Despite its name, the Global Positioning System is not about maps; it's about time.
Each satellite in the constellation (24 are needed, plus the U.S. has several spares) has
multiple atomic docks on board, synchronized with each other and to Coordinated
Universal Time (UTC)—the time standard used across the world—down to the
5 nanosecond. The satellites continually broadcast their time and position information
down to Earth, where GPS receivers in equipment from iPhones to automated tractors
acquire signals and use the minuscule differences in their arrival time to determine an
exact position.
While GPS was initially conceived to aid navigation, globally synchronized time is
10 now a much more critical function oldie system. Telecom networks rely on GPS clocks
to keep cell towers synchronized so calls can be passed between them. Many electrical
power grids use the docks in equipment that fine-tunes current flow in overloaded
networks. The finance sector uses GPS-derived timing systems to timestamp ATM,
credit card, and high-speed market transactions. Computer network synchronization,
15 digital television and radios Doppler radar weather reporting, seismic monitoring, even
multi-camera sequencing for film production—GPS clocks have a hand in all.
What if all these flying clock radios were wiped out, and everything on the ground
started blinking 12:00? According to Mike Lombardi, a meteorologist at the National
Institute for Standards and Technology, "Nobody knows exactly what would happen."
20 Since so many of these technologies were designed specifically with GPS in mind, the
unsettling truth, he says, is "there's no backup.”
The bulk of a more promising, comprehensive backup system already exists, right
here on the ground. After the sextant but before GPS, navigators around the world used
Long Range Aids to Navigation, or "LORAN:' a terrestrial system of transmitters and
25 receiving equipment first developed during WWII. By the mid-1990s, Loran "tower
chains" provided coverage for North America, Europe, and other regions in the Northern
Hemisphere. Its use declined in favor of the much finer accuracy of GPS after it became
available for civil use in 1995, but the U.S. Coast Guard continued working on an
improved system using the existing infrastructure. If adopted, "Enhanced" LORAN, or
B E R A N I B E R J U A N G | 123
30 eLoran, could provide positioning accuracy comparable to GPS. Broadcast at hundreds
of thousands of watts, the signal is virtually un-jammable, and unlike GPS, can even be
received indoors, underwater, and in urban or natural canyons. It also turns out that
eLoran can provide a UTC time signal with sub-microsecond time resolution across a
large geographical area.
35 The technology is available—the Coast Guard demonstrated a working prototype last
year—so why isn't America using it? John Garamendi, a California congressman, asked
this question at a July 2015 congressional hearing on the Federal Radionavigation Plan,
the nation's primary planning document for position, navigation, and timing (PNT).
"There are two kinds of time he opened, "real time ... and then federal time, which
40 seems to be the forever Mile. The eLoran system was identified as a backup 15 years
ago, and here we are, federal time, not yet done'
Why is the sense of urgency among decision-makers so out of sync? Could some of
it be similar to why people delay backing up their computers even though they've been
telling themselves to for weeks? How do we decide, when presented a risk with
45 unknown odds, when it's time to sacrifice time and resources to prevent it?
Now is a critically important time to answer that question, as the world has actually
been given odds on another, even more catastrophic risk than GPS failure: destruction
of the electrical power infrastructure itself. On July 23, 2012, a billion-ton cloud of
electrified gases blasted off the far side of the sun at over six million miles per hour.
50 According to professor Daniel Baker at University of Colorado, this corona) mass
ejection (CME) "was in all respects at least as strong as the 1859 Carrington Event,"
referring to the strongest solar storm ever recorded, which set fire to telegraph stations
and caused auroras down to Cuba. As was widely reported two years ago, if the 2012
CME had occurred one week later, it would have hit Earth.

124 | I U P I T B
1. The main purpose of the passage is to 7. According to John Garamendi, the reason
A. present a problem with a current America isn’t using the e-Loran system
technology and highlight a potential even though the technology is available is
solution. that
B. provide an overview of how clocks and A. the system doesn’t use real time.
satellites determine distance and B. government approval takes longer
location. than the development of the
C. analyze the negative impacts of certain technology.
technologies across various industries. C. federal decision-makers are scared to
D. praise developers for their ability to change systems.
answer the hard questions. D. solar storms threaten the system.

2. As used in line 9, “conceived” most nearly 8. Which choice provides the best evidence
means for the answer to the previous question?
A. designed. A. Lines 30–32 (“Broadcast at...canyons”)
B. understood B. Lines 40–41 (“The eLoran...done”)
C. absorbed. C. Lines 44–45 (“How do...it”)
D. accepted. D. Lines 46–48 (“Now is...itself”)

3. The primary purpose of the question in 9. According to figure 1, which group is


lines 17–18 (“What if...12:00?”) is to closest in percentage to the percentage of
A. introduce a problem. users 18–29 in May 2011 who got location-
B. correct a misconception. based information on their smartphones?
C. reconsider a perspective. A. Ages 50+ in Feb 2012
D. undermine an idea. B. Ages 50+ in May 2011
C. Ages 30-49 in Feb 2012
4. Which of the following best characterizes D. Ages 18-29 in Feb 2012
Lombardi’s attitude toward “flying clock
radios” (line 17)? 10. Which statement is supported by figure 2?
A. He is confident about their ability to A. Less than a third of North American
handle a multitude of tasks. GPS revenue comes from the
B. He is concerned about how they will Automotive industry.
interact with the eLoran systems. B. GPS revenues for Surveying/Mapping
C. He is annoyed that no one knows are less than GPS revenues for
exactly how they work. Precision Agriculture.
D. He is worried that they have no C. GPS devices are less important for the
replacement systems in case of Aviation industry than they are for the
emergency. Marine industry.
D. The Automotive and Marine industries
5. Which choice provides the best evidence make up a greater percentage of North
for the answer to the previous question? American GPS revenues than the rest
A. Line 16 (“GPS clocks...all”) of the industries combined.
B. Line 19 (“Nobody knows...happen”)
C. Lines 20–21 (“Since so...backup”)
D. Lines 22–23 (“The bulk...ground”)

6. As used in line 22, “bulk” most nearly


means
A. dimensions.
B. mass.
C. majority.
D. totality.

B E R A N I B E R J U A N G | 125
Questions 11–20 are based on the following passage.

The Changing Face of Fast Food

Over the last two decades there (11) are a number of studies that have shown a strong
correlation between the rise of fast food restaurants in the United States and the rise of obesity.
Partly in response to this epidemic, a new trend has taken off in the restaurant business: “fast
casual” food. It’s difficult to precisely define what fast casual is, but it generally refers to restaurants
that offer the traditional quick preparation and counter service of fast food at a slightly higher price
point, with a focus on fresh, high-quality ingredients. Though many fast casual restaurants have
menus that focus on a particular healthy type of food, such as salads or vegan foods, others offer
the traditional burgers- and-fries fast food fare. But even fast casual burger joints (12) are often
still cheaper than going to a sit-down restaurant.
It is generally agreed that Chipotle Mexican Grill started the fast casual trend when it opened
its first restaurant in Denver in 1993. The chain grew (13) quickly driven by customers who were
attracted to its fresh menu and sustainably sourced ingredients. Many within the fast food industry
took notice—McDonald’s even became a major investor—and the fast casual movement was
born.(14)

11. 14. At this point, the writer is considering


A. NO CHANGE adding the following sentence. It is
B. have been somewhat unclear where the term fast
C. will be casual came from —two different people
D. had been are generally credited with coining the
term, but it was trademarked in 1995 by
12. Horatio Lonsdale- Hands.
A. NO CHANGE Should the writer make this addition here?
B. quickly: driven A. Yes, because it provides important
C. quickly, driven background for the information
D. quickly; driven presented in the following paragraph.
B. Yes, because it clarifies the origins of
13. Which choice most effectively reinforces the term “fast casual.”
the definition of fast casual given earlier in C. No, because it blurs the paragraph’s
the paragraph? focus on the beginnings of the fast
A. NO CHANGE casual movement.
B. emphasize the higher quality and D. No, because it distracts from the
freshness of their ingredients. paragraph’s main idea by introducing a
C. serve salads as well as burgers and figure whose role in the movement is
fries. unclear.
D. may contribute to the obesity
epidemic.

Fast casual’s focus on high-quality ingredients is working. During the 2007–2009 recession,
spending in the restaurant business declined for two (15) unbending years. (16) Infact, during that
same period, fast casual business grew by double digits. Traditional fast food restaurants are
changing in response. McDonald’s, which is now a competitor of (17) Chipotle, having sold its
interest in the burrito business in 2006, announced recently that it is making the switch from frozen
burger patties to fresh meat for its Quarter Pounders at most of its restaurants. This comes after
the company recently eliminated high fructose corn syrup from (18) they’re buns and announced
a plan to transition to using only eggs from cage-free chickens.

126 | I U P I T B
15. 17.
A. NO CHANGE A. NO CHANGE
B. Linear B. Chipotle having sold its interest in the
C. Even burrito business in 2006
D. straight C. Chipotle, having sold its interest in the
burrito business in 2006
16. D. Chipotle having sold its interest, in the
A. NO CHANGE burrito business in 2006,
B. Unsurprisingly, 18.
C. In other words, A. NO CHANGE
D. In contrast, B. their
C. it’s
D. its

While the move to cage-free eggs has the potential to make a big impact on how chicken farms
are run, it does nothing to address one of the bigger problems of fast food: (19) therefore, its lack
of nutritional value. Critics also (20) question whether increasingly fast casual restaurants are
actually healthy? The typical meal from Chipotle, for example, contains over 1,000 calories and a
full-day’s allowance of sodium. But change is slowly coming. The pressure that fast casual
restaurants are putting on their more traditional counterparts as well as the growing number that
truly do serve healthy food are indications that consumers are paying more attention to what they
put in their mouths and that the restaurant industry is responding.

19. C. question whether increasingly fast


A. NO CHANGE casual restaurants are actually
B. indeed, healthy.
C. for instance, D. increasingly question whether fast
D. specifically, casual restaurants are actually
20. healthy.
A. NO CHANGE
B. increasingly question whether fast
casual restaurants are actually
healthy?

Questions 21-25 are based on the following passage.

Interviews are very important for all of us. We must face interviews with confidence. There is ---
(21)--- any person who at one time or the other has not ---(22)--- an interview situation. While ---
(23)--- are a day to day ---(24)--- for people who ---(25)--- with others in their daily chores.

21. In each of the following passages, there are A. faced


blanks each of which has been numbered. B. measured
Find out the appropriate words. C. seen
A. possibly D. known
B. probably
C. hardly 23. In each of the following passages, there are
D. perhaps blanks each of which has been numbered.
Find out the appropriate words .
22. In each of the following passages, there are A. discussions
blanks each of which has been numbered. B. interviews
Find out the appropriate words. C. seminars
D. meetings
B E R A N I B E R J U A N G | 127
24. In each of the following passages, there are 25. In each of the following passages, there are
blanks each of which has been numbered. blanks each of which has been numbered.
Find out the appropriate words. Find out the appropriate words.
A. experience A. concern
B. happening B. react
C. feeling C. speak
D. possibility D. interact

26. Which of the following words has a similar 30. On another planet martians would say
meaning to pardon: “enb litpa penta” when saying to their
A. exit animals “good animal behaviour”. When
B. leave they got home from work they would ask
C. change “Ift enb penta?”, which means “was your
D. unfavourable behaviour good?”. The animals would reply
E. exonerate either, “Ift penta enb!” (“I showed good
behaviour!”) or, “Ift penta anb!”
27. Assume that all kibs are bics, some bics are If anb symbolises the opposite of enb, what
vics, and some tics are dups. does “Ift penta anb” mean?
Therefore it makes sense that: A. I showed bad behaviour
A. all kibs are vics B. I showed difficult behaviour
B. some bics are kibs C. C:I showed fun behaviour
C. all tics are bics D. I showed good behaviour
D. all vics are kibs E. I showed happy behaviour
E. some kibs are bics
31. Some who favor putting governmental
28. Which of the following is not similar in enterprises into private hands suggest that
meaning to blunder: conservation objectives would in general
A. inaccuracy be better served if private environmental
B. gaffe groups were put in charge of operating and
C. mistake financing the national park system, which is
D. error now run by the government.
E. detour Which of the following, assuming that it is
a realistic possibility, argues most strongly
29. All of the words in the box below are alike against the suggestion above?
in some way. A. Those seeking to abolish all
unarranged ad lib restrictions on exploiting the natural
Impulsive ad hoc resources of the parks might join the
Which of the following options could not go private environmental groups as
in the box? members and eventually take over
A. unprepared their leadership.
B. practice B. Private environmental groups might
C. impromptu not always agree on the best ways to
D. spontaneous achieve conservation objectives.
E. improvised C. If they wished to extend the park
system, the private environmental
groups might have to seek

128 | I U P I T B
contributions from major donors and some of the income urban and suburban
general public. households use for food and shelter can be
D. There might be competition among used by rural households for other needs.
private environmental groups for Which of the following inferences is best
control of certain park areas. supported by the statement made above?
E. Some endangered species, such as the A. The average rural household includes
California condor, might die out more people than does the average
despite the best efforts of the private urban or suburban household.
environmental groups, even if those B. Rural households have lower food and
groups are not hampered by housing costs than do either urban or
insufficient resources. suburban households.
C. Suburban households generally have
32. A recent spate of launching and operating more purchasing power than do either
mishaps with television satellites led to a rural or urban households.
corresponding surge in claims against D. The median income of urban and
companies underwriting satellite suburban households is generally
insurance. As a result, insurance premiums higher than that of rural households.
shot up, making satellites more expensive E. All three types of households spend
to launch and operate. This, in turn, has more of their income on food and
added to the pressure to squeeze more housing than on all other purchases
performance out of currently operating combined.
satellites.
Which of the following, if true, taken 34. In 1985 state border colleges in Texas lost
together with the information above, best the enrollment of more than half, on
supports the conclusion that the cost of average, of the Mexican nationals they had
television satellites will continue to previously served each year. Teaching
increase? faculties have alleged that this extreme
A. Since the risk to insurers of satellites is drop resulted from a rise in tuition for
spread over relatively few units, international and out-of-state students
insurance premiums are necessarily from $ 40 to $ 120 per credit hour.
very high. Which of the following, if feasible, offers
B. When satellites reach orbit and then the best prospects for alleviating the
fail, the causes of failure are generally problem of the drop in enrollment of
impossible to pinpoint with Mexican nationals as the teaching faculties
confidence. assessed it?
C. The greater the performance demands A. Providing grants-in-aid to Mexican
placed on satellites, the more nationals to study in Mexican
frequently those satellites break universities.
down. B. Allowing Mexican nationals to study in
D. Most satellites are produced in such Texas border colleges and to pay in-
small numbers that no economies of state tuition rates, which are the same
scale can be realized. as the previous international rate
E. Since many satellites are built by C. Reemphasizing the goals and mission
unwieldy international consortia, of the Texas state border colleges as
inefficiencies are inevitable. serving both in-state students and
Mexican nationals
33. Rural households have more purchasing D. Increasing the financial resources of
power than do urban or suburban Texas colleges by raising the tuition for
households at the same income level, since

B E R A N I B E R J U A N G | 129
in-state students attending state A. The percentage of minority and
institutions women workers in business has
E. Offering career counseling for those increased more slowly than many
Mexican nationals who graduate from minority and women's groups would
state border colleges and intend to prefer.
return to Mexico B. Those businesses with the highest
percentages of minority and women
35. Affirmative action is good business. So workers are those that have been the
asserted the National Association of most innovative and profitable.
Manufacturers while urging retention of an C. Disposable income has been rising as
executive order requiring some federal fast among minorities and women as
contractors to set numerical goals for among the population as a whole.
hiring minorities and women. "Diversity in D. The biggest growth in sales in the
work force participation has produced new manufacturing sector has come in
ideas in management, product industries that market the most
development, and marketing," the innovative products.
association claimed. E. Recent improvements in management
The association's argument as it is practices have allowed many
presented in the passage above would be manufacturers to experience
most strengthened if which of the enormous gains in worker
following were true? productivity.

36. 𝑦 = −75𝑥 + 5,000


The equation above models the amount of
money 𝑦, in dollars, remaining in Bo’s bank
account 𝑥 days after the start of the fall
semester. The amount of money in Bo’s
bank account is based on the money he
earned over the summer and how much he
spends per day during the fall semester. A. $200
When the equation is graphed in the 𝑥𝑦- B. $500
plane, what does the slope of the graph C. $900
represent in terms of the model? D. $1,200
A. The total amount in Bo’s bank account
𝑥
B. Daily spending of $5,000 38. If = 4and 𝑥 + 𝑦 = 32, what is the
3
C. Daily spending of $75 value of 𝑥 – 𝑦 ?
D. The amount of money Bo earned over A. −24
the summer B. −8
C. 12
37. The scatterplot below shows data for ten D. 32
accounts opened by a company, along with 39. What is the value of 𝑓(−2) if (𝑥) =
the line of best fit. For the account that 𝑥 2 +4𝑥−8
contains the least amount of money, which ?
𝑥−2
of the following is closest to the difference A. −3
of the actual amount and the amount B. −1
predicted by the line of best fit? C. 1
D. 3

130 | I U P I T B
5𝑤
40. The scatterplot below shows the height in C.
4ℎ2
centimeters for both the drop and bounce 5𝑤+100
D.
of eight different balls of the same type. 4ℎ2

The line of best fit for the data is also


shown. According to the line of best fit, 43. The number of bacteria colonies ℎ hours
which of the following is closest to the after the beginning of an experiment is
predicted increase in bounce height, in given by the function 𝐶(ℎ) = 3ℎ – 2ℎ +
centimeters, for every increase of 100 20. What does the number 20 represent in
centimeters in drop height? the function?
A. The final rate of growth, in colonies per
hour
B. The initial rate of growth, in colonies
per hour
C. One less than the initial number of
bacteria colonies
D. One more than the final number of
bacteria colonies

44. The World Bank measures the amount of


A. 25 land devoted to agriculture among all 196
B. 20 countries in the world. The results from 9
C. 15 of the countries are given in the table
D. 10 above. The median percent of agricultural
land for all 196 countries is 34.95%.
Questions 41 and 42 refer to the following Brazil 33.8%
information. Canada 7.2%
𝒘 Greenland 0.6%
𝑭𝒐𝒓𝒎𝒖𝒍𝒂 𝑨 ∶ 𝑩𝑴𝑰 = 𝟐
𝒉 Latvia 30.1%
𝟒𝒘 − 𝟏𝟎𝟎 Mexico 54.9%
𝑭𝒐𝒓𝒎𝒖𝒍𝒂 𝑩 ∶ 𝑩𝑴𝑰 =
𝟓 New Zealand 42.2%
The formulas above are used in nutrition to Russian 13.3%
estimate the body mass index BMI, in kilograms Federation
per square meter, of adults whose weight 𝑤 Turkey 50.1%
ranges between 50 and 100 kilograms and United States 44.6%
whose height ℎ is measured in meters. What is the difference between the median
percent of agricultural land for these 9
41. Based on Formula 𝐵, what is w in terms of countries and the median for all 196
BMI? countries?
A. 𝑤 = 5𝐵𝑀𝐼 + 25 A. 1.15%
B. 𝑤 = 5𝐵𝑀𝐼 – 25 B. 4.19%
5𝐵𝑀𝐼+100
C. 𝑤 = C. 9.65%
4
5𝐵𝑀𝐼−100 D. 19.95%
D. 𝑤 =
4

42. If both Formulas 𝐴 and 𝐵 give the same 45. For 𝑖 = √−1, which of the following
estimate for BMI, which of the following complex numbers is equivalent to (10𝑖 −
expressions is equivalent to 4𝑤 – 100 ? 4𝑖 2 ) − (7 − 3𝑖) ?
𝑤 A. −11 + 7𝑖
A. 2

5𝑤
B. −3 + 13𝑖
B.
ℎ2 C. 3 – 13𝑖
D. 11 – 7𝑖
B E R A N I B E R J U A N G | 131
10
46. To ship figurines, the figurines are placed in A. 100 − (𝑝 + 2𝑛)
9
a rectangular box and then small packing 10
B. 100 + (𝑝 + 2𝑛)
pellets are added. The base of the box has 9
10
an area of 4.4 𝑖𝑛2 , and the height of the C. 100 − (𝑝 − 2𝑛)
9
box is 6.5 𝑖𝑛. If the volume of the air in the 10
D. 100 + (𝑝 − 2𝑛)
9
box after the figures and pellets are added
is 8.0 𝑖𝑛3 , which of the following is closest
50. (𝑥 − 2)2 + (𝑦 + 5)2
to the combined volume of the figurines If a circle in the 𝑥𝑦-plane has the equation
and pellets in the box? above, which of the following does NOT lie
A. 1.9 𝑖𝑛3 on the exterior of the circle?
B. 20.6 𝑖𝑛3
A. (2, 1)
C. 28.6 𝑖𝑛3
B. (2, 5)
D. 117.84 𝑖𝑛3
C. (5, 2)
D. (-1, 1)
47. The economy of Argentina as measured by
its Gross Domestic Product (GDP) is
51. A farmer counted the number of peppers
shrinking at a rate of 2.6% per year. In
produced by a certain field in June and July.
2015, the GPD of Argentina was $630
The number counted for each month was
billion. Which of the following functions
recorded in the table below. The farmer
represents Argentina’s GPD, A, in billions of
estimates that the percent increase from
dollars, 𝑦 years since 2015?
June to July would be half the percent
A. 𝐴(𝑦) = 630 – (1 – 0.26) 𝑦
increase from July to August. How many
B. 𝐴(𝑦) = 630(1 – 0.26) 𝑦
peppers does the farmer expect the field to
C. 𝐴(𝑦) = 630 – (1 – 0.026) 𝑦
D. 𝐴(𝑦) = 630(1 – 0.026) 𝑦 produce in August?
Month Number of Peppers
Questions 48 and 49 refer to the following June 2,200
July 2,640
information.
A. 2,860
Coin Grams Drams
B. 2,904
Penny 2.50 1.41
C. 3,520
Nickel 5.00 2.82
Dime 2.25 1.27 D. 3,696

The table above gives the average weight, 52. Which of the following is equivalent to
expressed in both grams and drams, of three 10 + 2(𝑥 − 7) ?
types of modern U.S. coins. A. −14𝑥 + 10
B. 2𝑥 + 24
C. 2𝑥 + 3
48. If 𝑦 grams is equivalent to 𝑑 drams, of the
D. 2𝑥 – 4
following, which best represents the
relationship between 𝑦 and 𝑑? 𝑦
53. 3𝑥 − = 21
A. 𝑦 = 1.8𝑑 3
B. 𝑑 = 1.8𝑦 𝑥 =𝑦+7
C. 𝑦𝑑 = 1.8 Which ordered pair (𝑥, 𝑦) satisfies the
D. 𝑦 = 0.56𝑑 system of equations shown above?
A. (0, −7)
49. If a bag of coins weighing 225 grams is filled B. (4, 27)
with p pennies, 𝑛 nickels, and 𝑑 dimes, C. (7, 0)
which of the following expresses 𝑑 in terms D. (9, −16)
of 𝑛 and 𝑝 ?

132 | I U P I T B
54. 𝑥 2 − 8𝑥 + 5 A. -4 to 1
Which of the following is equivalent to the B. -2 to 1
expression above? C. -1 to 2
A. (𝑥 – 4)2 – 11 D. -1 to 4
B. (𝑥 – 4)2 + 11
C. (𝑥 + 4)2 – 11 58. Heinrich must buy at least 100 shares of
D. (𝑥 + 4)2 + 11 stock for his portfolio. The shares he buys
55. 𝑎 + 𝑏 = 15 will be from Stock 𝑋, which costs $22 per
The equation above relates the number of share and Stock 𝑌, which costs $35 per
hours, a, Kevin spends doing homework share. His budget for buying stock is no
each week and the number of hours he more than $4,500. He must buy at least 20
spends watching television each week. If shares of Stock 𝑋 and 15 shares of Stock 𝑌.
Kevin spends a total of 15 hours doing Which of the following represents the
homework and watching television each situation described if 𝑎 is the number of
week, what does the variable b represent? shares of Stock 𝑋 purchased and 𝑏 is the
A. The number of hours spent watching number of shares of Stock 𝑌 purchased?
television for each hour spent doing A. 22𝑎 + 35𝑏 ≤ 4,500
homework 𝑎 + 𝑏 ≥ 100
B. The number of hours spent watching 𝑎 ≤ 20
television each week 𝑏 ≤ 15
C. The number of hours spent doing B. 22𝑎 + 35𝑏 ≤ 4,500
homework each week 𝑎 + 𝑏 ≤ 100
D. The total number of hours spent doing 𝑎 ≤ 20
homework and watching television 𝑏 ≤ 15
each Week C. 22𝑎 + 35𝑏 ≤ 4,500
𝑎 + 𝑏 ≤ 100
56. Josephine purchases a computer for 𝑎 ≥ 20
$4,590. The computer decreases in value at 𝑏 ≥ 15
a constant rate for 9 years, after which it is D. 22𝑎 + 35𝑏 ≤ 4,500
considered not to have any monetary 𝑎 + 𝑏 ≥ 100
value. How much is the computer worth 6 𝑎 ≥ 20
𝑏 ≥ 15
years after it is purchased?
A. $1,530
59. Which of the following is equivalent to
B. $2,295
2𝑥(𝑥 2 − 3𝑥) ?
C. $3,060
A. −4𝑥 2
D. $4,080
B. 3𝑥 3 − 𝑥 2
C. 2𝑥 3 − 3𝑥
57. In the 𝑥𝑦-plane above, a point (not shown)
D. 2𝑥 3 − 6𝑥 2
with coordinates (𝑎, 𝑏) lies on the graph of
the linear function ℎ. If 𝑎 and 𝑏 are nonzero 𝑥+3
60. 𝑓(𝑥) =
integers, what is the ratio of 𝑏 to 𝑎? 2
For the function 𝑓 defined above, what is
the value of (−1) ?
A. −2
B. −1
C. 1
D. 2

B E R A N I B E R J U A N G | 133
61. 𝒙 < 𝟎 < 𝒚
𝑸𝒖𝒂𝒏𝒕𝒊𝒕𝒚 𝑨 𝑸𝒖𝒂𝒏𝒕𝒊𝒕𝒚 𝑩
−2(𝑥 + 𝑦) −𝑥𝑦
A. Quantity A is greater.
B. Quantity B is greater.
C. The two quantities are equal.
D. The relationship cannot be determined from the information given.

62. 𝒇(𝒙) = 𝒙𝟐 – 𝟏𝟎𝒙 + 𝟐𝟏


𝑸𝒖𝒂𝒏𝒕𝒊𝒕𝒚 𝑨 𝑸𝒖𝒂𝒏𝒕𝒊𝒕𝒚 𝑩
𝑓(3) 𝑓(7)
A. Quantity A is greater.
B. Quantity B is greater.
C. The two quantities are equal.
D. The relationship cannot be determined from the information given.

63.

𝑨𝑩 𝒊𝒔 𝒂 𝒅𝒊𝒂𝒎𝒆𝒕𝒆𝒓 𝒐𝒇 𝒕𝒉𝒆 𝒄𝒊𝒓𝒄𝒍𝒆. ∠𝑨𝑩𝑪 = 𝟐𝟎°


𝑸𝒖𝒂𝒏𝒕𝒊𝒕𝒚 𝑨 𝑸𝒖𝒂𝒏𝒕𝒊𝒕𝒚 𝑩
𝑇ℎ𝑒 𝑚𝑒𝑎𝑠𝑢𝑟𝑒 𝑜𝑓 ∠𝐵𝐴𝐶 70°
A. Quantity A is greater.
B. Quantity B is greater.
C. The two quantities are equal.
D. The relationship cannot be determined from the information given.

64. Five consecutive even integers have a sum of −20.


𝑸𝒖𝒂𝒏𝒕𝒊𝒕𝒚 𝑨 𝑸𝒖𝒂𝒏𝒕𝒊𝒕𝒚 𝑩
𝑇ℎ𝑒 𝑔𝑟𝑒𝑎𝑡𝑒𝑠𝑡 𝑜𝑓 𝑡ℎ𝑒 𝑓𝑖𝑣𝑒 0
𝑒𝑣𝑒𝑛 𝑖𝑛𝑡𝑒𝑔𝑒𝑟𝑠
A. Quantity A is greater.
B. Quantity B is greater
C. The two quantities are equal.
D. The relationship cannot be determined from the information given.

65. 𝒔 = ( 𝒕 + 𝒓 )𝟐
𝒔=𝟒
𝑸𝒖𝒂𝒏𝒕𝒊𝒕𝒚 𝑨 𝑸𝒖𝒂𝒏𝒕𝒊𝒕𝒚 𝑩
2 − 𝑟 𝑡
A. Quantity A is greater.
B. Quantity B is greater.
C. The two quantities are equal.
D. The relationship cannot be determined from the information given.

134 | I U P I T B
66. Data Sufficiency :Is 𝑎𝑛 > 𝑏 𝑛 ?
(1) 𝑎 > 𝑏
(2) 𝑎𝑏 < 0
A. Statement (1) ALONE is sufficient, but statement (2) alone is not sufficient.
B. Statement (2) ALONE is sufficient, but statement (1) alone is not sufficient.
C. BOTH statements TOGETHER are sufficient, but NEITHER statement ALONE is sufficient.
D. EACH statement ALONE is sufficient.
E. Statements (1) and (2) TOGETHER are NOT sufficient.

67. If 𝑚, 𝑠 are the average and standard deviation of integers 𝑎, 𝑏, 𝑐, and 𝑑, is 𝑠 > 0 ?
(1) 𝑚 > 𝑎
(2) 𝑎 + 𝑏 + 𝑐 + 𝑑 = 0
A. Statement (1) ALONE is sufficient, but statement (2) alone is not sufficient.
B. Statement (2) ALONE is sufficient, but statement (1) alone is not sufficient.
C. BOTH statements TOGETHER are sufficient, but NEITHER statement ALONE is sufficient.
D. EACH statement ALONE is sufficient.
E. Statements (1) and (2) TOGETHER are NOT sufficient.

68. Is ‘𝑏’ the median of 3 numbers , 𝑏, and 𝑐 ?


𝑏 𝑐
(1) =
𝑎 𝑏
(2) 𝑎𝑏 < 0
A. Statement (1) ALONE is sufficient, but statement (2) alone is not sufficient.
B. Statement (2) ALONE is sufficient, but statement (1) alone is not sufficient.
C. BOTH statements TOGETHER are sufficient, but NEITHER statement ALONE is sufficient.
D. EACH statement ALONE is sufficient.
E. Statements (1) and (2) TOGETHER are NOT sufficient.

69. Data Sufficiency : If a salesman received a commission of 3% of the sales that he has booked in
a month, what was the sales booked by the salesman in the month of November 2003?
(1) The sales booked by the salesman in the month of November 2003 minus salesman's
commission was $245,000
(2) The selling price of the sales booked by the salesman in the month of November 2003
were 125 percent of the original purchase price of $225,000.
A. Statement (1) ALONE is sufficient, but statement (2) alone is not sufficient.
B. Statement (2) ALONE is sufficient, but statement (1) alone is not sufficient.
C. BOTH statements TOGETHER are sufficient, but NEITHER statement ALONE is sufficient.
D. EACH statement ALONE is sufficient.
E. Statements (1) and (2) TOGETHER are NOT sufficient.

70. The figure below shows the complete graph of the function 𝑓 in the 𝑥𝑦-plane. The function 𝑔
(not shown) is defined by 𝑔(𝑥) = 𝑓 (𝑥) + 6. What is the maximum value of the function ?

B E R A N I B E R J U A N G | 135
71. Triangle 𝑃𝑄𝑅 has right angle 𝑄. If 𝑠𝑖𝑛 𝑅 = 45, what is the value of 𝑡𝑎𝑛 𝑃 ?

72. The graph of the linear function 𝑓 is shown in the 𝑥𝑦-plane below. The graph of the linear
function 𝑔 (not shown) is perpendicular to the graph of 𝑓 and passes through the point (1, 3).
What is the value of 𝑔(0)?

1
73. The raw score on a certain standardized test is determined by subtracting of the number of
4
incorrect answers from the number of correct answers. If a student answered 30 questions
and received a raw score of 20, how many questions did the student answer incorrectly?

74. One of the first diets to limit the intake of carbohydrates was prescribed by Dr. William Harvey
in 1862. This diet consisted of three meals a day containing equal amounts of protein per meal.
If protein contains 4 dietary calories per gram, and the diet consisted of 672 dietary calories of
protein per meal, how much protein, to the nearest ounce, was in each meal? (1 ounce is
approximately 28 grams.)

75. What is the slope of line 𝑙 shown in the 𝑥𝑦-plane below?

136 | I U P I T B
SET-8

Questions 1–10 are based on the following passage.

This passage is excerpted from Joshua Hammer, “The Dying of the Dead Sea.” © 2005 by
Smithsonian Magazine.

Line A refuge over the millennia for messiahs, martyrs and zealots. the Dead Sea region
abounds with sites sacred to Islam, Christianity and Judaism. Some Muslims believe that
Moses, whom they regard as a prophet, lies buried in a hilltop mosque just off the main
road from Jerusalem. Jesus Christ was said to have been baptized in the Jordan River after
5 traveling down to the Dead Sea from Galilee. And despite its name, the Dead Sea helps
support one of the world's most complex and vibrant ecosystems. Fed by fresh water
springs and aquifers, a half-dozen oases along the shore harbor scores of indigenous
species of plants, fish and mammals, including ibex and leopards. About 500 million birds
representing at least 300 species, including storks, pelicans, lesser spotted eagles, lesser
10 kestrels and honey buzzards, take refuge here during a biannual great migration from
Africa to Europe and back again. Ein Feshka., a lush expanse of tamarisk, papyri., oleander
and pools of crystal water, was used by the late king Hussein of Jordan as a private
playground in the 1950s and early '60s. But as the Dead Sea recedes, the springs that feed
the oases are moving along with it; many experts believe that Ein Feshka and other oases
15 could wither away within five years.
One reason for the decline, according to environmentalists and various government
officials, is a water policy on the part of Israel, Jordan and Syria that encourages
unrestricted agricultural use. From the first years of Israel's existence as a Jewish state, for
example, when collective farming transformed much of it into fertile vineyards and
20 vegetable fields, both Labor and Likud governments have bestowed generous water
subsidies on the nation's farmers. The results have been disastrous: today, agriculture
accounts for just 3 percent of Israel's gross national product and uses up to half of its fresh
water. Recently, Uri Sagie, chairman of Israel's national water company, told a conference
of Israeli farmers that a growing and irreversible gap between production and
25 consumption looms. The water sources are being depleted without the deficit being
restored; he warned. Jordan lavishes similar water subsidies on its farmers with similar
consequences: the kingdom takes about 71 billion gallons of water a year from the
Yarmouk River and channels it into the King Abdullah Canal, constructed by USAID in the
B E R A N I B E R J U A N G | 137
1970s to provide irrigation for the Jordan Valley; Syria takes out another 55 billion gallons.
30 So what is the answer?
Environmental activists say that one solution is to eliminate the water subsidies
altogether. "Unless water is priced at its real costs:' says Ra'ed Daoud, managing director
of ECO Consult, a water-use consulting firm, "there's no way you're going to reduce
agriculture:' But because the region's agricultural lobby is strong and the environmental
35 movement weak, says Daoud, there has been insufficient leverage for change. Israel's
water commissioner, Shimon Tal, recently spoke publicly about the need to reduce some
subsidies, but he admitted that it would be a long and difficult battle. Another approach
is to encourage alternate water sources. Friends of the Earth Middle East is part of a
coalition of 21 environmental groups that has developed proposals to conserve household
40 water use (about 133 billion gallons a year, as much as that used in agriculture) and to
regulate the amount that can be taken out of Israel's springs. In addition, the Israeli
government is promoting the building of wastewater treatment plants and desalination
facilities; the first large one on the Mediterranean was completed this past August. Over
the next five years, the government says, these facilities will provide as much as 106 billion
45 gallons of fresh water annually for agricultural and domestic consumption.
Friends of the Earth is also taking its message to the farmers themselves—
encouraging them to plant crops that use less water and spelling out the advantages of
renewed tourism in the area. "Israeli agriculture is incredibly mismanaged," Friends of the
Earth director Gidon Bromberg says. "The farmers here could be planting olives, flowers
50 and other crops like dates that don't require fresh water. They could be using treated
sewage water and allow fresh water to flow back into the Jordan River." Friends of the
Earth cites a Haifa University study that argues that current uses of the Jordan River make
no sense. "The potential tourism-dollar return of a healthy river and a healthy Dead Sea
outweighs the little return that agriculture offers: says Bromberg.

1. The primary purpose of the passage is to 4. The author mentions Ein Feshka primarily
A. discuss how the governments of Israel, in order to
Syria, and Jordan create water policy. A. describe a historical site in need of
B. describe the many species of plants, preservation.
fish, and mammals that live in the B. transition from a description of the
Dead Sea region. region to a discussion of a problem the
C. explain the consequences of region faces.
continuing current water policy in the C. identify a need for recreation areas in
Dead Sea region. the Dead Sea region.
D. discuss one cause of the decline of the D. indicate that environmental impacts
Dead Sea and outline a possible vary with different types of land use.
remedy.
5. What is the most likely reason the author
2. As used in line 7, “harbor” most nearly includes “both Labor and Likud
means governments” in lines 18–21
A. nourish. (“From...nation’s farmers”)?
B. shelter. A. To describe the level of cooperation
C. entertain. between political parties in the Israeli
D. consider. government
B. To emphasize the popularity of current
3. Which choice provides the best evidence water policy among all political parties
for the answer to the previous question? C. To indicate broad political support for
A. Lines 21–23 (“today...water”) water subsidies
B. Lines 34–35 (“But...change”) D. To criticize the politicization of natural
C. Lines 43–45 (“Over...consumption”) resources
D. Lines 48–49 (“Israeli...says”)
138 | I U P I T B
6. According to the passage, water laws favor C. Tourism may provide more benefit to
A. the tourism industry. the Dead Sea region’s economy than
B. government. agriculture currently does.
C. environmental activists. D. Officials in the tourism and agriculture
D. farmers. industries should work together to
create policy.
7. What can reasonably be inferred about the
agriculture industry in the Dead Sea 9. Which choice provides the best evidence
region? for the answer to the previous question?
A. Its use of water is disproportionate to A. Lines 11–13 (“Ein Feshka...’60s”)
its impact on the economy. B. Lines 41–43 (“the Israeli...facilities”)
B. It is an industry in decline. C. Lines 49–50 (“The farmers...water”)
C. The agriculture lobby is the most D. Lines 53–54 (“The
powerful influence on governments in potential...Bromberg”)
the Dead Sea region.
D. It will soon use more than 100 billion 10. What is the main idea of the final
gallons of water from treatment paragraph?
facilities A. Farmers can adopt new practices to
use less water and help maintain the
8. What does the author suggest about Dead Sea.
tourism? B. Farmers should stop planting crops
A. It was most vibrant in the mid-20th and focus on tourism.
century. C. Tourism will soon replace agriculture
B. Eco-tourism will be an important part as the main industry of the region.
of the future economy in the region. D. In the absence of policy change,
farmers are adjusting their practices to
conserve water.

Questions 11–21 are based on the following passage.

Did the Rite Cause a Riot?


It is commonly understood that at the premiere of Igor Stravinsky’s The Rite of Spring on May
29, 1913, the shocking nature of the ballet caused a riot to break out in the audience. The music
started with an unnaturally high bassoon solo, which elicited shouts and jeers from the audience
almost immediately. The furor only rose when the dancers from the Ballets Russes took the stage
and began the jerky, convulsive movements of Vaslav Nijinsky’s (11) choreography. The
choreography was shocking because it was not characteristic of the grace and fluidity typically
expected from ballet. A fight soon (12) broke up amidst the spectators, the orchestra was pelted
with vegetables thrown by outraged audience members, and the police were called to restore
order.
There are many theories (13) as to what caused the audience of The Rite of Spring to react so
strongly. It may have been the dissonant nature of Stravinsky’s music, with its constantly changing
rhythms and jarring percussion, or the purposefully awkward, ungraceful movements of the
dancers. The theme of the ballet (14) itself was a pagan ritual in which a virgin sacrifices herself to
the god of spring, may also have upset some viewers. The negative reception was summed up by a
(15) review, in Le Figaro, that proclaimed “We are sorry to see an artist such as M. Stravinsky involve
himself in this disconcerting adventure.”

11. Which choice most effectively combines C. choreography, because of it lacking in


the sentences at the underlined portion? conveyance of
A. choreography, which was D. choreography through which Nijinsky
uncharacteristic through its lack of tried not to convey
B. choreography that lacked
B E R A N I B E R J U A N G | 139
12. D. surrounding which Russian folk
A. NO CHANGE traditions Stravinsky drew his
B. brought up among inspiration from.
C. broke out among 14.
D. broke out between A. NO CHANGE
B. itself, being
13. Which choice best establishes the main C. itself
idea of the paragraph? D. itself,
A. NO CHANGE 15.
B. about why Stravinsky composed such A. NO CHANGE
a controversial piece of music. B. review, in Le Figaro that proclaimed,
C. regarding the role of the police in the C. review in Le Figaro that proclaimed,
uproar. D. review in Le Figaro, that proclaimed

Although several dozen eyewitness accounts of the evening exist, they are often contradictory
and do little (16) to sort of exactly what happened in the Theatre des Champs-Elysees that night. In
fact, if one makes a timeline of first-person accounts of the Rite’s premiere, descriptions of the level
of disruption and violence increase as the accounts get further away from the actual event. (17) On
the other hand, it’s likely that stories of the riot have gotten exaggerated over time.
16. 17.
A. NO CHANGE A. NO CHANGE
B. to sort out B. As a result,
C. for sort out C. At the same time,
D. for sort of D. In other words,

[1] Scholars have also recently begun to suspect that the uproar may have been planned. [2] It
is possible that he actually planted someone to start shouting, but more likely he simply set up an
expectation for controversy. [3] There was impassioned debate in Paris in the early years of the
20th century between traditionalists and modernists that was (18) exemplified by the debate over
the Eiffel Tower: modernists saw it as a mark of progress while traditionalists saw it as a
monstrosity.
[4] Diaghilev likely caused members of both groups (19) that believed that others would react
strongly to The Rite of Spring. [5] Serge Diaghilev, the founder of the Ballets Russes, was a savvy
entrepreneur who understood that any publicity is good publicity. [6] From such charged
expectations, it is (20) not unsurprising that controversy arose. (21)
18. The writer wants to add a supporting detail C. who believed
to explain the different views of the D. to believe
traditionalists and the modernists. Which
choice best accomplishes this goal? 20. Which choice most effectively signals that
A. NO CHANGE the result the author mentions was
B. still going on today: The Rite of Spring expected?
remains a controversial piece of music A. NO CHANGE
in many circles. B. Surprising
C. not limited to music: people also C. not surprising
argued over visual arts, architecture, D. unusual
and literature.
D. nothing new: there have always been 21. To make this paragraph most logical,
people who will be upset by sentence 5 should be placed
innovation of any kind, and there A. where it is now.
always will be. B. before sentence 2.
19. C. before sentence 3.
A. NO CHANGE D. before sentence 4.
B. Believing
140 | I U P I T B
Questions 22-25 are based on the following passage.

We are here ---(22)- -- with interview situations when one is a ---(23)--- aspiring for an ---(24)--- .
Such situations, not very ---(25)--- happen in one life, are ---(26)---- when the candidate likes to
come on top of the situation. This calls for careful study and preparation.

22. In each of the following passages, there are 24. In each of the following passages, there are
blanks each of which has been numbered. blanks each of which has been numbered.
Find out the appropriate words. Find out the appropriate words.
A. interested A. establishment
B. involved B. elevation
C. meeting C. estimation
D. concerned D. employment

23. In each of the following passages, there are 25. In each of the following passages, there are
blanks each of which has been numbered. blanks each of which has been numbered.
Find out the appropriate words. Find out the appropriate words.
A. student A. consequent
B. person B. important
C. candidate C. happy
D. beggar D. frequent

26. PAEDIATRICIAN is to CHILDREN as A. plane A


PODIATRIST is to: B. plane B
A. heart C. plane C
B. feet D. plane D
C. adult E. plane E
D. bones
E. None of these 29. Which of the following words is OPPOSITE
in meaning to commotion:
27. Which of the following words has a similar A. bustle
meaning to arrest: B. serene
A. apprehend C. movement
B. appreciate D. locomotion
C. chase E. miniature
D. enforce
E. sleep 30. All of the words in the box below are alike
in some way.
28. Five planes leave the airport at the same clothes garb outfit
time. Plane B arrives at the destination
Which of the following options could not go
before planes A & D, but after plane C.
in the box?
Plane E arrives before plane D but after
A. apparel
plane A.
B. jumper
Which plane is the last to arrive at the
C. attire
destination?
D. clothing
E. uniform

B E R A N I B E R J U A N G | 141
31. If the airspace around centrally located C. Many pilots of private planes would
airports were restricted to commercial rather buy radar equipment than be
airliners and only those private planes excluded from centrally located
equipped with radar, most of the private- airports.
plane traffic would be forced to sue D. The number of midair collisions that
outlying airfields. Such a reduction in the occur near centrally located airports
amount of private-plane traffic would has decreased in recent years.
reduce the risk of midair collision around E. Private planes not equipped with radar
the centrally located airports. systems cause a disproportionately
The conclusion draw in the first sentence large number of midair collisions
depends on which of the following around centrally located airports.
assumptions?
A. Outlying airfields would be as 33. Which of the following best completes the
convenient as centrally located passage below?
airports for most pilots of private Established companies concentrate on
planes. defending what they already have.
B. Most outlying airfields are not Consequently, they tend not to be
equipped to handle commercial-airline innovative themselves and tend to
traffic. underestimate the effects of the
C. Most private planes that use centrally innovations of others. The clearest
located airports are not equipped with example of this defensive strategy is the
radar. fact that___.
D. Commercial airliners are at greater risk A. ballpoint pens and soft-tip markers
of becoming involved in midair have eliminated the traditional market
collisions than are private planes. for fountain pens, clearing the way for
E. A reduction in the risk of midair the marketing of fountain pens as
collision would eventually lead to luxury or prestige items
increases in commercial-airline traffic. B. a highly successful automobile was
introduced by the same company that
32. If the airspace around centrally located had earlier introduced a model that
airports were restricted to commercial had been a dismal failure
airliners and only those private planes C. a once-successful manufacturer of
equipped with radar, most of the private- slide rules reacted to the introduction
plane traffic would be forced to sue of electronic calculators by trying to
outlying airfields. Such a reduction in the make better slide rules
amount of private-plane traffic would D. one of the first models of modern
reduce the risk of midair collision around accounting machines, designed for use
the centrally located airports. in the banking industry, was purchased
Which of the following, if true, would most by a public library as well as by banks
strengthen the conclusion drawn in the E. the inventor of a commonly used
second sentence? anesthetic did not intend the product
A. Commercial airliners are already to be used by dentists, who currently
required by law to be equipped with account for almost the entire market
extremely sophisticated radar for that drug.
systems.
B. Centrally located airports are 34. Most archaeologists have held that people
experiencing overcrowded airspace first reached the Americas less than 20,000
primarily because f sharp increases in years ago by crossing a land bridge into
commercial-airline traffic. North America. But recent discoveries of

142 | I U P I T B
human shelters in South America dating weevils known to be efficient pollinators of
from 32,000 years ago have led researchers palm flowers were introduced into Asia in
to speculate that people arrived in South 1980, palm fruit productivity increased-by
America first, after voyaging across the up to fifty percent in some areas-but then
Pacific, and then spread northward. decreased sharply in 1984.
Which of the following, if it were Which of the following statements, if true,
discovered, would be pertinent evidence would best explain the 1984 decrease in
against the speculation above? productivity?
A. A rock shelter near Pittsburgh, A. Prices for palm fruit fell between 1980
Pennsylvania, contains evidence of use and 1984 following the rise in
by human beings 19,000 years ago. production and a concurrent fall in
B. Some North American sites of human demand.
habitation predate any sites found in B. Imported trees are often more
South America. productive than native trees because
C. The climate is warmer at the 32,000- the imported ones have left behind
year-old South American site than at their pests and diseases in their native
the oldest known North American site. lands.
D. The site in South America that was C. Rapid increases in productivity tend to
occupied 32,000 years ago was deplete trees of nutrients needed for
continuously occupied until 6,000 the development of the fruit-
years ago. producing female flowers.
E. The last Ice Age, between 11,500 and D. The weevil population in Asia
20,000 years ago, considerably remained at approximately the same
lowered worldwide sea levels. level between 1980 and 1984.
E. Prior to 1980 another species of insect
35. In Asia, where palm trees are non-native, pollinated the Asian palm trees, but
the trees' flowers have traditionally been not as efficiently as the species of
pollinated by hand, which has kept palm weevil that was introduced in 1980.
fruit productivity unnaturally low. When

36. The graph of function 𝑔 in the 𝑥𝑦-plane is a C. 𝑎 – 6


parabola defined by 𝑔(𝑥) = (𝑥 − D. 𝑎 + 6
2)(𝑥 − 4).Which of the following
intervals contains the 𝑥-coordinate of the 38. If the expression (𝑥 + 𝑐)(𝑥 – 𝑐), where 𝑐
vertex of the graph? is a positive constant, can be rewritten as
1
A. 6 < 𝑥 < 8 𝑥 2 – 5, what is the value of 𝑐 ?
2
B. 4 < 𝑥 < 6
A. √5
C. −2 < 𝑥 < 4
B. √10
D. −4 < 𝑥 < −2
C. 5
D. 10
37. 𝑥𝑎3 + 𝑦𝑎2 + 𝑧𝑎 = 0
In the equation above, 𝑥, 𝑦, and 𝑧 are
39. If 20 − 𝑥 = 15, what is the value of 3𝑥 ?
constants. If the equation has roots −6, 0,
A. 5
and 4, which of the following is a factor of
B. 10
𝑥𝑎3 + 𝑦𝑎2 + 𝑧𝑎 ?
C. 15
A. 𝑎 – 2
B. 𝑎 + 4 D. 35

B E R A N I B E R J U A N G | 143
40. Which of the following is equivalent to feet, 𝑡 seconds after the helicopter begins
𝑥 2 +7𝑧−3 to gain altitude?
?
𝑧+2
13 A. 𝑦 = 40 + 21
A. 𝑧 + 5 − B. 𝑦 = 40 + 21𝑡
𝑧+2
7
B. 𝑧+5− C. 𝑦 = 40 − 21𝑡
𝑧+2
21 D. 𝑦 = 40𝑡 + 21
C. 𝑧+9−
𝑧−2
15
D. 𝑧 + 9 − 1
𝑝4 𝑞 −3
𝑧−2
44. 1
𝑝−2 𝑞 2
41. Juliet is selling photographs as part of a Which of the following is equivalent to the
project for her entrepreneurship class. She expression above, where 𝑝 > 1 and 𝑞 >
sells the first 20 photographs for $10 each. 1?
Because the first 20 photographs sold so 𝑝2 4√𝑝
A.
quickly, she raised the price of the 𝑞 3 √𝑞
𝑝2 √𝑝
photographs to $15 each for the rest of the B.
𝑞 3 √𝑞
project. After her expenses, Juliet earns a 4 𝑝

profit of 80% of the revenues from her C.
𝑞 3 √𝑞
4 𝑝
sales. What is the least number of D. √
3
photographs she must sell for the rest of √𝑞

the project to earn a profit of at least $400?


45. A text messaging plan charges a flat fee of
A. 18
$5 per month for up to 100 text messages
B. 20
sent plus $0.25 for each additional text
C. 24
message sent that month. Which of the
D. 32
following graphs represents the cost, 𝑦, of
sending 𝑥 texts in a month?
42. A homeowners’ association limits the
dimensions of the pools that it will allow in
a particular subdivision. The bylaws state
that permits will only be granted for pools
shaped like rectangular prisms, for which
the sum of the length of the pool and the A.
perimeter of the vertical side containing
the ladder cannot exceed 200 meters. The
perimeter of the ladder side is determined
using the width and the depth of the pool.
If a pool has a length of 75 meters and its B.
width is 1.5 times its depth, which of the
following shows the allowable depth a, in
meters, of the pool?
1
A. 0 < 𝑎 ≤ 62 C.
2
B. 0 < 𝑎 ≤ 50
1
C. 0 < 𝑎 ≤ 31
4
D. 0 < 𝑎 ≤ 25
D.
43. A helicopter, initially hovering 40 feet
above the ground, begins to gain altitude at
a rate of 21 feet per second. Which of the
following functions represents the
helicopter’s altitude above the ground 𝑦, in
144 | I U P I T B
46. Jake buys a bag of popcorn at a movie B. Selecting 10 employees from each
theater. He eats half of the popcorn during store at random and then surveying
the 15 minutes of previews. After eating each employee selected
half of the popcorn, he stops eating for the C. Surveying the 25 highest-paid
next 30 minutes. Then he gradually eats the employees and the 25 lowest-paid
popcorn until he accidentally spills all of the employees
remaining popcorn. Which of the following D. Creating a website on which
graphs could represent the situation? employees can express their opinions
and then using the first 50 responses

48. The two graphs below show the total


amounts of money that Ian and Jeremy
each have deposited into their savings
accounts for the first seven weeks after
opening their accounts. After they made
A. their initial deposits, how much more did
Ian deposit each week than Jeremy?

B.

A. $200
B. $100
C. C. $50
D. $25

49. ℎ(𝑥) = 2𝑥
The function ℎ is defined above. What is
ℎ(5) − ℎ(3)?
A. 2
D. B. 4
C. 24
47. A retail company has 50 large stores D. 28
located in different areas throughout a
state. A researcher for the company 50. A book was on sale for 40% off its original
believes that employee job satisfaction price. If the sale price of the book was
varies greatly from store to store. Which of $18.00, what was the original price of the
the following sampling methods is most book? (Assume there is no sales tax.)
appropriate to estimate the proportion of A. $7.20
all employees of the company who are B. $10.80
satisfied with their job? C. $30.00
A. Selecting one of the 50 stores at D. $45.00
random and then surveying each
employee at that store
B E R A N I B E R J U A N G | 145
51. A researcher surveyed a random sample of 52. Which of the following colonies showed a
students from a large university about how decrease in size every two weeks after the
often they see movies. Using the sample initial treatment with pesticide?
data, the researcher estimated that 23% of I. Colony 𝐴
the students in the population saw a movie II. Colony 𝐵
at least once per month. The margin of III. Colony 𝐶
error for this estimation is 4%. Which of the A. I only
following is the most appropriate B. III only
conclusion about all students at the C. I and II only
university, based on the given estimate and D. I, II, and III
margin of error?
A. It is unlikely that less than 23% of the 53. Of the following, which is closest to the
students see a movie at least once per ratio of the total number of insects in all
month. three colonies in week 8 to the total
B. At least 23%, but no more than 25%, of number of insects at the time of initial
the students see a movie at least once treatment?
per month. A. 2 to 5
C. The researcher is between 19% and B. 1 to 4
27% sure that most students see a C. 3 to 5
movie at least once per month. D. 1 to 2
D. It is plausible that the percentage of
students who see a movie at least once 54. In 2015 the populations of City 𝑋 and City
per month is between 19% and 27%. 𝑌 were equal. From 2010 to 2015, the
population of City 𝑋 increased by 20% and
Questions 53 and 54 refer to the following the population of City 𝑌 decreased by 10%.
information. If the population of City 𝑋 was 120,000 in
2010, what was the population of City 𝑌 in
2010?
A. 60,000
B. 90,000
C. 160,000
D. 240,000

55. The volume of a sphere is given by the


4
formula 𝑣 = 𝜋𝑟 3 . Where 𝑟 is the radius
3
of the sphere. Which of the following gives
the radius of the sphere in terms of the
volume of the sphere?
4𝜋
Three colonies of insects were each treated A.
3𝑉
3𝑉
with a different pesticide over an 8-week period B.
4𝜋
to test the effectiveness of the three pesticides. 3 4𝜋
Colonies 𝐴, 𝐵, and 𝐶 were treated with C. √
3𝑣
Pesticides 𝐴, 𝐵, and 𝐶, respectively. Each 3 3𝑉
pesticide was applied every 2 weeks to one of D. √
4𝜋
the three colonies over the 8-week period. The
bar graph above shows the insect counts for
each of the three colonies 0, 2, 4, 6, and 8 weeks
after the initial treatment.

146 | I U P I T B
56. A right circular cone has a volume of 24𝜋 Answer Percent
cubic inches. If the height of the cone is 2 Never 31.3%
inches, what is the radius, in inches, of the Rarely 24.3%
base of the cone? Often 13.5%
A. 2√3 Always 30.9%
B. 6 A. 0.31
C. 12 B. 0.38
D. 36 C. 0.45
D. 0.69
57. The table below shows two lists of
numbers. Which of the following is a true 59. 𝑦 = −(𝑥 − 3)2 + 𝑎
statement comparing list 𝐴 and list 𝐵 ? In the equation above, 𝑎 is a constant. The
graph of the equation in the 𝑥𝑦-plane is a
List 𝑨 1 2 3 4 5 6 parabola. Which of the following is true
List 𝑩 2 3 3 4 4 5 about the parabola?
A. The means are the same, and the A. Its minimum occurs at (−3, 𝑎).
standard deviations are different. B. Its minimum occurs at (3, 𝑎).
B. The means are the same, and the C. Its maximum occurs at (−3, 𝑎).
standard deviations are the same. D. Its maximum occurs at (3, 𝑎).
C. The means are different, and the
standard deviations are different. 60. The maximum value of a data set consisting
D. The means are different, and the of 25 positive integers is 84. A new data set
standard deviations are the same. consisting of 26 positive integers is created
by including 96 in the original data set.
58. The table above shows the results of a Which of the following measures must be
survey in which tablet users were asked 12 greater for the new data set than for the
how often they would watch video original data set?
advertisements in order to access A. The mean
streaming content for free. Based on the B. The median
table, which of the following is closest to C. The range
the probability that a tablet user answered D. The standard deviation
“Always,” given that the tablet user did not
answer “Never”?

61. Data set 𝑺 has 𝒙 members, where 𝒙 is an odd integer. The average (arithmetic mean)
and the median of the elements in 𝑺 are both equal to 𝟎.
𝑸𝒖𝒂𝒏𝒕𝒊𝒕𝒚 𝑨 𝑸𝒖𝒂𝒏𝒕𝒊𝒕𝒚 𝑩
𝑇ℎ𝑒 𝑛𝑢𝑚𝑏𝑒𝑟 𝑜𝑓 𝑒𝑙𝑒𝑚𝑒𝑛𝑡𝑠 𝑖𝑛 𝑆 𝑇ℎ𝑒 𝑛𝑢𝑚𝑏𝑒𝑟 𝑜𝑓 𝑒𝑙𝑒𝑚𝑒𝑛𝑡𝑠 𝑖𝑛 𝑆
𝑡ℎ𝑎𝑡 𝑎𝑟𝑒 𝑙𝑒𝑠𝑠 𝑡ℎ𝑎𝑛 𝑡ℎ𝑒 𝑚𝑒𝑑𝑖𝑎𝑛 𝑡ℎ𝑎𝑡 𝑎𝑟𝑒 𝑔𝑟𝑒𝑎𝑡𝑒𝑟 𝑡ℎ𝑎𝑛 𝑡ℎ𝑒 𝑚𝑒𝑑𝑖𝑎𝑛.
A. Quantity A is greater.
B. Quantity B is greater.
C. The two quantities are equal.
D. The relationship cannot be determined from the information given.

B E R A N I B E R J U A N G | 147
62. 𝒎<𝒏
𝑸𝒖𝒂𝒏𝒕𝒊𝒕𝒚 𝑨 𝑸𝒖𝒂𝒏𝒕𝒊𝒕𝒚 𝑩

𝑚2 𝑛2 1

A. Quantity A is greater.
B. Quantity B is greater.
C. The two quantities are equal.
D. The relationship cannot be determined from the information given.

63. The shorter leg of right triangle 𝑨𝑩𝑪 is 𝟐𝟓 𝒄𝒎 and the hypotenuse is 𝟔𝟓 𝒄𝒎. The two
legs of right triangle 𝑫𝑬𝑭 have lengths of 𝟑𝟔 𝒄𝒎 and 𝟒𝟖 𝒄𝒎.
𝑸𝒖𝒂𝒏𝒕𝒊𝒕𝒚 𝑨 𝑸𝒖𝒂𝒏𝒕𝒊𝒕𝒚 𝑩

𝑇ℎ𝑒 𝑙𝑒𝑛𝑔𝑡ℎ 𝑜𝑓 𝑡ℎ𝑒 𝑙𝑜𝑛𝑔𝑒𝑟 𝑙𝑒𝑔 𝑇ℎ𝑒 𝑙𝑒𝑛𝑔𝑡ℎ 𝑜𝑓 𝑡ℎ𝑒


𝑜𝑓 𝑡𝑟𝑖𝑎𝑛𝑔𝑙𝑒 𝐴𝐵𝐶 ℎ𝑦𝑝𝑜𝑡𝑒𝑛𝑢𝑠𝑒 𝑜𝑓 𝑡𝑟𝑖𝑎𝑛𝑔𝑙𝑒 𝐷𝐸

A. Quantity A is greater.
B. Quantity B is greater.
C. The two quantities are equal.
D. The relationship cannot be determined from the information given.

64. 𝑸𝒖𝒂𝒏𝒕𝒊𝒕𝒚 𝑨 𝑸𝒖𝒂𝒏𝒕𝒊𝒕𝒚 𝑩


0.3 30 × 102
10−4

A. Quantity A is greater.
B. Quantity B is greater.
C. The two quantities are equal.
D. The relationship cannot be determined from the information given.

65. 𝑸𝒖𝒂𝒏𝒕𝒊𝒕𝒚 𝑨 𝑸𝒖𝒂𝒏𝒕𝒊𝒕𝒚 𝑩


𝑇ℎ𝑒 𝑎𝑟𝑒𝑎 𝑜𝑓 𝑎 𝑠𝑞𝑢𝑎𝑟𝑒 34
𝑤𝑖𝑡ℎ 𝑝𝑒𝑟𝑖𝑚𝑒𝑡𝑒𝑟 36

A. Quantity A is greater.
B. Quantity B is greater.
C. The two quantities are equal.
D. The relationship cannot be determined from the information given.

66. What is the value of 𝑋, if 𝑋 and 𝑌 are two distinct integers and their product is 30?
(1) 𝑋 is an odd integer
(2) 𝑋 > 𝑌
A. Statement (1) ALONE is sufficient, but statement (2) alone is not sufficient.
B. Statement (2) ALONE is sufficient, but statement (1) alone is not sufficient.
C. BOTH statements TOGETHER are sufficient, but NEITHER statement ALONE is sufficient.
D. EACH statement ALONE is sufficient.
E. Statements (1) and (2) TOGETHER are NOT sufficient.

148 | I U P I T B
67. What is the standard deviation (SD) of the four numbers 𝑝, 𝑞, 𝑟, 𝑠?
(1) The sum of 𝑝, 𝑞, 𝑟 and 𝑠 is 24
(2) The sum of the squares of 𝑝, 𝑞, 𝑟 and 𝑠 is 224
A. Statement (1) ALONE is sufficient, but statement (2) alone is not sufficient.
B. Statement (2) ALONE is sufficien t, but statement (1) alone is not sufficient.
C. BOTH statements TOGETHER are sufficient, but NEITHER statement ALONE is sufficient.
D. EACH statement ALONE is sufficient.
E. Statements (1) and (2) TOGETHER are NOT sufficient.

68. How is Bill related to Betty?


1. Cindy, the wife of Bill's only brother Chris does not have any siblings.
2. Betty is Cindy's brother in law's wife.
A. Statement (1) ALONE is sufficient, but statement (2) alone is not sufficient.
B. Statement (2) ALONE is sufficient, but statement (1) alone is not sufficient.
C. BOTH statements TOGETHER are sufficient, but NEITHER statement ALONE is sufficient.
D. EACH statement ALONE is sufficient.
E. Statements (1) and (2) TOGETHER are NOT sufficient.

69. If a salesman received a commission of 3% of the sales that he has booked in a month, what
was the sales booked by the salesman in the month of November 2003?
1. The sales booked by the salesman in the month of November 2003 minus salesman's
commission was $245,000
2. The selling price of the sales booked by the salesman in the month of November 2003
were 125 percent of the original purchase price of $225,000.
A. Statement (1) ALONE is sufficient, but statement (2) alone is not sufficient.
B. Statement (2) ALONE is sufficient, but statement (1) alone is not sufficient.
C. BOTH statements TOGETHER are sufficient, but NEITHER statement ALONE is sufficient.
D. EACH statement ALONE is sufficient.
E. Statements (1) and (2) TOGETHER are NOT sufficient.

70. −9 − 𝑎 = 𝑏
𝑎2 − 6𝑎 − 9 = 𝑏
If the ordered pair (𝑎, 𝑏) satisfies the system of equations above, what is one possible value of
𝑎?

71. Point 𝑂 is the center of the circle below. What fraction of the circumference of the circle is the
length of the bolded arc?

B E R A N I B E R J U A N G | 149
5
72. In the figure below, 𝑠𝑖𝑛 𝑇 = . If 𝑇𝑉 = 24, 𝑋𝑍 = 13, and ∠𝑊 ≅ ∠𝑍 , what is 𝑉𝑊 – 𝑌𝑍 ?
13

Questions 73 and 74 refer to the following information.

The same 15 participants, on each of 3 days, threw 5 darts in order to win a bullseye contest. The
number of players throwing a given number of bullseyes on each day is shown in the table above.

73. No participant threw the same number of bullseyes on two different days. If a participant is
selected at random, what is the probability that the selected participant threw 3 bullseyes on
Day 1 or Day 2, given that the contestant threw 3 bullseyes on one of the three days?

74. What is the mean number of bullseyes each participant threw on Day 2 ?

75. In the figure below, point 𝐷 is on line 𝐴𝐶, 𝐴𝐵 = 𝐵𝐷 = 𝐶𝐷, and 𝐴𝐷 = 15. What is the
measure, in degrees, of ∠𝐵𝐶𝐷 ? (Disregard the degree symbol when gridding your answer.)

150 | I U P I T B
SET-9

Questions 1–10 are based on the following passages.

Passage 1 is adapted from James Platt, Poverty. Originally published in 1884. Passage 2 is adapted
from Will Reason, Poverty. Originally published in 1909. As societies became more industrialized
and urban in the late nineteenth century, discussions arose regarding the root causes of poverty.

Line Passage 1
The aim of charitable persons should not be so much the giving with money in hand,
or religious teaching on their bps, as the sympathetic, friendly intercourse of man with
man, woman with woman, irrespective of class, and actuated by the desire to stimulate
hope and energy, and to show the lowest outcast that the world, even to them, may be
5 made more enjoyable, if they have the desire to live a life more in harmony with the better
part of their nature; and so in time, by degrees, as the child is taught to walk, step by step,
we may improve the lowest types of humanity.
To obtain better dwellings for the poor, it is essential for the poorer class to feel a
want for, and a desire to have, houses better, cleaner, and more wholesome than those
10 they now inhabit. Too many, at present, wantonly and maliciously, but more frequently,
through neglect and apathy, injure the houses in which they live. Many of them prefer to
be uncomfortable.
The man or men that are wanted to solve this difficult question, the only true or real
reformers, will be those who study how to alter the character, estimate of life, the ideal
15 of existence amongst the poor. From the clergy of all denominations, from every man who
has influence over the masses, we want the moral courage to tell the poor not to marry
until they are in a position to support a wife and family, to be reasonably industrious,
uniformly thrifty, and unswervingly sober. The real reformer, the only savior of society, is
the man who will tell the poor how they may become less so, by teaching them how to
20 earn more or spend less, not by robbing others of what they have.
We must raise the status of the poorer class, improve their individuality, give them a
higher character, and thus prepare the way for a well-working humanity that should result
in well-working institutions. We must regenerate the entire social fabric, through the
working of juster laws, purer aims, nobler instincts—through individual cooperation of the
25 many, as one, in contributing to the welfare of all.

B E R A N I B E R J U A N G | 151
Passage 2
It is clear, therefore, that the readjustment of distribution must be effected through
a readjustment of the ownership and management of the essential factors of production.
It is not possible to effect this readjustment on the basis of unrestricted competition,
because in the case on the prime factor of production, the land monopoly is caused, not
30 by any artificial arrangement, but by the simple natural fact that it is essential and
restricted in quantity. The only course that remains is to find some way of democratizing
the monopoly value, so that everyone benefits by his due share of what has not been
made by the industry of any and is indispensable to the industry of all.
This cannot be effected by parceling it out to individuals, for very obvious reasons. It
35 would have to be continually redistributed, it would be impossible to equate the value of
the different parts, and such distribution would destroy its productive efficiency.
We therefore must look to some means of collective ownership and use of these
natural forces, and also of all the value that accrues not from individual energy, industry
or skill, but from the conditions which society itself affords. "To deal with causes we must
40 strike at the error of distribution, by gradually substituting public ownership for private
ownership of the means of production. In no other way can we secure for each worker in
the hive the full reward of his labor. So long as between the worker and his just wage
stands the private landlord and the private capitalist, so long will poverty remain, and not
poverty alone, but the moral degradations which inevitably arise from the devotion of
45 labor to the service of waste: [writes L.G. Chiozza in °Riches and Poverty”]
Here we touch the greatest controversy of the present time, and another volume
would be needed to examine all the reasons for and against such a social adjustment. But
it is clear that unless some way of effecting it is found, consistent with fairness to all,
poverty, undeserved and unavoidable, must be the lot of many, while equally undeserved
50 income will continue to be reaped, without conscious or intentional fraud on their part,
by many others. As the poverty reacts on the character for ill effect, so also it is to be
feared that the unearned riches also produce qualities, of a different and more subtle
nature, but equally inimical to the true well-being both of the individuals concerned and
of the community.

1. In Passage 1, Platt suggests that one way a A. Neither the rich nor the poor would
society can reduce poverty is to reap benefits.
A. provide in-demand technical training B. All parties to the economy would
to citizens in need of marketable skills. improve their position.
B. redistribute wealth directly from C. The rich would take advantage to
certain types of rich men. create their own monopolies.
C. teach that the benefits of budgeting D. The poor would be able to purchase
outweigh the benefits of handouts. property at decreased value.
D. instruct children in financial matters
from a young age. 4. Which choice provides the best evidence
for the answer to the previous question?
2. Which choice provides the best evidence A. Lines 31–32 (“The only...value”)
for the answer to the previous question? B. Lines 34–36 (“It would...efficiency”)
A. Lines 6–7 (“as the...humanity”) C. Lines 37–39 (“We therefore...affords”)
B. Lines 13–15 (“The man...poor”) D. Lines 46–47 (“Here we...adjustment”)
C. Lines 18–20 (“The real...have”)
D. Lines 23–25 (“We must...all”) 5. As used in line 42, “just” most nearly means
A. only.
3. In Passage 2, Reason implies that evenly B. strict.
distributing wealth and property among C. equal.
individuals would have what consequence? D. fair.

152 | I U P I T B
6. As used in line 1, “aim” most nearly means A. Platt believes that financial education
A. focus. is the key to reducing poverty in a
B. shot. society, while Reason believes that
C. gift. financial education is irrelevant if
D. ability. individuals do not participate.
B. Platt believes that poverty will be
7. Reason in Passage 2 would most likely reduced only if those who are poor
characterize the position taken by Platt in truly desire to lift themselves out of
lines 11–12 (“Many of... uncomfortable”) poverty, while Reason believes that
as poverty will be reduced only if
A. an accurate description of the root society’s wealth is redistributed.
cause of the current economic C. Platt believes wage regulation from
situation of the poorer class. major industry is a step in the right
B. a flawed assumption that generalizes direction, while Reason believes that
the negative behavior of a few. such regulation will be ineffective.
C. a surprising but realistic statement D. Platt believes training clergy to
consistent with the economic trends of minister to the poor is the most
the times. effective way of improving the lives of
D. an offensive characterization of the those in poverty, while Reason
rich that is not applicable to other believes that unearned riches are a
classes of society. necessary feature of a society.

8. Both authors would most likely agree that 10. Based on Passage 1, Platt would most likely
any social changes undertaken to fight say that the “readjustment of distribution”
poverty would be Reason mentions in lines 26 would have no
A. likely to completely solve the effect unless
problems of financial inequality. A. property were owned collectively by
B. a stopgap measure needed to prevent all members of society.
the wealthy from keeping their B. profits from property sales were
money. evenly distributed equally amongst all
C. unsuccessful without a drastic change citizens.
in the mindset of the poor. C. those in poverty exhibited sufficient
D. partial solutions that would present effort to better themselves.
new sets of challenges. D. the number of available jobs increased
enough to significantly lower
9. Which choice best describes the way the unemployment.
two authors characterize effective ways to
reduce poverty?

Questions 11-21 are based on the following passageand supplementary material.

Compost: Don’t Waste This Waste


Over the past generation, people in many parts of the United States have become accustomed
to dividing their household waste products into different categories for recycling. (11) Regardless,
paper may go in one container, glass and aluminum in another, regular garbage in a third. Recently,
some US cities have added a new category: compost, organic matter such as food scraps and yard
debris. Like paper or glass recycling, composting demands a certain amount of effort from the

11.
A. NO CHANGE
B. However,
C. Furthermore,
D. For example,
B E R A N I B E R J U A N G | 153
public in order to be successful. But the inconveniences of composting are far outweighed by its
benefits.
Most people think of banana peels, eggshells, and dead leaves as “waste,” but compost is
actually a valuable resource with multiple practical uses. When utilized as a garden fertilizer,
compost provides nutrients to soil and improves plant growth while deterring or killing pests and
preventing some plant diseases. It also enhances soil texture, encouraging healthy roots and
minimizing or (12) annihilating the need for chemical fertilizers. Better than soil at holding
moisture, compost minimizes water waste and storm runoff, (13) it increases savings on watering
costs, and helps reduce erosion on embankments near bodies of water. In large (14) quantities,
which one would expect to see when it is collected for an entire municipality), compost can be
converted into a natural gas that can be used as fuel for transportation or heating and cooling
systems.

12. Which choice best maintains the style and 14.


tone of the passage? A. NO CHANGE
A. NO CHANGE B. quantities (which
B. eliminating C. quantities which
C. ousting D. quantities; (which
D. closing the door on
13.
A. NO CHANGE
B. savings increase
C. increases savings
D. also it increases savings

In spite of all compost’s potential uses, however, most of this so-called waste is wasted.
According to the Environmental Protection Agency (EPA), over (15) 13 million tons of metal ended
up in US landfills in 2009, along with over 13 million tons of yard debris. Remarkably, (16) less glass
was discarded in landfills in that year than any other substance, including plastics or paper. Even
(17) worse, then the squandering of this useful resource is the fact that compost in landfills cannot
break down due to the lack of necessary air and moisture.

15. The writer wants to include information A. NO CHANGE


from the graph that is consistent with the B. more metal
description of compost in the passage. C. more food waste
Which choice most effectively D. more yard waste
accomplishes this goal?
A. NO CHANGE 17.
B. 6 million tons of rubber and leather A. NO CHANGE
C. 10 million tons of textiles B. worse than
D. 33 million tons of food waste C. worse then
D. worse, than
16. The writer wants to support the
paragraph’s main idea with accurate,
relevant information from the graph.
Which choice most effectively
accomplishes this goal?

154 | I U P I T B
As a result, organic material that is sent to landfills (18) contribute to the release of methane, a
very (19) potent greenhouse gas

18. 19.
A. NO CHANGE A. NO CHANGE
B. are contributing B. sturdy
C. contributes C. influential
D. have contributed D. commanding

(20) While composting can sometimes lead to accidental pollution through the release of methane
gas, cities such as San Francisco and Seattle have instituted mandatory composting laws requiring
individuals and businesses to use separate bins for compostable waste. This strict approach may
not work everywhere. However, given the clear benefits of composting and the environmental
costs of not composting, all municipalities should encourage their residents either to create their
own compost piles for use in backyard gardens (21) or to dispose of compostable materials in bins
for collection.

20. Which choice provides the most effective 21.


transition from the previous paragraph? A. NO CHANGE
a. NO CHANGE B. Nor
b. Though government regulations vary, C. But
c. Armed with these facts, D. And
d. Mindful of this setback,

Questions 22-25 are based on the following passage.

The joint family gave way to the ---(22)--- family. Despite the ---(23)--- that it offers, children feel --
-(24)---, lonely and frustrated leading to psychiatrist problems, pressures and ---(25)---

22. In each of the following passages, there a 23. In each of the following passages, there are
re blanks each of which has been blanks each of which has been numbered.
numbered. Find out the appropriate Find out the appropriate words.
words. A. separation
a. self centred B. isolation
b. nuclear C. privacy
c. individual D. seclusion
d. lonely
B E R A N I B E R J U A N G | 155
24. In each of the following passages, there are 25. In each of the following passages, there are
blanks each of which has been numbered. blanks each of which ha s been numbered.
Find out the appropriate words. Find out the appropriate words.
A. segregated A. controversies
B. disregarded B. complications
C. neglected C. complexities
D. avoided D. burdens

26. In ancient Rome they spoke early Roman, A. affable


which is different to the words they use B. aloof
now. For examplhypt juna bolla was used C. unfeeling
in place of today is Friday; juna bolla toho D. detached
means Friday is tomorrow, and toho juna E. distant
bolla rum means tomorrow is Friday and it
will rain. 30. NOVICE is to EXPERIENCED as APPRENTICE
What is the early Roman word for ‘today’? is to:
A. juna A. many vices
B. rum B. senior citizen
C. bolla C. oldie
D. toho D. exotic
E. hypt E. veteran

27. Four of the following words are alike in 31. Since the mayor's publicity campaign for
some way. Greenville's bus service began six months
Which is the odd word out? ago, morning automobile traffic into the
A. needle midtown area of the city has decreased
B. thread seven percent. During the same period,
C. wool there has been an equivalent rise in the
D. knit number of persons riding buses into the
E. thimble midtown area. Obviously, the mayor's
publicity campaign has convinced many
28. Four of the following words are alike in people to leave their cars at home and ride
some way. the bus to work.
Which is the odd word out? Which of the following, if true, casts the
A. cold most serious doubt on the conclusion
B. humid drawn above?
C. mild A. Fares for all bus routes in Greenville
D. frosty have risen an average of five percent
E. sleet during the past six months.
B. The mayor of Greenville rides the bus
29. All of the words in the box below are alike to City Hall in the city's midtown area.
in some way. C. Road reconstruction has greatly
impersonal unfriendly reduced the number of lanes available
unapproachable cold to commuters in major streets leading
Which of the following options could not go to the midtown area during the past
in the box? six months.

156 | I U P I T B
D. The number of buses entering the turn, would slow the development of
midtown area of Greenville during the biological science and engineering.
morning hours is exactly the same now Which of the following, if true, would tend
as it was one year ago. to weaken most seriously the prediction of
E. Surveys show that longtime bus riders scientific secrecy described above?
are no more satisfied with the A. Biotechnological research funded by
Greenville bus service than they were industry has reached some
before the mayor's publicity campaign conclusions that are of major scientific
began. importance.
B. When the results of scientific research
32. In the aftermath of a worldwide stock- are kept secret, independent
market crash, Country T claimed that the researchers are unable to build on
severity of the stock-market crash it those results.
experienced resulted from the accelerated C. Since the research priorities of
process of denationalization many of its biotechnology companies are not the
industries underwent shortly before the same as those of academic
crash. institutions, the financial support of
Which of the following, if it could be carried research by such companies distorts
out, would be most useful in an evaluation the research agenda.
of Country T's assessment of the causes of D. To enhance the companies' standing in
the severity of its stock-market crash? the scientific community, the
A. calculating the average loss biotechnology companies encourage
experienced by individual traders in employees to publish their results,
Country T during the crash especially results that are important.
B. using economic theory to predict the E. Biotechnology companies devote
most likely date of the next crash in some of their research resources to
Country T problems that are of fundamental
C. comparing the total number of shares scientific importance and that are not
sold during the worst days of the crash expected to produce immediate
in Country T to the total number of practical applications.
shares sold in Country T just prior to
the crash 34. Some people have questioned the judge's
D. comparing the severity of the crash in objectivity in cases of sex discrimination
Country T to the severity of the crash against women. But the record shows that
in countries otherwise economically in sixty percent of such cases, the judge has
similar to Country T that have not decided in favor of the women. This record
experienced recent denationalization demonstrates that the judge has not
E. comparing the long-term effects of the discriminated against women in cases of
crash on the purchasing power of the sex discrimination against women.
currency of Country T to the The argument above is flawed in that it
immediate, more severe short-term ignores the possibility that
effects of the crash on the purchasing A. a large number of the judge's cases
power of the currency of Country T arose out of allegations of sex
discrimination against women
33. With the emergence of biotechnology B. many judges find it difficult to be
companies, it was feared that they would objective in cases of sex discrimination
impose silence about proprietary results on against women
their in-house researchers and their
academic consultants. This constraint, in

B E R A N I B E R J U A N G | 157
C. the judge is biased against women A. During this year, the number of
defendants or plaintiffs in cases that women who have begun to smoke is
do not involve sex discrimination greater than the number of men who
D. the majority of the cases of sex have quit smoking
discrimination against women that B. The number of teen-age children who
have reached the judge's court have have begun to smoke this year is
been appealed from a lower court greater than the number of adults who
E. the evidence shows that the women have quit smoking during the same
should have won in more than sixty period
percent of the judge's cases involving C. During this year, the number of
sex discrimination against women nonsmokers who have begun to use
chewing tobacco or snuff is greater
35. The tobacco industry is still profitable and than the number of people who have
projections are that it will remain so. In the quit smoking
United States this year, the total amount of D. The people who have continued to
tobacco sold by tobacco-farmers has smoke consume more tobacco per
increased, even though the number of person than they did in the past
adults who smoke has decreased. E. More of the cigarettes made in the
Each of the following, if true, could explain United States this year were exported
the simultaneous increase in tobacco sales to other countries than was the case
and decrease in the number of adults who last year.
smoke EXCEPT:

36. 0.10𝑥 + 0.20𝑦 = 0.18(𝑥 + 𝑦) A. The function 𝑓 is a decreasing linear


Clayton will mix 𝑥 milliliters of a 10% by function.
mass saline solution with 𝑦 milliliters of a B. The function 𝑓 is an increasing linear
20% by mass saline solution in order to function.
create an 18% by mass saline solution. The C. The function 𝑓 is a decreasing
equation above represents this situation. If exponential function.
Clayton uses 100 milliliters of the 20% by D. The function 𝑓 is an increasing
mass saline solution, how many milliliters exponential function.
of the 10% by mass saline solution must he
use? 38. Some values of 𝑥 and their corresponding
A. 5 values of 𝑦 are shown in the table below,
B. 25 where 𝑎 is a constant. If there is a linear
C. 50 relationship between 𝑥 and 𝑦, which of the
D. 100 following equations represents the
relationship?
37. The first year Eleanor organized a fund- 𝑥 𝑎 3𝑎 5𝑎
raising event, she invited 30 people. For 𝑦 0 −𝑎 −2𝑎
each of the next 5 years, she invited double A. 𝑥 + 2𝑦 = 𝑎
the number of people she had invited the B. 𝑥 + 2𝑦 = 5𝑎
previous year. If 𝑓 (𝑛) is the number of C. 2𝑥 − 𝑦 = −5𝑎
people invited to the fund-raiser n years D. 2𝑥 − 𝑦 = 7𝑎
after Eleanor began organizing the event,
which of the following statements best
describes the function 𝑓 ?
158 | I U P I T B
39. The scatterplot above shows the number of C. 𝑃(𝑡) = 310(0.99)𝑡
registered voters, 𝑥, and the number of D. 𝑃(𝑡) = 310(1.01)𝑡
people who voted in the last election, 𝑦, for
2
seven districts in a town. A line of best fit 42. (9𝑥 − 6) − 4 = 9𝑥 − 6
3
for the data is also shown. Which of the Based on the equation above, what is the
following could be the equation of the line value of 3𝑥 − 2 ?
of best fit? A. −4
4
B. −
5
2
C. −
3
D. 4

43. 𝑓 (𝑥) = (𝑥 + 3)(𝑥 − 𝑘)


The function 𝑓 is defined above. If 𝑘 is a
positive integer, which of the following
could represent the graph of 𝑦 = 𝑓 (𝑥) in
the 𝑥𝑦-plane?
A. 𝑦 = −0.5𝑥
B. 𝑦 = 0.5𝑥
C. 𝑦 = −2𝑥
D. 𝑦 = 2𝑥

40. 2.4𝑥 − 1.5𝑦 = 0.3


1.6𝑥 + 0.5𝑦 = −1.3
The system of equations above is graphed A.
in the 𝑥𝑦-plane. What is the 𝑥-coordinate
of the intersection point (𝑥, 𝑦) of the
system?
A. −0.5
B. −0.25
C. 0.8
D. 1.75
B.
41. Keith modeled the growth over several
hundred years of a tree population by
estimating the number of the trees’ pollen
grains per square centimeter that were
deposited each year within layers of a
lake’s sediment. He estimated there were
310 pollen grains per square centimeter
the first year the grains were deposited, C.
with a 1% annual increase in the number of
grains per square centimeter thereafter.
Which of the following functions models
𝑃(𝑡), the number of pollen grains per
square centimeter 𝑡 years after the first
year the grains were deposited?
A. 𝑃(𝑡) = 310𝑡
D.
B. 𝑃(𝑡) = 3101.01𝑡

B E R A N I B E R J U A N G | 159
44. 𝐻 = 1.88𝐿 + 32.01 48. A television with a price of $300 is to be
The formula above can be used to purchased with an initial payment of $60
approximate the height 𝐻, in inches, of an and weekly payments of $30. Which of the
adult male based on the length 𝐿, in inches, following equations can be used to find the
of his femur. What is the meaning of 1.88 number of weekly payments, 𝑤, required
in this context? to complete the purchase, assuming there
A. The approximate femur length, in are no taxes or fees?
inches, for a man with a height of A. 300 = 30𝑤 – 60
32.01 inches B. 300 = 30𝑤
B. The approximate increase in a man’s C. 300 = 30𝑤 + 60
femur length, in inches, for each D. 300 = 60𝑤 – 30
increase of 32.01 inches in his height
C. The approximate increase in a man’s 49. The table below shows shipping charges for
femur length, in inches, for each one- an online retailer that sells sporting goods.
inch increase in his height There is a linear relationship between the
D. The approximate increase in a man’s shipping charge and the weight of the
height, in inches, for each one-inch merchandise. Which function can be used
increase in his femur length to determine the total shipping charge
𝑓 (𝑥), in dollars, for an order with a
45. In quadrilateral 𝐴𝐵𝐶𝐷 below, ̅̅̅̅ ̅̅̅̅ and
𝐴𝐷 ||𝐵𝐶 merchandise weight of 𝑥 pounds?
1 Merchandise
𝐶𝐷 = 𝐴𝐵. What is the measure of angle Shipping charge
2
weight (pounds)
𝐵?
5 $16.94
10 $21.89
20 $31.79
40 $51.59
A. 𝑓 (𝑥) = 0.99𝑥
A. 150° B. 𝑓 (𝑥) = 0.99𝑥 + 11.99
B. 135° C. 𝑓 (𝑥) = 3.39𝑥
C. 120° D. 𝑓 (𝑥) = 3.39𝑥 + 16.94
D. 90°
50. The line in the 𝑥𝑦-plane above represents
46. 2𝑧 + 1 = 𝑧 the relationship between the height ℎ(𝑥),
What value of 𝑧 satisfies the equation in feet, and the base diameter 𝑥, in feet, for
above? cylindrical Doric columns in ancient Greek
A. −2 architecture. How much greater is the
B. −1 height of a Doric column that has a base
1
C. diameter of 5 feet than the height of a
2
D. 1 Doric column that has a base diameter of 2
feet?
47. √9𝑥 2
If 𝑥 > 0, which of the following is
equivalent to the given expression?
A. 3𝑥
B. 3𝑥 2
C. 18𝑥
D. 18𝑥 4

160 | I U P I T B
A. 7 feet
B. 14 feet
C. 21 feet
D. 24 feet

𝑥 2 −1
51. = −2
𝑥−1
What are all values of 𝑥 that satisfy the
equation above?
A. −3
B. 0 A. 𝑥 − 𝑦 = −4
C. 1 B. 𝑥 − 𝑦 = 4
D. −3 𝑎𝑛𝑑 − 1 C. 𝑥 + 𝑦 = −4
D. 𝑥 + 𝑦 = 4
52. The graph of 𝑦 = 𝑓 (𝑥) is shown in the 𝑥𝑦-
plane. What is the value of 𝑓(0)? 55. The graph of 𝑦 = 2𝑥 2 + 10𝑥 + 12 is
shown. If the graph crosses the 𝑦-axis at
the point (0, 𝑘), what is the value of 𝑘 ?

A. 0
B. 2 A. 2
C. 3 B. 6
D. 4 C. 10
D. 12
53. In the figure below, point 𝐵 lies on ̅̅̅̅
𝐴𝐷 .
What is the value of 3𝑥 ? 56. A circle in the 𝑥𝑦-plane has center (5, 7)
and radius 2. Which of the following is an
equation of the circle?
A. (𝑥 − 5)2 + (𝑦 − 7)2 = 4
B. (𝑥 + 5)2 + (𝑦 + 7)2 = 4
C. (𝑥 − 5)2 + (𝑦 − 7)2 = 2
D. (𝑥 + 5)2 + (𝑦 + 7)2 = 2

57. −3𝑥 + 𝑦 = 6
A. 18 𝑎𝑥 + 2𝑦 = 4
B. 36 In the system of equations above, 𝑎 is a
C. 54 constant. For which of the following values
D. 72 of 𝑎 does the system have no solution?
A. −6
54. Which of the following is an equation of B. −3
line 𝑙 in the 𝑥𝑦-plane below? C. 3
D. 6

B E R A N I B E R J U A N G | 161
58. In the figure below, triangle 𝐴𝐵𝐶 is similar 59. In the 𝑥𝑦-plane, the graph of the function
to triangle 𝐸𝐹 . What is the value of 𝑓 (𝑥) = 𝑥 2 + 5𝑥 + 4 has two 𝑥-
𝑐𝑜𝑠(𝐸)? intercepts. What is the distance between
the 𝑥 −intercepts?
A. 1
B. 2
C. 3
D. 4

60. √4 = 𝑥 − 3
What are all values of x that satisfy the
12
given equation?
A. I. 1
5
12
B. II. 9
13
5 A. I only
C.
12 B. II only
5
D. C. I and II
13
D. Neither I nor II

61. 𝑥>1
𝑸𝒖𝒂𝒏𝒕𝒊𝒕𝒚 𝑨 𝑸𝒖𝒂𝒏𝒕𝒊𝒕𝒚 𝑩

−5√𝑥 + 30 −5(𝑥 − 6)
A. Quantity A is greater.
B. Quantity B is greater.
C. The two quantities are equal.
D. The relationship cannot be determined from the information given.

62.

𝑸𝒖𝒂𝒏𝒕𝒊𝒕𝒚 𝑨 𝑸𝒖𝒂𝒏𝒕𝒊𝒕𝒚 𝑩

8 𝑥
A. Quantity A is greater.
B. Quantity B is greater.
C. The two quantities are equal.
D. The relationship cannot be determined from the information given.

63. 𝐿𝑖𝑠𝑡 𝑇: 8, −1, 10, 0, −5, 8, 2


𝑸𝒖𝒂𝒏𝒕𝒊𝒕𝒚 𝑨 𝑸𝒖𝒂𝒏𝒕𝒊𝒕𝒚 𝑩

𝑇ℎ𝑒 𝑎𝑣𝑒𝑟𝑎𝑔𝑒 (𝑎𝑟𝑖𝑡ℎ𝑚𝑒𝑡𝑖𝑐 𝑚𝑒𝑎𝑛) (𝑇ℎ𝑒 𝑟𝑎𝑛𝑔𝑒 𝑜𝑓 𝐿𝑖𝑠𝑡 𝑇) − (𝑇ℎ𝑒


𝑜𝑓 𝐿𝑖𝑠𝑡 𝑇 𝑚𝑜𝑑𝑒 𝑜𝑓 𝐿𝑖𝑠𝑡 𝑇) − (𝑇ℎ𝑒 𝑚𝑒𝑑𝑖𝑎𝑛
𝑜𝑓 𝐿𝑖𝑠𝑡 𝑇)
A. Quantity A is greater.
B. Quantity B is greater.
C. The two quantities are equal.
D. The relationship cannot be determined from the information given.
162 | I U P I T B
64. 𝑎𝑏 < 0 𝑎𝑛𝑑 𝑎2 𝑏 > 0
𝑸𝒖𝒂𝒏𝒕𝒊𝒕𝒚 𝑨 𝑸𝒖𝒂𝒏𝒕𝒊𝒕𝒚 𝑩

𝑎𝑏 2 0
A. Quantity A is greater.
B. Quantity B is greater.
C. The two quantities are equal.
D. The relationship cannot be determined from the information given.

65. 𝑚≠1
𝑸𝒖𝒂𝒏𝒕𝒊𝒕𝒚 𝑨 𝑸𝒖𝒂𝒏𝒕𝒊𝒕𝒚 𝑩

𝑚 𝑚3
A. Quantity A is greater.
B. Quantity B is greater.
C. The two quantities are equal.
D. The relationship cannot be determined from the information given.

66. Is the positive integer 𝑚 divisible by 6?


1. 𝑚 is divisible by 3
2. 𝑚 is divisible by 4.
A. Statement (1) ALONE is sufficient, but statement (2) alone is not sufficient.
B. Statement (2) ALONE is sufficient, but statement (1) alone is not sufficient.
C. BOTH statements TOGETHER are sufficient, but NEITHER statement ALONE is sufficient.
D. EACH statement ALONE is sufficient.
E. Statements (1) and (2) TOGETHER are NOT sufficient.

67. Determine the price of two type A footballs if the total cost of a type 𝐴 and a type 𝐵 football is
$500.
1. Type 𝐵 football costs $200.
2. Two type 𝐴 and three type 𝐵 footballs costs $1200.
A. Statement (1) ALONE is sufficient, but statement (2) alone is not sufficient.
B. Statement (2) ALONE is sufficient, but statement (1) alone is not sufficient.
C. BOTH statements TOGETHER are sufficient, but NEITHER statement ALONE is sufficient.
D. EACH statement ALONE is sufficient.
E. Statements (1) and (2) TOGETHER are NOT sufficient.

68. What is the value of the positive number, 𝑝?


1. One of its divisors is 7.
2. 𝑝 is divisible by two positive numbers only
A. Statement (1) ALONE is sufficient, but statement (2) alone is not sufficient.
B. Statement (2) ALONE is sufficient, but statement (1) alone is not sufficient.
C. BOTH statements TOGETHER are sufficient, but NEITHER statement ALONE is sufficient.
D. EACH statement ALONE is sufficient.
E. Statements (1) and (2) TOGETHER are NOT sufficient.

B E R A N I B E R J U A N G | 163
69. Ann deposited $3000 in her bank account at the beginning of the year. Determine the amount
the funds accumulated to.
1. The bank offered 4.3% interest rate.
2. The amount was deposited for a period of 5 years.
A. Statement (1) ALONE is sufficient, but statement (2) alone is not sufficient.
B. Statement (2) ALONE is sufficient, but statement (1) alone is not sufficient.
C. BOTH statements TOGETHER are sufficient, but NEITHER statement ALONE is sufficient.
D. EACH statement ALONE is sufficient.
E. Statements (1) and (2) TOGETHER are NOT sufficient.

70. If 15 – 3𝑏 = 21, what is the value of 5 – 𝑏 ?

71. The graph of a line in the 𝑥𝑦-plane passes through the point (−2, 𝑘) and crosses the 𝑥-axis at
the point (−4, 0). The line crosses the 𝑦-axis at the point (0, 12). What is the value of 𝑘 ?

72. 5(10𝑥 2 − 300) + (9844 + 50𝑥 2 )


The expression above can be rewritten in the form 𝑐𝑥 2 + 𝑑, where 𝑐 and 𝑑 are constants.
What is the value of 𝑑 – 𝑐 ?

73. If 𝑛 is a constant equal to the number of degrees in an angle measuring 3𝜋 radians, what is the
value of 𝑛 ?

74. Lynne has $8.00 to spend on apples and oranges. Apples cost $0.65 each, and oranges cost
$0.75 each. If there is no tax on this purchase and she buys 5 apples, what is the maximum
number of whole oranges she can buy?

75. In the triangle below, 𝑎 = 34. What is the value of 𝑏 + 𝑐 ?

164 | I U P I T B
SET-10

Questions 1–10 are based on the following passage and supplementary material.

This passage is excerpted from Thomas Hayden, “What Darwin Didn’t Know.” © 2009 by
Smithsonian Magazine. On the Origin of Species, published 1859

Line [The] first public airing of Darwinian evolution caused almost no stir whatsoever. But
when Darwin published his ideas in book form the following year, the reaction was quite
different. On the Origin of Species by Means of Natural Selection, or the Preservation of
Favoured Races in the Struggle for Life soon sold out its first press run of 1,250 copies, and
5 within a year some 4,250 copies were in circulation. Allies applauded it as a brilliant
unifying breakthrough; scientific rivals called attention to the gaps in his evidence,
including what would come to be known as "missing links" in the fossil record; and
prominent clergymen, politicians and others condemned the work and its far-reaching
implications. In 1864 Benjamin Disraeli, later Britain's prime minister, famously decried
10 the idea—barely mentioned in Origin—that human beings too had evolved from earlier
species. "Is man an ape or an angel?" he asked rhetorically at a conference. "I, my lord, I
am on the side of the angels. I repudiate with indignation and abhorrence those
newfangled theories."
Darwin knew that plant and animal species could be sorted into groups by similarity,
15 such that birds clustered into songbirds and raptors, say, with each group subdivided again
and again down to dozens or hundreds of distinct species. He also saw that the individuals
within any given species, despite many similarities, also differed from one another—and
some of those differences were passed from parents to their offspring. And Darwin
observed that nature had a brutally efficient method of rewarding any variation that
20 helped an individual live longer, breed faster or leave more progeny. The reward for being
a slightly faster or more alert antelope? The lions would eat your slower neighbors first,
granting you one more day in which to reproduce. After many generations and a great
deal of time, the whole population would run faster, and with many such changes over
time eventually become a new species. Evolution, Darwin's "descent with modification
25 through natural selection," would have occurred.
But what was the source of variation and what was the mechanism for passing change
from generation to generation? Darwin "didn't know anything about why organisms

B E R A N I B E R J U A N G | 165
resemble their parents, or the basis of heritable variations in populations:' says Niles
Eldredge, a paleontologist at the American Museum of Natural History in New York City.
30 In Darwin's era, the man who did make progress on the real mechanism of inheritance
was the Austrian monk Gregor Mendel. In his abbey garden in the late 1850s and early
1860s, Mendel bred pea plants and found that the transmission of traits such as flower
color and seed texture followed observable rules. An offspring inherits a set of these
genetic units from each parent. Since the early 1900s, those units of inheritance have been
35 known as genes.
The objection certainly applied to the paucity of ancestral human fossils in Darwin's time.
Years of painstaking work by paleontologists, however, have filled in many of the
important gaps. There are many more extinct species to be discovered, but the term
"missing link" has for the most part become as outdated as the idea of special creation for
40 each species. Anthropologists once depicted human evolution as a version of the classic
"March of Progress" image—a straight line from a crouching proto-ape, through
successive stages of knuckle draggers and culminating in upright modern human beings.
"It was a fairly simple picture, but it was a simplicity born of ignorance; says biological
anthropologist William Jungers of Stony Brook University in New York "The last 30 years
45 have seen an explosion of new finds."
Asked about gaps in Darwin's knowledge, Francisco Ayala, a biologist at the
University of California at Irvine, laughs. "That's easy" he says. "Darwin didn't know 99
percent of what we know." Which may sound bad, Ayala goes on, but "the 1 percent he
did know was the most important part"

1. Over the course of the passage, the main D. the initial reception for Darwin’s work
focus shifts from to a broader discussion of how his
A. a description of Darwin’s life to an findings continue to guide scientific
overview of Darwin’s published works. research.
B. detailed criticism of Darwin’s
controversial theory to qualified 2. Which choice provides the best evidence
support for that theory. for the answer to the previous question?
C. Darwin’s explanation of a scientific A. Lines 3–5 (“On the...copies”)
mystery to a summary of how other B. Lines 31–32 (“In Darwin’s...Mendel”)
scientists facilitated that mystery’s C. Lines 32–33 (“The objection...time”)
resolution. D. Lines 45–46 (“The last...finds”)

166 | I U P I T B
3. The author most strongly suggests that the C. He presents a criticism from an expert
largest reason Darwin’s intellectual to disprove the theory presented by
competitors took issue with his work was Darwin.
that it D. He undermines the importance of
A. didn’t present a complete explanation Darwin’s theory by discussing the work
of the hypothesized phenomenon. of another scientist.
B. presented ideas that didn’t match
what the church believed. 8. As used in line 44, “born of” most nearly
C. offended readers with its absurd means
questions. A. carried by.
D. unified what had been intentionally B. generated from.
disparate ideas. C. possessed by.
D. admitted to.
4. Which choice provides the best evidence
for the answer to the previous question? 9. What purpose does the graph serve in
A. Lines 5–6 (“Allies relation to the passage as a whole?
applauded...breakthrough”) A. It connects the genetic findings of
B. Lines 6–8 (“scientific rivals...record”) Mendel with the biological findings of
C. Lines 9–11 (“In 1864...species”) Darwin.
D. Lines 12–13 (“I repudiate...theories”) B. It provides indisputable evidence to
prove the theories contained within
5. The main purpose of the reference to lions Darwin’s On the Origin of Species.
in line 21 is to C. It reinforces a statement from an
A. disprove a questionable theory. expert that much more has been
B. introduce a completely new idea. learned since Darwin’s scientific era.
C. reject a burgeoning controversy. D. It offers evidence that the fossils found
D. provide a clarifying example. by Darwin and his contemporaries
were different than the fossils found in
6. The author notes that those who criticized the next century.
Darwin’s work when it first came out were
A. misguided in attacking scientific 10. Which statement is best supported by the
discovery based solely on the work of data presented in the graph?
another scientist. A. Human fossils had been discovered in
B. transparent in their jealous slander at least five locations by the time
against his success. Darwin published On the Origin of
C. inaccurate because other scientists Species.
had already proven what Darwin had B. Human fossils discovered in Tanzania
not. were named more quickly than those
D. correct in their complaints that his found in Kenya.
theory lacked sufficient supporting C. Both Paranthropus species shown in
evidence. the graph were discovered in the same
country.
7. Which statement best describes the D. The more recently a new type of fossil
technique the author uses to advance the was discovered, the less time it took
main point of the third paragraph (lines 26– scientists to name the species.
30)?
A. He ponders an unproven possibility to
highlight the utility of Darwin’s
research.
B. He poses a question that puzzled those
of Darwin’s era to foreshadow a
forthcoming finding.

B E R A N I B E R J U A N G | 167
Questions 11-19 are based on the following passage.

A Lion’s Share of Luck


It’s the beginning of February, and as they do every year, thousands of people line H Street,
the heart of Chinatown in Washington, DC. The crowd has gathered to celebrate Lunar New Year.
The street is a sea of (11) red. Red is the traditional Chinese color of luck and happiness. Buildings
are (12) draped with festive, red, banners, and garlands. Lampposts are strung with crimson paper
lanterns, which bob in the crisp winter breeze. The eager spectators await the highlight of the New
Year parade: the lion dance.
Experts agree that the lion dance originated in the Han dynasty (206 BCE–220 CE); however,
there is little agreement about the dance’s original purpose. Some evidence suggests that the
earliest version of the dance was an attempt to ward off an evil spirit; (13) lions are obviously very
fierce. Another theory is that an emperor, upon waking from a dream about a lion, hired an artist
to choreograph the dance. (14) The current function of the dance is celebration.

11. Which choice most effectively combines 14. Which choice most effectively concludes
the sentences at the underlined portion? the paragraph?
A. red, A. NO CHANGE
B. red; in addition, red is B. It turns out that the origins of the lion
C. red; in other words, red is dance are irrelevant.
D. red, the color; that is C. Whatever its origins, today the lion
dance is a joyous spectacle, a
12. celebration of the promise of the New
A. NO CHANGE Year.
B. draped, with festive red banners, D. Things are different these days, of
C. draped with festive red banners— course.
D. draped with festive red banners

13. Which choice most effectively completes


the explanation of a possible origin of the
lion dance?
A. NO CHANGE
B. the evil spirit was called Nian.
C. villagers dressed in lion costumes to
scare the spirit away.
D. the precise location of the village
remains lost to history.

The lion dance requires the strength, grace, and coordination of two dancers, (15) both of
whom are almost completely hidden by the elaborate bamboo and papier-mâché lion costume that
they maneuver. One person operates the lion’s head as the other guides the torso and tail. Many
of the moves in the dance, such as jumps, rolls, and kicks, are similar to (16) martial arts and
acrobatics. The dancers must be synchronized with the music accompanying the dance—drums,
cymbals, and gongs that supply the lion’s roar—as well as with each other.

15. 16.
A. NO CHANGE A. NO CHANGE
B. of which both B. the disciplines of martial arts and
C. both of them acrobatics.
D. both C. martial artists and acrobats.
D. those in martial arts and acrobatics.

168 | I U P I T B
1
While there are many regional variations of the lion dance costume, all make extensive use
of symbols and colors. 2 The lion’s head is often adorned with a phoenix (17) (a mythical bird) or a
tortoise (for longevity). 3 Green lions encourage friendliness. [4] Golden and red lions represent
liveliness and bravery, respectively. [5] Their older counterparts, yellow and white lions, dance
more slowly and deliberately. [6] In some variations, lions of different colors are different ages, and
they move accordingly. [7] Black lions are the youngest; therefore, they dance quickly and playfully.
[8] The appearance of the lions varies, but their message is consistent: Happy New Year.

17. Which choice provides information that is most consistent in style and content with the
information about the symbolism of the tortoise?
A. NO CHANGE
B. (for new beginnings)
C. (from Chinese mythology)
D. (for symbolic reasons)

As the parade winds its way through Chinatown, the music crescendos, and the lion dance
reaches (18) it’s climax with the “plucking of the greens.” Approaching a doorway in which dangles
a red envelope filled with green paper money, the (19) lion’s teeth snare the envelope. It then
chews up the bills and spits out the money-filled envelope instead of chewing it up. The crowd
cheers for the lion dancers and for the prosperity and good fortune their dance foretells.

18. 19.
A. NO CHANGE A. NO CHANGE
B. its B. lion snares the envelope with its teeth.
C. there C. envelope is snared by the lion with its
D. their teeth.
D. teeth of the lion snare the envelope.

Questions 20-25 are based on the following passage.


A cross selection of children were interviewed, and they matter of factly ---(20)--- problems at the
same time --- (21)--- showing the way to ---(22)--- solutions. Tell your children about your work.
Young as they are, they ---(23)--- where you have been all day. Explain to them Mummy has a
teaching job. She teaches little children to read and write or Daddy works in a factory. The factory
makes scooters. At least now the child ---(24)--- a mental picture of how you spend the day. There
develops a certain ---(25)--- in that knowledge.

20. In each of the following passages, there are 22. In each of the following passages, there are
blanks each of which has been numbered. blanks each of which has been numbered.
Find out the appropriate words. Find out the appropriate words.
A. designated A. referred
B. pin-pointed B. remarkable
C. declared C. reasonable
D. marked D. plausible

21. In each of the following passages, there are 23. In each of the following passages, there are
blanks each of which has been numbered. blanks each of which has been numbered.
Find out the appropriate words. Find out the appropriate words.
A. painstakingly A. estimates
B. obviously B. feel
C. unknowingly C. amaze
D. inadvertently D. wonder

B E R A N I B E R J U A N G | 169
24. In each of the following passages, there are 25. In each of the following passages, there are
blanks each of which has been numbered. blanks each of which has been numbered.
Find out the appropriate words. Find out the appropriate words.
A. establishes A. restfulness
B. develops B. solace
C. inculcates C. assurance
D. constructs D. security

26. Which of the following words is most B. dig


similar in meaning to reputable: C. machinery
A. established D. dread
B. build E. doom
C. create
D. antique 31. Kale has more nutritional value than
E. fresh spinach. But since collard greens have
more nutritional value than lettuce, if
27. DRIVE is to LICENCE as BREATHE is to: follows that kale has more nutritional value
A. oxygen than lettuce.
B. atmosphere Any of the following, if introduced into the
C. windpipe argument as an additional premise, makes
D. inhale the argument above logically correct
E. None of these EXCEPT:
A. Collard greens have more nutritional
28. Five mothers are comparing their babies’ value than kale
weights. Bert is heavier than Annie, but B. Spinach has more nutritional value
lighter than Judy. Sally is also heavier than than lettuce
Annie, but not as heavy as Bert. Rob turns C. Spinach has more nutritional value
out to be just a little heavier than both Bert than collard greens
and Judy. D. Spinach and collard greens have the
Who is the second heaviest? same nutritional value
A. Bert E. Kale and collard greens have the same
B. Sally nutritional value
C. Annie
D. Judy 32. On the basis of a decrease in the college-
E. Rob age population, many colleges now
anticipate increasingly smaller freshman
29. EDUCATION is to CLASSROOM as classes each year. Surprised by a 40
GROCERIES are to: percent increase in qualified applicants
A. milk over the previous year, however,
B. fruit administrators at Nice College now plan to
C. supermarket hire more faculties for courses taken by all
D. list freshmen.
E. None of these Which of the following statements about
Nice College's current qualified applicants,
30. Which of the following words is most if true, would strongly suggest that the
similar in meaning to slog: administrators' plan is flawed?
A. drudgery
170 | I U P I T B
A. A substantially higher percentage than Which of the following, if true, would
usual plan to study for advanced explain the discrepancy described above?
degrees after graduation from college. A. The numbers on the next milepost had
B. According to their applications, their been reversed.
level of participation in extracurricular B. The numbers on the mileposts indicate
activities and varsity sports is kilometers, not miles.
unusually high. C. The facing numbers indicate miles to
C. According to their applications, none the end of the path, not miles from the
of them lives in a foreign country. beginning.
D. A substantially lower percentage than D. A milepost was missing between the
usual rate Nice College as their first two the hiker encountered.
choice among the colleges to which E. The mileposts had originally been put
they are applying in place for the use of mountain bikers,
E. A substantially lower percentage than not for hikers.
usual list mathematics as their
intended major. 35. Airline: Newly developed collision-
avoidance systems, although not fully
33. A researcher discovered that people who tested to discover potential malfunctions,
have low levels of immune-system activity must be installed immediately in passenger
tend to score much lower on tests of planes. Their mechanical warnings enable
mental health than do people with normal pilots to avoid crashes.
or high immune-system activity. The Pilots: Pilots will not fly in planes with
researcher concluded from this experiment collision-avoidance systems that are not
that the immune system protects against fully tested. Malfunctioning systems could
mental illness as well as against physical mislead pilots, causing crashes.
disease. The pilots' objection is most strengthened
The researcher's conclusion depends on if which of the following is true?
which of the following assumptions? A. It is always possible for mechanical
A. High immune-system activity protects devices to malfunction.
against mental illness better than B. Jet engines, although not fully tested
normal immune-system activity does. when first put into use, have achieved
B. Mental illness is similar to physical exemplary performance and safety
disease in its effects on body systems. records.
C. People with high immune-system C. Although collision-avoidance systems
activity cannot develop mental illness. will enable pilots to avoid some
D. Mental illness does not cause people's crashes, the likely malfunctions of the
immune-system activity to decrease. not-fully-tested systems will cause
E. Psychological treatment of mental even more crashes.
illness is not as effective as is medical D. Many airline collisions are caused in
treatment. part by the exhaustion of overworked
pilots.
34. A milepost on the towpath read "21" on the E. Collision-avoidance systems, at this
side facing the hiker as she approached it stage of development, appear to have
and "23" on its back. She reasoned that the worked better in passenger planes
next milepost forward on the path would than in cargo planes during
indicate that she was halfway between one experimental flights made over a six-
end of the path and the other. However, month period.
the milepost one mile further on read "20"
facing her and "24" behind.

B E R A N I B E R J U A N G | 171
36. (3𝑥 4 + 2𝑥 3 − 7) + (4𝑥 6 − 5𝑥 3 + 9) 40. 0 = 7𝑦 − 5𝑥 + 9
Which of the following expressions is What are the 𝑦-intercept and the slope of
equivalent to the expression above? the line defined by the equation above?
A. 4𝑥 6 + 3𝑥 4 – 3𝑥 3 + 2 A. The slope is − , and the 𝑦-intercept is
5
7
B. 4𝑥 6 + 3𝑥 4 + 7𝑥 3 + 2 9
− .
C. 4𝑥 6 + 3𝑥 4 – 5𝑥 3 + 2 7
5
D. 7𝑥 10 – 3𝑥 3 + 2 B. The slope is − , and the 𝑦-intercept is
7
9
.
37. The lines graphed in the 𝑥𝑦-plane above 7
5
represent a system of two linear equations. C. The slope is , and the 𝑦-intercept is
7
9
What is the solution (𝑥, 𝑦) to the system? − .
7
```` 5 9
D. The slope is , and the 𝑦-intercept is .
7 7

1
41. If 5 + 𝑛 = 9 − 𝑛, what is the value of 𝑛 ?
3
A. 3
B. 4
C. 6
21
D.
2

42. Line 𝑝 (not shown) is perpendicular to line


𝑞 shown above and passes through the
point (0, 4). Which of the following
A. (– 1, – 6)
equations could represent line 𝑝?
B. (0, – 3)
C. (2, 3)
D. (3, 0)

38. Rosa has already eaten 10 pretzels from a


bag that originally contained 𝑝 pretzels. If
Rosa is able to eat each remaining pretzel
in 18 seconds, which of the following
represents the amount of additional time,
in seconds, needed for Rosa to eat all the
4
pretzels in the bag? A. 𝑦 = − 𝑥 + 4
3
A. 10(18 – 𝑝) 3
B. 𝑦=− 𝑥+4
B. 10(𝑝 – 18) 3
4

C. 18(10 – 𝑝) C. 𝑦 = 𝑥+4
4
4
D. 18(𝑝 – 10) D. 𝑦 = 𝑥 + 4
3

39. (4 + 7𝑖) − (6 + 2𝑖) 4 5


What complex number is equivalent to the 43. =
𝑛−3 𝑛+2
expression above if? Given the equation above, what is the
A. 2 value of 𝑛?
B. 2 – 5𝑖 A. –7
C. – 2 + 5𝑖 B. –2
D. – 10 – 9𝑖 C. 8
D. 23
172 | I U P I T B
5
44. What is the solution set to the equation A. 3𝑎6 √3
0 = (3𝑎 + 1)2 (𝑎 – 4) ? B.
5
−3𝑎6 √81
1 5
A. { , −4}
3
C. 3𝑎5 √81
5
B. {− , 4}
1 D. −3𝑎5 √3
3
1 1
C. {− , , −4} 50. The amount of carbon-15 in a given sample
3 3
1 1 decays exponentially with time. If the
D. {− , 4}
3 3
1 24𝑚
function 𝐶(𝑚) = 100 ( ) models the
2
45. What is the solution set to the equation amount of carbon-15 remaining in the
2 4 5−𝑚
= − ? sample after 𝑚 minutes, which of the
7−𝑚 𝑚 7−𝑚
A. {4, 7} following must be true?
B. {4, 5} A. The amount of carbon in the sample
C. {1, 7} halves every minute.
D. {4} B. The amount of carbon in the sample
halves every 24 minutes.
46. If 3 is a root of the function 𝑓(𝑥) = 𝑥 2 + C. The amount of carbon in the sample
13𝑥 + 𝑐 and 𝑐 is a constant, what is the halves 24 times every minute.
value of 𝑐? D. The amount of carbon in the sample
A. –48 reduces by a factor of 24 every 2
B. –3 minutes
C. 5
D. 48 51. If the function 𝑔 is defined by 𝑔(𝑥) =
3𝑥 + 5, what is the value of 𝑔(– 5) ?
7 3 A. −20
47. −
12𝑏 2 4𝑏 2
B. −10
The above expression is equivalent to
C. 20
which of the following expressions for all
D. 60
𝑏 > 0?
1
A. − 52. According to the table above, 100-Watt
6𝑏 3
1
B. − bulbs made up what fraction of the
4𝑏 3
1 working lightbulbs?
C.
4𝑏 3 Working Defective Total
1
D. 60- 1,230 127 1,357
6𝑏 3
Watt
100- 2,384 271 2,655
48. 𝑦 = 𝑥 2 + 3
Watt
𝑦 = 15𝑥 − 33 Total 3,614 398 4,012
The system of equations shown above is 1,230
A.
graphed in the 𝑥𝑦- plane. If system has two 3,614
2,384
solutions, what is the product of the 𝑥- B.
3,614
coordinates of the two solutions? 271
C.
398
A. 36 2,384
B. 4 D.
2,655
C. -4
D. -36 53. The expression (4𝑛 – 5)(5𝑛 – 4) is
equivalent to which of the following?
3
A. 20𝑛2 – 41𝑛 + 20
49. (−27𝑎10 )5
B. 20𝑛2 – 39𝑛 + 9
For all values of 𝑎, which of the following is
C. 9𝑛2 – 41𝑛 + 20
equivalent to the expression above?
D. 4𝑛2 – 18𝑛 + 9
B E R A N I B E R J U A N G | 173
2.7
54. The ratio of is equivalent to the ratio of Questions 58–60 refer to the following
1.2
𝑏 information.
. What is the value of 𝑏 ?
4.8 Thomas was hired for a new job in 1977, with a
A. 2.13 starting salary of $40,000. Beginning in 1978,
B. 4 Thomas received an annual raise, increasing his
C. 6.3 salary by $2,300 each year.
D. 10.8
58. If Thomas retired at the end of 1999, what
55. 60 = 15𝑚𝑛 + 20 was his salary in his final year?
What is the value of 3𝑚𝑛 + 4, according A. $90,600
to the equation above? B. $76,000
A. 20 C. $54,600
B. 15 D. $40,000
C. 12
D. 4 59. Which of the following must be true, given
that Thomas’s salary after 𝑦 years was
56. A high school principal is seeking to between $54,000 and $60,000 ?
determine the likelihood that students in A. 3 < 𝑦 < 6
Santana High School will attend the B. 6 < 𝑦 < 9
upcoming dance. Which of the following C. 9 < 𝑦 < 12
data collection methods is most likely to D. 𝑦 > 12
yield an accurate prediction by the
principal? 60. Which of the following graphs could
A. Polling a randomly selected group of represent Thomas’s salary, 𝑆, in dollars, as
1,500 teenagers in the town a function of the number of years, 𝑦, after
B. Conducting a survey of 180 randomly 1977 ?
selected students in the senior class at
Santana High School
C. Polling a group of 250 randomly
selected Santana High School students
D. Posting an internet poll on the school’s
website open only to Santana High A.
School Students

57. An investor is deciding between two


options for a short-term investment. One
option has a return 𝑅, in dollars, 𝑡 months
after investment, and is modelled by the B.
equation 𝑅 = 100(3𝑡 ). The other option
has a return 𝑅, in dollars, 𝑡 months after
investment, and is modeled by the
equation 𝑅 = 350𝑡. After 4 months, how
much less is the return given by the linear
model than the return given by the C.
exponential model?
A. $1,400
B. $4,050
C. $6,700
D. $8,100 D.

174 | I U P I T B
61. 𝑸𝒖𝒂𝒏𝒕𝒊𝒕𝒚 𝑨 𝑸𝒖𝒂𝒏𝒕𝒊𝒕𝒚 𝑩
(3−1 + 3−2 )−1 1
A. Quantity A is greater.
B. Quantity B is greater.
C. The two quantities are equal.
D. The relationship cannot be determined from the information given.

62. 𝑥 2 − 3𝑥 = 18
𝑸𝒖𝒂𝒏𝒕𝒊𝒕𝒚 𝑨 𝑸𝒖𝒂𝒏𝒕𝒊𝒕𝒚 𝑩

𝑇ℎ𝑒 𝑔𝑟𝑒𝑎𝑡𝑒𝑠𝑡 𝑝𝑜𝑠𝑠𝑖𝑏𝑙𝑒 𝑣𝑎𝑙𝑢𝑒 𝑜𝑓 𝑥 3


A. Quantity A is greater.
B. Quantity B is greater.
C. The two quantities are equal.
D. The relationship cannot be determined from the information given.

63.

𝑸𝒖𝒂𝒏𝒕𝒊𝒕𝒚 𝑨 𝑸𝒖𝒂𝒏𝒕𝒊𝒕𝒚 𝑩

𝑇ℎ𝑒 𝑙𝑒𝑛𝑔𝑡ℎ 𝑜𝑓 𝑠𝑖𝑑𝑒 𝑃𝑅 15 𝑖𝑛𝑐ℎ𝑒𝑠


A. Quantity A is greater.
B. Quantity B is greater.
C. The two quantities are equal.
D. The relationship cannot be determined from the information given.

64. The circles have radii of 𝒙, 𝒚, and 𝒙 + 𝒚 units, respectively


𝑸𝒖𝒂𝒏𝒕𝒊𝒕𝒚 𝑨 𝑸𝒖𝒂𝒏𝒕𝒊𝒕𝒚 𝑩

𝑇ℎ𝑒 𝑠𝑢𝑚 𝑜𝑓 𝑡ℎ𝑒 𝑎𝑟𝑒𝑎𝑠 𝑜𝑓 𝑇ℎ𝑒 𝑎𝑟𝑒𝑎 𝑜𝑓 𝑡ℎ𝑒 𝑙𝑎𝑟𝑔𝑒𝑠𝑡 𝑐𝑖𝑟𝑐𝑙𝑒


𝑇ℎ𝑒 𝑡𝑤𝑜 𝑠𝑚𝑎𝑙𝑙𝑒𝑟 𝑐𝑖𝑟𝑐𝑙𝑒𝑠
A. Quantity A is greater.
B. Quantity B is greater.
C. The two quantities are equal.
D. The relationship cannot be determined from the information given.

65. A child ate 1 less than 25% of the 28 apples his father purchased
𝑸𝒖𝒂𝒏𝒕𝒊𝒕𝒚 𝑨 𝑸𝒖𝒂𝒏𝒕𝒊𝒕𝒚 𝑩

𝑇ℎ𝑒 𝑛𝑢𝑚𝑏𝑒𝑟 𝑜𝑓 𝑎𝑝𝑝𝑙𝑒𝑠 8


𝑇ℎ𝑒 𝑐ℎ𝑖𝑙𝑑 𝑎𝑡𝑒
A. Quantity A is greater.
B. Quantity B is greater.
C. The two quantities are equal.
D. The relationship cannot be determined from the information given.

B E R A N I B E R J U A N G | 175
66. Find the mean of the data.
1. The data has 8 data values
2. The data is 3, 4, 5, 6, 4, 1, 0, 5
A. Statement (1) ALONE is sufficient, but statement (2) alone is not sufficient.
B. Statement (2) ALONE is sufficient, but statement (1) alone is not sufficient.
C. BOTH statements TOGETHER are sufficient, but NEITHER statement ALONE is sufficient.
D. EACH statement ALONE is sufficient.
E. Statements (1) and (2) TOGETHER are NOT sufficient.

67. Determine the volume of a cuboids.


1. The length is twice the width and the height is 4 inches.
2. The length is 6 inches.
A. Statement (1) ALONE is sufficient, but statement (2) alone is not sufficient.
B. Statement (2) ALONE is sufficient, but statement (1) alone is not sufficient.
C. BOTH statements TOGETHER are sufficient, but NEITHER statement ALONE is sufficient.
D. EACH statement ALONE is sufficient.
E. Statements (1) and (2) TOGETHER are NOT sufficient.

68. Find the value of r if 4𝑟 + 2𝑡 = 14.


1. 𝑡 = 2
2. 𝑟 > 𝑡
A. Statement (1) ALONE is sufficient, but statement (2) alone is not sufficient.
B. Statement (2) ALONE is sufficient, but statement (1) alone is not sufficient.
C. BOTH statements TOGETHER are sufficient, but NEITHER statement ALONE is sufficient.
D. EACH statement ALONE is sufficient.
E. Statements (1) and (2) TOGETHER are NOT sufficient.

69. Find the common difference of the arithmetic sequence.


1. The third term of the sequence is 1.428.
2. The first and the fifth terms of the sequence is 1 and 1.856.
A. Statement (1) ALONE is sufficient, but statement (2) alone is not sufficient.
B. Statement (2) ALONE is sufficient, but statement (1) alone is not sufficient.
C. BOTH statements TOGETHER are sufficient, but NEITHER statement ALONE is sufficient.
D. EACH statement ALONE is sufficient.
E. Statements (1) and (2) TOGETHER are NOT sufficient.

70. 700, 1200, 1600, 2000, 𝑥


If the mean of the five numbers above is 1600, what is the value of 𝑥 ?

71. The relationship between 𝑥 and 𝑦 can be written as 𝑦 = 𝑚𝑥, where 𝑚 is a constant. If 𝑦 =
17 when 𝑥 = 𝑎, what is the value of 𝑦 when 𝑥 = 2𝑎 ?

72. 𝑎(𝑥 + 𝑏) = 4𝑥 + 10
In the equation above, 𝑎 and 𝑏 are constants. If the equation has infinitely many solutions for
𝑥, what is the value of 𝑏 ?

73. In the 𝑥𝑦-plane, a line that has the equation 𝑦 = 𝑐 for some constant 𝑐 intersects a parabola
at exactly one point. If the parabola has the equation 𝑦 = −𝑥 2 + 5𝑥 , what is the value of 𝑐
?

176 | I U P I T B
Questions 74 and 75 refer to the following information.
The peregrine falcon can reach speeds of up to 200 miles per hour while diving to catch prey,
making it the fastest animal on the planet when in a dive

74. What is a peregrine falcon’s maximum speed while diving to catch prey, in feet per second?
(Round your answer to the nearest whole number. (1 mile = 5280 feet)

75. If a peregrine falcon dove at its maximum speed for half a mile to catch prey, how many
seconds would the dive take? (Round your answer to the nearest second.)

B E R A N I B E R J U A N G | 177
“Work Hard, Dream Bigger and
Be The Best”

178 | I U P I T B

You might also like